You are on page 1of 228

River Publishers Series in Mathematical, Statistical and

Elementary Vector Calculus Computational Modelling for Engineering

and its Applications with


MATLAB Programming Elementary Vector Calculus

Elementary Vector Calculus and its Applications with


Nita H. Shah and its Applications with
Jitendra Panchal
MATLAB Programming

MATLAB Programming
Sir Isaac Newton, one of the greatest scientists and mathematicians
of all time, introduced the notion of a vector to define the existence of
gravitational forces, the motion of the planets around the sun, and the
motion of the moon around the earth. Vector calculus is a fundamental
scientific tool that allows us to investigate the origins and evolution of
space and time, as well as the origins of gravity, electromagnetism, and
nuclear forces. Vector calculus is an essential language of mathemati-
cal physics, and plays a vital role in differential geometry and studies
related to partial differential equations widely used in physics, engi-
neering, fluid flow, electromagnetic fields, and other disciplines. ­Vector
calculus represents physical quantities in two or three-­dimensional
space, as well as the variations in these quantities.
The machinery of differential geometry, of which vector calculus
is a subset, is used to understand most of the analytic results in a more
general form. Many topics in the physical sciences can be mathemati-
cally studied using vector calculus techniques.
This book is designed under the assumption that the readers have
no prior knowledge of vector calculus. It begins with an introduction to
vectors and scalars, and also covers scalar and vector products, vector
differentiation and integrals, Gauss’s theorem, Stokes’s theorem, and
Green’s theorem. The MATLAB programming is given in the last chapter.
This book includes many illustrations, solved examples, practice
examples, and multiple-choice questions.

Jitendra Pancha
Nita H. Shah
Nita H. Shah
River Publishers River Jitendra Panchal
Elementary Vector Calculus
and its Applications with
MATLAB Programming
RIVER PUBLISHERS SERIES IN MATHEMATICAL,
STATISTICAL AND COMPUTATIONAL MODELLING
FOR ENGINEERING
Series Editors:

MANGEY RAM
Graphic Era University, India
TADASHI DOHI
Hiroshima University, Japan
ALIAKBAR MONTAZER HAGHIGHI
Prairie View Texas A&M University, USA
Applied mathematical techniques along with statistical and computational data analysis has
become vital skills across the physical sciences. The purpose of this book series is to present
novel applications of numerical and computational modelling and data analysis across the
applied sciences. We encourage applied mathematicians, statisticians, data scientists and
computing engineers working in a comprehensive range of research fields to showcase dif-
ferent techniques and skills, such as differential equations, finite element method, algorithms,
discrete mathematics, numerical simulation, machine learning, probability and statistics, fuzzy
theory, etc
Books published in the series include professional research monographs, edited vol-
umes, conference proceedings, handbooks and textbooks, which provide new insights for
researchers, specialists in industry, and graduate students.
Topics included in this series are as follows:-
• Discrete mathematics and computation
• Fault diagnosis and fault tolerance
• Finite element method (FEM) modeling/simulation
• Fuzzy and possibility theory
• Fuzzy logic and neuro-fuzzy systems for relevant engineering applications
• Game Theory
• Mathematical concepts and applications
• Modelling in engineering applications
• Numerical simulations
• Optimization and algorithms
• Queueing systems
• Resilience
• Stochastic modelling and statistical inference
• Stochastic Processes
• Structural Mechanics
• Theoretical and applied mechanics
For a list of other books in this series, visit www.riverpublishers.com
Elementary Vector Calculus
and its Applications with
MATLAB Programming

Nita H. Shah
Gujarat University, India

Jitendra Panchal
Parul University, India
Published, sold and distributed by:
River Publishers
Alsbjergvej 10
9260 Gistrup
Denmark

www.riverpublishers.com

ISBN: 978-87-7022-387-4 (Hardback)


978-87-7022-386-7 (Ebook)


c 2022 River Publishers

All rights reserved. No part of this publication may be reproduced, stored in


a retrieval system, or transmitted in any form or by any means, mechanical,
photocopying, recording or otherwise, without prior written permission of
the publishers.
Contents

Preface ix

List of Figures xi

1 Basic Concept of Vectors and Scalars 1


1.1 Introduction and Importance . . . . . . . . . . . . . . . . . 1
1.2 Representation of Vectors . . . . . . . . . . . . . . . . . . . 1
1.3 Position Vector and Vector Components . . . . . . . . . . . 2
1.4 Modulus or Absolute Value of a Vector . . . . . . . . . . . . 3
1.5 Zero Vector and Unit Vector . . . . . . . . . . . . . . . . . 4
1.6 Unit Vectors in the Direction of Axes . . . . . . . . . . . . 4
1.7 Representation of a Vector in terms of Unit Vectors . . . . . 5
1.8 Addition and Subtraction of Vectors . . . . . . . . . . . . . 6
1.9 Product of a Vector with a Scalar . . . . . . . . . . . . . . . 6
1.10 Direction of a Vector . . . . . . . . . . . . . . . . . . . . . 7
1.11 Collinear and Coplanar Vectors . . . . . . . . . . . . . . . . 8
1.11.1 Collinear Vectors . . . . . . . . . . . . . . . . . . . 8
1.11.2 Coplanar Vectors . . . . . . . . . . . . . . . . . . . 8
1.12 Geometric Representation of a Vector Sum . . . . . . . . . . 8
1.12.1 Law of Parallelogram of Vectors . . . . . . . . . . . 8
1.12.2 Law of Triangle of Vectors . . . . . . . . . . . . . . 9
1.12.3 Properties of Addition of Vectors . . . . . . . . . . . 9
1.12.4 Properties of Scalar Product . . . . . . . . . . . . . 10
1.12.5 Expression of Any Vector in Terms of the Vectors
Associated with its Initial Point and Terminal Point . 10
1.12.6 Expression of Any Vector in Terms of Position
Vectors . . . . . . . . . . . . . . . . . . . . . . . . 11
1.13 Direction Cosines of a Vector . . . . . . . . . . . . . . . . . 12
1.14 Exercise . . . . . . . . . . . . . . . . . . . . . . . . . . . . 26

v
vi Contents

2 Scalar and Vector Products 29


2.1 Scalar Product, or Dot Product, or Inner Product . . . . . . . 29
2.2 The Measure of Angle Between two Vectors and
Projections . . . . . . . . . . . . . . . . . . . . . . . . . . 30
2.2.1 Properties of a Dot Product . . . . . . . . . . . . . . 30
2.3 Vector Product or Cross Product or Outer Product of Two
Vectors . . . . . . . . . . . . . . . . . . . . . . . . . . . . 37
2.4 Geometric Interpretation of a Vector Product . . . . . . . . . 38
2.4.1 Properties of a Vector Product . . . . . . . . . . . . 39
2.5 Application of Scalar and Vector Products . . . . . . . . . . 45
2.5.1 Work Done by a Force . . . . . . . . . . . . . . . . 46
2.5.2 Moment of a Force About a Point . . . . . . . . . . 46
2.6 Exercise . . . . . . . . . . . . . . . . . . . . . . . . . . . . 52

3 Vector Differential Calculus 55


3.1 Introduction . . . . . . . . . . . . . . . . . . . . . . . . . . 55
3.2 Vector and Scalar Functions and Fields . . . . . . . . . . . . 55
3.2.1 Scalar Function and Field . . . . . . . . . . . . . . 56
3.2.2 Vector Function and Field . . . . . . . . . . . . . . 56
3.2.3 Level Surfaces . . . . . . . . . . . . . . . . . . . . 56
3.3 Curve and Arc Length . . . . . . . . . . . . . . . . . . . . . 57
3.3.1 Parametric Representation of Curves . . . . . . . . . 57
3.3.2 Curves with Tangent Vector . . . . . . . . . . . . . 58
3.3.2.1 Tangent Vector . . . . . . . . . . . . . . . 59
3.3.2.2 Important Concepts . . . . . . . . . . . . 60
3.3.3 Arc Length . . . . . . . . . . . . . . . . . . . . . . 61
3.3.3.1 Unit Tangent Vector . . . . . . . . . . . . 61
3.4 Curvature and Torsion . . . . . . . . . . . . . . . . . . . . . 64
3.4.1 Formulas for Curvature and Torsion . . . . . . . . . 67
3.5 Vector Differentiation . . . . . . . . . . . . . . . . . . . . . 70
3.6 Gradient of a Scalar Field and Directional Derivative . . . . 73
3.6.1 Gradient of a Scalar Field . . . . . . . . . . . . . . 73
3.6.1.1 Properties of Gradient . . . . . . . . . . . 73
3.6.2 Directional Derivative . . . . . . . . . . . . . . . . 74
3.6.2.1 Properties of Gradient . . . . . . . . . . . 75
3.6.3 Equations of Tangent and Normal to the Level
Curves . . . . . . . . . . . . . . . . . . . . . . . . 84
3.6.4 Equation of the Tangent Planes and Normal Lines
to the Surfaces . . . . . . . . . . . . . . . . . . . . 85
Contents vii

3.7 Divergence and Curl of a Vector Field . . . . . . . . . . . . 86


3.7.1 Divergence of a Vector Field . . . . . . . . . . . . . 86
3.7.1.1 Physical Interpretation of Divergence . . . 86
3.7.2 Curl of a Vector Field . . . . . . . . . . . . . . . . . 89
3.7.2.1 Physical Interpretation of Curl . . . . . . . 89
3.7.3 Formulae for grad, div, curl Involving Operator ∇ . 96
3.7.3.1 Formulae for grad, div, curl Involving
Operator ∇ Once . . . . . . . . . . . . . 96
3.7.3.2 Formulae for grad, div, curl Involving
Operator ∇ Twice . . . . . . . . . . . . . 100
3.8 Exercise . . . . . . . . . . . . . . . . . . . . . . . . . . . . 104

4 Vector Integral Calculus 111


4.1 Introduction . . . . . . . . . . . . . . . . . . . . . . . . . . 111
4.2 Line Integrals . . . . . . . . . . . . . . . . . . . . . . . . . 111
4.2.1 Circulation . . . . . . . . . . . . . . . . . . . . . . 112
4.2.2 Work Done by a Force . . . . . . . . . . . . . . . . 112
4.3 Path Independence of Line Integrals . . . . . . . . . . . . . 113
4.3.1 Theorem: Independent of Path . . . . . . . . . . . . 113
4.4 Surface Integrals . . . . . . . . . . . . . . . . . . . . . . . 122
4.4.1 Flux . . . . . . . . . . . . . . . . . . . . . . . . . . 123
4.4.2 Evaluation of Surface Integral . . . . . . . . . . . . 123
4.4.2.1 Component form of Surface Integral . . . 124
4.5 Volume Integrals . . . . . . . . . . . . . . . . . . . . . . . 129
4.5.1 Component Form of Volume Integral . . . . . . . . 129
4.6 Exercise . . . . . . . . . . . . . . . . . . . . . . . . . . . . 131

5 Green’s Theorem, Stokes’ Theorem, and Gauss’ Theorem 135


5.1 Green’s Theorem (in the Plane) . . . . . . . . . . . . . . . . 135
5.1.1 Area of the Plane Region . . . . . . . . . . . . . . . 137
5.2 Stokes’ Theorem . . . . . . . . . . . . . . . . . . . . . . . 146
5.3 Gauss’ Divergence Theorem . . . . . . . . . . . . . . . . . 154
5.4 Exercise . . . . . . . . . . . . . . . . . . . . . . . . . . . . 163

6 MATLAB Programming 167


6.1 Basic of MATLAB Programming . . . . . . . . . . . . . . . 167
6.1.1 Basic of MATLAB Programming . . . . . . . . . . 167
6.1.1.1 Introductory MATLAB programmes . . . 168
6.1.1.2 Representation of a Vector in MATLAB . 183
viii Contents

6.1.1.3 Representation of a Matrix in MATLAB . 186


6.2 Some Miscellaneous Examples using MATLAB
Programming . . . . . . . . . . . . . . . . . . . . . . . . . 188

Index 207

About the Authors 213


Preface

Vector calculus is an essential language of mathematical physics. Vector


calculus plays a vital role in differential geometry, and the study related to
partial differential equations is widely used in physics, engineering, fluid
flow, electromagnetic fields, and other disciplines. Vector calculus represents
physical quantities in two or three-dimensional space, as well as the variations
in these quantities.
The machinery of differential geometry, of which vector calculus is a
subset, is used to understand most of the analytic results in a more general
form. Many topics in the physical sciences can be mathematically studied
using vector calculus techniques.
Description of the book:
This book is meant for readers who have a basic understanding of vector
calculus. This book is designed to provide accurate information to readers.
The language in the book is kept simple so that all readers can easily
understand each concept.
This book begins with the introduction of vectors and scalars in chapter 1.
Chapter 1 contains essential basic definitions and concepts, vector in terms of
unit vectors, geometric representation of vector sum, and direction cosines.
The scalar and vector products, measurement of angle and projections, geo-
metric interpretation of a vector product, and their applications are given in
chapter 2. In chapter 3, vector and scalar functions and fields, curves, arc
length, formulae for curvature and torsion, and its derivation, curl, diver-
gence, and gradient with important properties and physical interpretation, and
important results are given in vector differential calculus. Chapter 4 vector
integral calculus includes line integrals, circulation, path independence, sur-
face integrals, volume integrals, and its applications like flux and work done
by a force are given. In chapter 5, derivation of Green’s theorem, Stokes’s
theorem, and Gauss’ divergence theorem are given with various solve exam-
ples. MATLAB programming is given in the last chapter 6 includes basic
information about MATLAB. Initially, basic examples are given with proper

ix
x Preface

explanation wherever possible that helps readers to understand basic input


and output, arithmetic operations, functions, plotting commands available
in MATLAB. Variety of solved programs with MATLAB codes along with
compiles and debug outputs. So, the reader can run the program using given
codes and observe results.

For MATLAB product information, please contact:


The MathWorks, Inc.
3 Apple Hill Drive
Natick, MA 01760-2098 USA
Tel: 508-647-7000
Fax: 508-647-7001
E-mail: info@mathworks.com
Web: www.mathworks.com
List of Figures

Figure 1.1 Represents the geometrical representation


of a vector . . . . . . . . . . . . . . . . . . . . . . 1
Figure 1.2 Represents a position vector . . . . . . . . . . . . 2
Figure 1.3 Represents equal and negative vectors . . . . . . . 3
Figure 1.4 Represents unit vectors in xy-plane . . . . . . . . . 4
Figure 1.5 Represents unit vectors in the direction of x-axis,
y-axis, and z-axis . . . . . . . . . . . . . . . . . . 5
Figure 1.6 Represents a vector OP in terms of unit vectors . . 6
Figure 1.7 Represents the product of a vector with a scalar . . 7
Figure 1.8 Represents the law of parallelogram of vectors . . . 9
Figure 1.9 Represents the law of the triangle of vectors . . . . 9
Figure 1.10 Represents any vector in terms of various vectors
associated with its endpoints. . . . . . . . . . . . . 11
Figure 1.11 Represents any vector in terms of the position
vector . . . . . . . . . . . . . . . . . . . . . . . . 11
Figure 1.12 Represents direction cosines of a vector . . . . . . 12
Figure 1.13 Represents a parallelogram . . . . . . . . . . . . . 14
Figure 1.14 Represents a regular hexagon . . . . . . . . . . . . 15
Figure 1.15 Represents a triangle ΔABC . . . . . . . . . . . . 16
Figure 1.16 A space shuttle of 1000 tons weight hangs from two
skyscrapers using steel cables . . . . . . . . . . . . 25
Figure 2.1 Represents a scalar or dot product . . . . . . . . . 29
Figure 2.2 Represents a vector or cross product . . . . . . . . 38
Figure 2.3 Represents the geometric interpretation of a vector
or cross product . . . . . . . . . . . . . . . . . . . 39
Figure 2.4 Represents work done by a force F on a particle . . 46
Figure 2.5 Represents the moment of a force about a point . . 47
Figure 3.1 Represents a curve with a tangent vector . . . . . . 59
Figure 3.2 Represents the plane of curvature of the curve . . . 60
Figure 3.3 Represents the arc rate of rotation of binormal . . . 65
Figure 3.4 Represents B̂, T̂ , and N̂ orthogonal unit vectors . 66

xi
xii List of Figures

Figure 3.5 Represents the derivative of the vector v (t) . . . . 71


Figure 3.6 Represents the directional derivative . . . . . . . . 74
Figure 3.7 Represents the parallelopiped . . . . . . . . . . . . 87
Figure 4.1 Representation of a vector function defined at every
point of a curve C . . . . . . . . . . . . . . . . . . 111
Figure 4.2 Representation of a closed curve C . . . . . . . . . 114
Figure 4.3 Representation of parabola x = y 2 . . . . . . . . . 115
Figure 4.4 Representation of the rectangle in xy-plane
bounded by lines . . . . . . . . . . . . . . . . . . 117
Figure 4.5 Representation of curved surface S and a plane
region R . . . . . . . . . . . . . . . . . . . . . . . 122
Figure 4.6 Representation of the projection of the plane in the
first octant . . . . . . . . . . . . . . . . . . . . . . 124
Figure 4.7 Representation of the positive octant of the sphere . 126
Figure 4.8 Representation of the sphere . . . . . . . . . . . . 127
Figure 4.9 Representation of the cylinder in positive octant . . 129
Figure 5.1 Represents the region R bounded by the curve C . 135
Figure 5.2 Represents the region R bounded two parabolas . . 138
Figure 5.3 Represents the plane triangle enclosed by given
lines . . . . . . . . . . . . . . . . . . . . . . . . . 139
Figure 5.4 Represents the rectangle in the xy-plane bounded
by lines . . . . . . . . . . . . . . . . . . . . . . . 140
Figure 5.5 Represents the triangle in the xy-plane bounded by
lines . . . . . . . . . . . . . . . . . . . . . . . . . 142
Figure 5.6 Represents the region bounded by the parabola and
lines . . . . . . . . . . . . . . . . . . . . . . . . . 144
Figure 5.7 Represents an open surface bounded by a closed
curve C . . . . . . . . . . . . . . . . . . . . . . . 146
Figure 5.8 Represents the rectangle bounded by the lines . . . 149
Figure 5.9 Represents the surface of a rectangular lamina
bounded by the lines . . . . . . . . . . . . . . . . 152
Figure 5.10 Represents the boundary of the triangle . . . . . . 153
Figure 5.11 Represents the region bounded by a closed
surface S . . . . . . . . . . . . . . . . . . . . . . 155
Figure 5.12 Represents the cube . . . . . . . . . . . . . . . . . 157
Figure 5.13 Represents the cube . . . . . . . . . . . . . . . . . 159
Figure 5.14 Represents the circle . . . . . . . . . . . . . . . . 161
1
Basic Concept of Vectors and Scalars

1.1 Introduction and Importance


The word “Vector” was first given by W. R. Hamilton. In the nineteenth cen-
tury, Hamilton and Grassmann have formed vector analysis independently.
Today, all physical quantities are classified into two different quantities. The
physical quantities can be measured directly or indirectly. Some physical
quantities are independent of each other or dependent. A quantity that has
its value or a magnitude but no direction then it is called scalar quantity or
scalar. For example, time, temperature, density, mass, length, power, distance,
area, volume, speed, work, energy, electric charge, frequency, gravitational
potential, etc., in this list of examples all quantities are having magnitudes but
are independent of the direction. Whereas a quantity that has magnitude, as
well as direction, is known as vector quantity or vector. Velocity, acceleration,
magnetic field, force, momentum, lift, drag, thrust, displacement, fluid flow,
the intensity of an electrical field, centrifugal force, etc., are examples of
vector quantities. Vectors are generally denoted by capital bold letters or


letters with an arrow-like A , A, or a.

1.2 Representation of Vectors


A geometrical representation of a vector is given in Figure 1.1.
Let O be any arbitrary point in the space and let M be any point in
the space. A directed line segment joining
−−→both the points is known as the
−−→  
vector OM . The length of the vector OM  is the magnitude of the vector

Figure 1.1 Represents the geometrical representation of a vector

1
2 Basic Concept of Vectors and Scalars
−−→ −−−→
OM and is denoted by |OM | or|OM |. The point O is called the initial point
−−−→
and the point M is called the terminal point of the vector |OM |.

1.3 Position Vector and Vector Components


Consider the cartesian coordinate system shown in Figure 1.2. Let P be any
−−→
point in the three-dimensional system and let O be the origin then OP is
−−→
the position vector of the point P . If the vector OP is denoted by − →
a then


the point is denoted by P ( a). All three axis are perpendicular to each other.
The position vector can be obtained by taking perpendiculars on each axis. In
the cartesian coordinate system, AN (= OB) represents x-coordinate of P ,
BN (= OA) represents y-coordinate of P , and P N represents Z-coordinate
of P . Thus, the point P is denoted by P (x, y, z). Where x, y, and z are also
known as components in the direction of the X-axis, Y -axis, and Z-axis
respectively.
Equal Vectors: Two vectors with the same direction and magnitude are called
equal vectors irrespective of the position of their initial points. In Figure 1.3,

− →

a and b are equal vectors.
Negative Vectors: Two vectors with the same magnitude but opposite in
direction are called negative vectors. In Figure 1.3, −

c is a negative vector

− →

for both a and b .

Figure 1.2 Represents a position vector


1.4 Modulus or Absolute Value of a Vector 3

Figure 1.3 Represents equal and negative vectors

1.4 Modulus or Absolute Value of a Vector


−−→
In Figure 1.2, OP is the position vector of the point P (x, y, z). The modulus
−−→ −−→
or absolute value of a vector OP is the length of the vector OP .
i.e., OP 2 = BM 2 + M A2 + P M 2 = x2 + y 2 + z 2
−−→ 
 
∴ OP  = OP = x2 + y 2 + z 2

Illustration 1.1: Find the modulus of the vector (−3, 4, −5).


Solution: Let −→
a = (−3, 4, −5) be a given vector. It is a three-dimensional
vector. Here, x = −3, y = 4, and z = −5 then the modulus of the vector −→a
is given by
 
|→

a|= x + y + z = (−3)2 + (4)2 + (−5)2
2 2 2

√ √
= 9 + 16 + 25 = 50

=5 2

Thus, the modulus of a vector −

a is 5 2.
Illustration 1.2: Find the modulus of the vector (6, 8).
Solution: Let −→
a = (6, 8) be a given vector. It is a two-dimensional vector.
Here, x = 6 and y = 8 then the modulus of the vector − →a is given by
 
|−

a | = x2 + y 2 = (6)2 + (8)2
√ √
= 36 + 64 = 100
= 10
Thus, the modulus of a vector −

a is 10.
4 Basic Concept of Vectors and Scalars

1.5 Zero Vector and Unit Vector


Zero Vector: A vector with modulus zero is called a√zero vector. It is denoted
by 0 or θ. Here, θ = 0 = (0, 0, 0). So, |θ| = |0| = 02 + 02 + 02 = 0. Note
−→ −−→
that |θ| = 0. AA, BB etc. are zero vectors.
Unit Vector: A vector with modulus unity (i.e., 1) is called a unit vector.
Illustration 1.3: (−1, 0, 0) is a unit vector as its modulus is

(−1)2 + 02 + 02 = 1.
  
Illustration 1.4: √13 , 0, − 23 is a unit vector as its modulus is
 2

 1 2
2 1 2 1+2 3
√ + 02 + − = +0+ = = = 1.
3 3 3 3 3 3

1.6 Unit Vectors in the Direction of Axes


Figure 1.4 represents a two-dimensional cartesian coordinate system (i.e., xy-
plane), in which (1, 0) and (0, 1) are called unit vectors in the direction of
x and y axis respectively. The unit vector in the direction of x-coordinate
is denoted by i = (1, 0) and in the direction of y-coordinate is denoted by
j = (0, 1).

Figure 1.4 Represents unit vectors in xy-plane


1.7 Representation of a Vector in terms of Unit Vectors 5

Figure 1.5 Represents unit vectors in the direction of x-axis, y-axis, and z-axis

Figure 1.5 represents a three-dimensional cartesian coordinate system


(i.e., xyz-axes), in which (1, 0, 0), (0, 1, 0), and (0, 0, 1) are called unit
vectors in the direction of x, y, and z axes respectively. The unit vector in
the direction of x-coordinate is denoted by i = (1, 0, 0), in the direction of
y-coordinate is denoted by j = (1, 0, 0), and in the direction of z-coordinate
is denoted by k = (1, 0, 0). Note that the modulus of each unit vector in the
direction of each axis is unity.

1.7 Representation of a Vector in terms of Unit Vectors


Let P (x, y, z) be a vector and i, j, and k be the unit vectors in the direction
−−→ −−→ −→
of OX, OY and OZ respectively in the given Figure 1.6.
Consider a perpendicular P M to the plane XOY . From Figure 1.6, we
can observe that M L⊥OX and M N ⊥OY . Then OL = x, ON = y, and
P M = z.
−→ −−→ −−→
∴ OL = xi, ON = yj, and OM = zk
The vectors xi, yj, and zk are called the rectangular components of
−−→ −−→
−−→ −−→  we can write OP = xi + yj + zk.
the vector OP and
−−→
The magnitude of
2 2 2
OP = |OP | = x + y + z . Thus, P (x, y, z) and OP = xi + yj + zk
represent the same vector.
6 Basic Concept of Vectors and Scalars

 in terms of unit vectors


Figure 1.6 Represents a vector OP

1.8 Addition and Subtraction of Vectors


If −

x = (x1 , x2 , x3 ) and −

y = (y1 , y2 , y3 ), then the sum vector →

x +−

y is
given by


x +− →
y = (x1 + y1 , x2 + y2 , x3 + y3 ) .
And the subtraction of vector −

x −− →y is given by


x −−

y = (x1 − y1 , x2 − y2 , x3 − y3 ) .



Illustration 1.5: If →

a = (4, −3, 2) and b = (−2, 5, 3).then

→ →

a + b = (4 − 2, −3 + 5, 2 + 3) = (2, 2, 5) .

Illustration 1.6: If →

x = (4, 10, −2) and →

y = (0, 1, −3).then


x −−→y = (4 − 0, 10 − 1, −2 − 3)
= (4 − 0, 10 − 1, −2 + 3)
= (4, 9, 1) .

1.9 Product of a Vector with a Scalar


Definition: Let k be a scalar and − →a be a vector. Then k −→
a is defined as
a vector whose modulus is k times the modulus of the vector − →
a and whose

− →

direction is the same as that of a or opposite to that of a according to k is
positive or negative (See Figure 1.7).
1.10 Direction of a Vector 7

Figure 1.7 Represents the product of a vector with a scalar

→ → −
− → → →

Note: −→
a  b ⇔− a =k b or b =k − a , k∈R
Illustration 1.7: If −
→a = (5, −3, 2), then

3−

a = 3 (5, −3, 2) = (15, −9, 6) .

Here 3−

a is a vector whose modulus is three times the modulus of →

a and
whose direction is the same as that of −

a.

1.10 Direction of a Vector


In this section, we try to understand the concept of the direction of a vector.
The direction of a vector in the direction from its initial point to its
terminal point. Two vectors can be of the same direction, opposite direction,
or different directions.


Definition: If − →
a and b are two non-null vectors and

→ →

(1) if there exists −
→a k > 0 such that −→a = k b . then −→a and b are of the
same direction.

− →

(2) if there exists −→
a k < 0 such that − →
a = k b . then − →a and b are of
opposite direction,


(3) if there does not exist −→
a k ∈ R − {0} such that − →a = k b , then the


directions of →−
a and b are different.

Illustration 1.8: Compare the directions of the vectors −



a = (2, −5, 3) and


b = (4, −10, 6).
Solution: Here −→a = (2, −5, 3) = 12 (4, −10, 6)


= 12 b and 12 > 0.


∴−→a and b are of the same direction.
8 Basic Concept of Vectors and Scalars

Illustration 1.9: Compare the direction of the vectors →



p = (3, −2, 1) and

−q = (−9, 6, −3)
Solution: Here −→q = (−9, 6, −3) = −3 (3, −2, 1)


= −3 p and −3 < 0.
∴−→p and −
→q are of opposite direction.


Illustration 1.10: Compare the directions of →

g = (2, 5, 7) and h =
(3, 1, 6).
Solution: Here 23 = 51 = 76 i.e., there does not exist the same ratio between


the elements of − →g and h .

− →

. . . We cannot express − →
g and h in the form −→
g = k h . (k = 0)


. . . The directions of −

g and h are different.

1.11 Collinear and Coplanar Vectors


1.11.1 Collinear Vectors

− →
− →

If two vectors −

a and b are such that −

a = k b , or b = k −→
a , where k ∈ R−



− →

{0}, then a and b are called collinear vectors. Thus a , →
1−
a , 3−
2
→a , −5−
→a are
2
collinear vectors.
Collinear vectors can be represented by parallel lines or line segments of
the same line.

1.11.2 Coplanar Vectors


Any number of vectors, which are parallel to the same plane, are called
coplanar vectors.

− →

If two non-linear vectors −

a and b are coplanar, then vector R given by

− →
− →

R = x− →a + y b is coplanar with the vectors −

a and b for any x, y ∈ R.

1.12 Geometric Representation of a Vector Sum


1.12.1 Law of Parallelogram of Vectors
−−→ →
− −−→
If −

a is a vector represented by AB and b is a vector represented by AD


(the two vectors →−
a and b have a common initial point), then their sum

− →
− −→
a + b is represented in magnitude and direction by AC, where ABCD is
1.12 Geometric Representation of a Vector Sum 9

Figure 1.8 Represents the law of parallelogram of vectors

a parallelogram (See Figure 1.8). This method of addition is called the law of
the parallelogram of vectors.

1.12.2 Law of Triangle of Vectors


If two vectors −

u and −

v are represented by the sides P Q and QR of ΔP QR,
then the sum u + v is represented by P R. The direction of −

− →
− →
u +−→
v is from
P to R and its modulus is the length of the side P R of ΔP QR.
This method of addition is called the law of the triangle of vectors.
Here the terminal point of the vector −
→u should be the initial point of the
vector v . The resultant (sum) vector u + −

− →
− →v can be obtained by joining the
initial point of the vector u and the terminal point of the vector −

− →
v as shown
in Figure 1.9.

Figure 1.9 Represents the law of the triangle of vectors

1.12.3 Properties of Addition of Vectors




Let −

a , b , and −

c be vectors then
10 Basic Concept of Vectors and Scalars
→ −
− → →
(1) Commutative law: −→
a + b = b +−a
 →
−  −
→ →
(2) Associative law: −→
a + b +−→c =−
→a + b +−
c


(3) Identity vector for addition: θ = (0, 0, 0) is identity vector for addition.

− → → −
− → →
a + θ =− a = θ +− a

(4) Opposite vector or negative of a vector: For a vector, there exists a


→ →

vector (−−→a ) such that −

a + (−−
→a ) = θ . −− a is called the opposite
or negative of −
→a.

Note: If −

a = (x, y, z), then −−→a = (−x, −y, −z). And −− →a = −1 · −→a.

− →

The moduli of a and (− a ) are equal but their directions are opposite to

− →

 −
each other.→We can define the
→ →
− 
difference

− 
of vectors a and b as the sum of

−a and − b i.e. − →a − b =− a + −b .

1.12.4 Properties of Scalar Product


→ →

Let −

a , b ,−
c be vectors and m, n ∈ R be scalars.

(1) m−

a =−

am

(2) m (n−

a ) = n (m−

a ) = (mn−

a)

(3) Distributive law: (m + n) −



a = m−

a + n−

a
 →
−  →

(4) m − →a + b = m− →a +mb


(5) θ→

a =−

aθ= θ

1.12.5 Expression of Any Vector in Terms of the Vectors


Associated with its Initial Point and Terminal Point
−−→ ←→
Let BC be a vector. Take a point A which is not on BC join AB and AC.
−−→ −→ −−→
We now have three different vectors AB, AC, and BC (See Figure 1.10).
By the law of the triangle of vectors, we have
−−→ −−→ −→
AB+BC=AC
−−→ −→ −−→
∴ BC=AC−AB
1.12 Geometric Representation of a Vector Sum 11

Figure 1.10 Represents any vector in terms of various vectors associated with its endpoints.

−−→
In other words, BC= vector of point C− vector of point B.
In general, any vector=vector of its terminal point-vector of its initial
point.

1.12.6 Expression of Any Vector in Terms of Position Vectors


The method of expression of a vector discussed above is true for any point A.
We can take the origin O in place of the point A.
As shown in Figure 1.11,
−→ −−→ −−→
OA+AB=OB

Figure 1.11 Represents any vector in terms of the position vector


12 Basic Concept of Vectors and Scalars
−−→ −−→ −→
∴ AB=OB−OA
In other words, any vector=position vector of its terminal point –position
vector of its terminal point –position vector of its initial point.
Illustration 1.11: Let the position vectors of the points A and B are (2, 5, -3)
and (3, -2, 5) respectively. Then
AB= position vector of B− position vector of A

= (3, −2, 5) − (2, 5, −3) = (1, −7, 8)

1.13 Direction Cosines of a Vector


In two dimensions, a vector makes angles with only two axes, namely X-
axis and Y-axis. Hence it is easier to understand. But in three-dimensional
space, a vector makes angles with three axes, and to understand the position
of the vector, direction cosines of the vector are useful. In Figure 1.12, three
important angles are shown considering OP .

(i) Angle formed by OP with the X-axis is ∠P OX. It is denoted by α.

(ii) Angle formed by OP with the Y-axis is ∠P OY . It is denoted by β.

(iii) Angle formed by OP with the Z-axis is ∠P OZ. It is denoted by γ.


The cosines of these angles are called direction cosines. Thus, we get
three direction cosines.

Figure 1.12 Represents direction cosines of a vector


1.13 Direction Cosines of a Vector 13

x
(1) l = cos α = OP . Taking OP = r, we have cos a = xr .
Now, r2 = x2 + y 2 + z 2 .

∴ OP = r = x2 + y 2 + z 2

x
∴ l = cos α =  (1.1)
x2 + y 2 + z 2
(2) cos β is denoted by m. It is connected with the Y-axis. As explained
above.
y
m = cos β =  (1.2)
x2 + y 2 + z 2
(3) cos γ is denoted by m. It is connected with the Z-axis.
z
n = cos γ =  (1.3)
x + y2 + z2
2

Relation among l, m and n,


Squaring and adding (1.1), (1.2), and (1.3), we get
x2 + y 2 + z 2
l 2 + m2 + n 2 = = 1.
x2 + y 2 + z 2
Thus l2 + m2 + n2 = 1
i.e., cos2 α + cos2 β + cos2 γ = 1.
Note that the direction cosines of a vector are the components of its unit
vector.
Illustration 1.12: If the position vectors of the vertices A, B, and Cof the


parallelogram ABCD are − →a , b and −
→c respectively, find the position vector
of the vertex D.


Solution: Here OA = − →a , OB = b, and OC = − →c . ABCD is a
parallelogram (See Figure 1.13).


∴ AD = BC = OC − OB = −

c − b

Now,
∴ AD = OD − OA
 → →

∴ OD = AD + OA = − →
c − b +−
a
14 Basic Concept of Vectors and Scalars

Figure 1.13 Represents a parallelogram

→ →

=−

a − b +−
c

Illustration 1.13: If the position vectors of the points A, B, C, D


→ →
− → →
− →

are respectively −

a , b , 3−
a − b , −− a + 3 b , then express the vectors


AB, AC, BC, BD, and CD in terms of − →a and b .

− →
− →

Solution: Here OA = − →
a , OB = b , OC = 3− →
a − b, and OD = −− →a +3 b .
→ →

Now, AB = OB − OA = b − − a
→ →
− →

AC = OC − OA = 3−

a − b −−a = 2−

a − b
→ →
− →

AD = OD − OA = −−→
a +3b −− a = 3 b − 2−

a
→ −
− → →

BC = OC − OB = 3−→a − b − b = 3− →
a −2b
→ −
− → → →

BD = OD − OB = −− →a +3b − b =2b −− a

−  →
− 
CD = OD − OC = −− →a + 3 b − 3− →
a − b

− →
−  → →

= −−
→a + 3 b − 3−

a + b =4 b −− a

Illustration 1.14: If ABCDEF is a regular hexagon, prove that


AB + AC + AD + AE + AF = 3AD.


Solution: Suppose AB = −

a , BC = b , and CD = −

c.
As ABCDEF is a regular hexagon,
CD = AF = − →
c,


AB = ED = −

a and BC = F E = b .
1.13 Direction Cosines of a Vector 15

Figure 1.14 Represents a regular hexagon



Now AB = →

a , AC = AB + BC = →

a + b , AD = AC + CD =

− →

a + b +−

c

AE = AD + DC = AD − ED
→ → −
− → →

=−→
a + b +−c −→a = b +−c


AF = CD = c
∴ LHS = AB + AC + AD + AE + AF
→ → −
− → → − → → −
=−→a +−→a + b +−a + b +−
c + b +−
c +→
c
 →
− 
=3 − →
a + b +−→
c = 3AD = RHS

Another method: From Figure 1.14, we have

LHS = AB + AC + AD + AE + AF
= ED + AC + AD + AE + CD
(∵ AB = ED and CD = AF )
= (AC + CD) + (AE + ED) + AD
= AD + AD + AD = 3AD = RHS

Illustration 1.15: If the sides AB and AC of ΔABC (See Figure 1.15)


represent two vectors and M is the mid-point of the side BC, then prove
that
AB + AC = 2AM
16 Basic Concept of Vectors and Scalars

Figure 1.15 Represents a triangle ΔABC

Solution:
Here
AB + BM = AM (1.4)
and
AC + CM = AM (1.5)
Adding (1.4) and (1.5), we have

AB + BM + AC + CM = 2AM (1.6)

But BM and CM are opposite vectors.

∴ BM + CM = 0 (1.7)

Using (1.7) in (1.6), we get

AB + AC = 2AM

Illustration 1.16: Find position vectors, moduli, unit vectors, and direction
cosines for vectors represented by the following points:
(i) P (3, −4) (ii) Q (6, 2) (iii) R (−4, −6)
1.13 Direction Cosines of a Vector 17

Solution:

(i) OP = −
→r = (3, −4) = 3î − 4ĵ

Modulus = |OP | = |−

r | = (3)2 + (−4)2
√ √
= 9 + 16 = 25 = 5


r 3î − 4ĵ 3 4
Unit vector r̂ = →− = = î − ĵ
|r| 5 5 5
Direction cosines: l = 35 , m = − 45

(ii) OQ = −
→r = (6, 2) = 6î + 2ĵ
√ √ √ √
Modulus = |−

r | = 62 + 22 = 36 + 4 = 40 = 2 10


r 6î − 2ĵ 3 1
Unit vector r̂ = →− = √ = √ î + √ ĵ
|r| 2 10 10 10

Direction cosines: l = √3 , m = √1 .
10 10

(iii) OR = −
→r = (−4, −6) = 4î − 6ĵ
 √ √ √


Modulus = | r | = (−4)2 + (6)2 = 16 + 36 = 52 = 2 13



r −4î − 6ĵ 2 3
Unit vector r̂ = →− = √ = √ î − √ ĵ
|r| 2 13 13 13

Direction cosines: l = − √213 , m = − √313

  →  1 1   
Illustration 1.17: If −
→ y = √2 , √2 and −
x = 1, 12 , − →
z = −2, − 32 then

(i) Find a unit vector in the direction of −



x +−→
z,
√ →
x − 2−
(ii) Find a unit vector in the direction of 2−
→ y + 2−

z.

Solution:
   
(i) Here −

x +−

z = 1, 12 + −2, − 32
18 Basic Concept of Vectors and Scalars

1 3
= 1 − 2, −
2 2

= (−1, −1) = −î − ĵ


 √ √
∴ |−
→x +− →z | = (−1)2 + (−1)2 = 1 + 1 = 2
. . . The unit vector in the direction of −

x +−

z


x +−
→z 1   1 1
= →− →
− = √ −î − ĵ = − √ î − √ ĵ
|x + z | 2 2 2
1 1
= −√ , −√
2 2
√ →
x − 2−
(ii) 2→
− y + 2−

z
1 √ 1 1 3
= 2 1, − 2 √ ,√ + 2 −2, −
2 2 2 2

= (2, 1) − (1, 1) + (−4, −3)


= (2 − 1 − 4, 1 − 1 − 3) = (−3, −3)
 √ −  
 −→ → →
−  2 2
 2 x − 2 y + 2 z  = (−3) + (−3)
√ √ √
= 9 + 9 = 18 = 3 2
√ →
. x − 2−
2−
→ y + 2−→
z
. . Required unit vector =  − √ 
2→ →

x − 2y +2z →

1 1 1
= √ (−3, −3) = −√ , −√
3 2 2 2

Illustration 1.18: Let −



a = (2, −1, 2) be a given vector.

(i) Find the unit vector in the direction of −



a.

(ii) Find a direction cosine of −



a.

(iii) Find a vector of magnitude 6 in the direction of −



a.

(iv) Find a vector of magnitude 4 in the opposite direction of −



a.
1.13 Direction Cosines of a Vector 19

Solution: Here −

a = (2, −1, 2) = 2î − ĵ + 2k̂.

(i) Unit vector in the direction of a




a 2î − ĵ + 2k̂ 1 
â = →− = √ = 2î − ĵ + 2k̂
|a| 4+1+4 3
2 1 2
= î − ĵ + k̂
3 3 3
(ii) Direction cosines: l = 23 , m = − 13 , n = 2
3

(iii) Vector of magnitude 6 in the direction of −→


a
6 
= 6â = 2î − ĵ + 2k̂ = 4î − 2ĵ + 4k̂
3

(iv) Vector of magnitude 4 in the opposite direction of −



a
4  8 4 8
= −4â = − 2î − ĵ + 2k̂ = − î + ĵ − k̂
3 3 3 3

Illustration 1.19: Answer the followings:




(i) If −→
a = 3î − ĵ − 4k̂, b = −2î + 4ĵ − 3k̂ and −

c = î + 2ĵ − k̂, then find


the direction cosines of the vector 3 a − 2 b + 4−

− →c.


(ii) If −→
a = 2î + ĵ − k̂, b = î − ĵ + 2k̂ and −

c = î − 2ĵ + k̂, then find the

− →
− →

direction cosines of a + b − 2 c .


(iii) If −

a = (3, −1, −4)  , b = (−2, 4, −3) and − →c = (−1, 2, −1) then find
 − → →
− →
− 
3 a − 2 b + 4 c .

−  →
− 
 → 
(iv) If −

a = î + ĵ, b = ĵ + k̂, and −

c = k̂ + î, then find 2−
a − 3 b − 5−

c .


(v) If −

a = 5î −3ĵ + 2k̂, b = 2î + →

 3ĵ − k̂ and c = î + 2ĵ + 3k̂, then find
 → →
− 
the value of 2−
a − 3 b + 4− →c .


− →
−  and −

 a = 3−
(vi) If î − 2ĵ +k̂, b = 2î − 4ĵ − 3k, c = −î + 2ĵ + 2k̂, find
 −→ → →
− 
2 a − 3 b − 5 c .
20 Basic Concept of Vectors and Scalars


− →

 a =
(vii) If 2î + ĵ − k̂, b = î − ĵ + 2k̂ and →
−c = î − 2ĵ + k̂ then find
− →
− 
→a + b − 2− →
c .


(viii) If −

a = (1, 2, 1), b = (2, 1, 1) and −

c = (3, 4, 1) then find
−→ → −
− → 
 a + 2 b + c .

−  →

 →
a = ĵ + k̂ − iand b = 2î + ĵ − 3k̂ then find 2−
(ix) If −
→ a + 3 b .


(x) If −
→a = (1, 2, 1),
 b = (1, −1, 2) and →
−c = (3, 2, −1) then find
 − →
− 
3→ a + b − 2−
→c .

Solution:


(i) Here −
→a = (3, −1, −4), b = (−2, 4, −3) and −

c = (1, 2, −1).


Let −

x = 3−→
a − 2 b + 4− →
c.

∴−

x = 3 (3, −1, −4) − 2 (−2, 4, −3) + (1, 2, −1)

= (9, −3, −12) − (−2, 4, −3) + (4, 8, −4)


= (9 + 4 + 4, −3 − 8 + 8, −12 + 6 − 4)
= (17, −3, −10)



∴ | x | = 172 + (−3)2 + (−10)2
√ √
= 289 + 9 + 100 = 398
If l, m, n are the direction cosines of −

x , then
x1 17 x2 −3 x3 −10
l= =√ ,m = =√ ,n = =√
|x| 398 |x| 398 |x| 398
17 −8 −10
∴ cosα = √ , cosβ = √ , cosγ = √
398 398 398

−  
(ii) −

a + b − 2→
−c = 2î + ĵ − k̂ + î − ĵ + 2k̂ − 2 î − 2ĵ + k̂

= 3î + k̂ − 2î + 4ĵ − 2k̂


1.13 Direction Cosines of a Vector 21

= î + 4ĵ − k̂ = (1, 4, −1)


  
→ − → 
∴ |−
→r | = −a + b − 2− →c  = 12 + 42 + (−1)2
√ √ √
= 1 + 16 + 1 = 18 = 3 2


If l, m, n are the direction cosines of − →a + b − 2− →
c , then
x1 1 x2 4
l = cosα = →− = √ , m = cosβ = →− = √ ,
|r| 3 2 |r| 3 2
x3 −1
n = cosγ = →− = √
|r| 3 2


(iii) 3→

a − 2 b + 4−

c

= 3 (3, −1, −4) − 2 (−2, 4, −3) + (−1, 2, −1)


= (9, −3, −12) + (4, −8, 6) + (−4, 8, −4)
= (9 + 4 − 4, −3 − 8 + 8, −12 + 6 − 4)
= (9, −3, −10)
 →
−  
 −
→ →
− 
∴ 3 a − 2 b + 4 c  = (9)2 + (−3)2 + (−10)2
√ √
= 81 + 9 + 100 = 190



(iv) Here −

a = î + ĵ, b = ĵ + k̂, −

c = k̂ + î = î + k̂


∴−→
a = (1, 1, 0) , b = (0, 1, 1) , −

c = (1, 0, 1)


∴ 2→

a = (2, 2, 0) , 3 b = (0, 3, 3) , 5−

c = (5, 0, 5)


∴ 2−→
a − 3 b − 5−→c = (2, 2, 0) − (0, 3, 3) − (5, 0, 5)
= (2 − 0 − 5, 2 − 3 − 0, 0 − 3 − 5)


∴ 2→−
a − 3 b − 5−→c = (−3, −1, −8)
 →
−  
 → 
∴ 2−a − 3 b − 5−→c  = (−3)2 + (−1)2 + (−8)2
(∵ Definition of magnitude)
√ √
= 9 + 1 + 64 = 74
22 Basic Concept of Vectors and Scalars


(v) Here −

a = (5, −3, 2) , b = (2, 3, −1) , −

c = (1, 2, 3).


Let−→
x = 2− →a − 3 b + 4−→c


∴ x = 2 (5, −3, 2) − 3 (2, 3, −1) + 4 (1, 2, 3)
= (10, −6, 4) − (6, 9, −3) + (4, 8, 12)
= (10 − 6 + 4, −6 − 9 + 8, 4 + 3 + 12)
∴− →
x = (8, −7, 19)

∴ |−→
x | = 82 + (−72 ) + 192
(∵ Definition of magnitude)
√ √
= 64 + 49 + 361 = 474


(vi) Here −

a = (3, −2, 1) , b = (2, −4, −3) , −→
c = (−1, 2, 2)


∴ 2−→
a − 3 b − 5−→c = 2 (3, −2, 1) − 3 (2, −4, −3) − 5 (−1, 2, 2)
= (6, −4, 2) + (−6, 12, 9) + (5, −10, −10)
= (6 − 6 + 5, −4, 12, 10, 2 + 9 − 10)


∴ 2−→
a − 3 b − 5−→c = (5, −2, 1)
 →
−  
 → 
∴ 2−a − 3 b − 5−→c  = (5)2 + (−2)2 + (1)2
(∵ Definition of magnitude)
√ √
= 25 + 4 + 1 = 30


(vii) Given that −

a = (3, −1, −4) , b = (−2, 4, −3) , −
→c = (−1, 2, −5)

− →
∴−→a +2b −− c = 2 (3, −1, 4) + 2 (−2, 4, −3) − (−1, 2, −5)
= (3, −1, −4) + (−4, 8, −6) + (1, −2, 5)
= (3 − 4 + 1, −1 + 8 − 2, −4 − 6 + 5)

− →
∴−→a +2b −− c = (0, 5, −5)
 →
−  
→ 
∴ −a +2b −− →
c  = 0 + 52 + (−5)2
(∵ Definition of magnitude)
√ √
= 25 + 25 = 50
 → →
− √
→
∴ −a +2b −− c=5 2
1.13 Direction Cosines of a Vector 23

(viii) Here −

a = 2î + ĵ − k̂,
∴− →
a = (2, 1, −1) ,


b = î − ĵ + 2k̂


∴ b = (1, −1, 2) and−

c = î − 2ĵ + k̂


∴ c = (1, −2, 1)


Now, −

a + b − 2−

c
= (2, 1, −1) + (1, −1, 2) − 2 (1, −2, 1)
= (2, 1, −1) + (1, −1, 2) + (−2, 4, −2)
= (2 + 1 − 2, 1 − 1 + 4, −1 + 2 − 1)


∴−→a + b − 2− →c = (1, 4, −1)
 
→ − → 
∴ −a + b − 2− →c  = |(1, 4, −1)|

= (1)2 + (4)2 + (−1)2
(∵ Definition of magnitude)

= 1 + 16 + 1

= 18

= 9 (2)

=3 2


(ix) Here −

a = (1, 2, 1) , b = (2, 1, 1) , −

c = (3, 4, 1) are given
→ →

∴− →a +2b +− c
= (1, 2, 1) + 2 (2, 1, 1) + (3, 4, 1)
= (1, 2, 1) + (4, 2, 2) + (3, 4, 1)
= (1 + 4 + 3, 2 + 2 + 4, 1 + 2 + 1)
→ →

∴− →a +2b −− c = (8, 8, 4)
 → −
−  
−→ → 
∴  a + 2 b + c  = (8)2 + (8)2 + (4)2
(∵ Definition of magnitude)
√ √ 
= 64 + 64 + 16 = 144 = (12)2
24 Basic Concept of Vectors and Scalars
 → →

→
∴ −a +2b +−
c  = 12

(x) Given that −



a = ĵ + k̂ − î = −î + ĵ + k̂


∴−→
a = −î + ĵ + k̂ = (−1, 1, 1) and b = 2î + ĵ − 3k̂ = (2, 1, −3)


∴ 2−

a +3b
= 2 (−1, 1, 1) + 3 (2, 1, −3)
= (−2 + 6, 2 + 3, 2 − 9)


∴ 2−→
a + 3 b = (4, 5, −7)
 →

 →
∴ 2−a + 3 b  = (4, 5, −7)

= (4)2 + (5)2 + (−7)2
(∵ Definition of magnitude)
√ √ √
= 16 + 25 + 49 = 90 = 9 × 10
 − 
→ √
 →
∴ 2−
a + 3 b  = 3 10



(xi) Given that −

a = (1, 2, 1) , b = (1, −1, 2) , −

c = (3, 2, −1)then find
 →
− 
 − 
3→a + b − 2− →
c


∴ 3−
→a + b − 2− →
c
= 3 (1, 2, 1) + (1, −1, 2) − 2 (3, 2, −1)
= (3, 6, 3) + (1, −1, 2) − (6, 4, −2)
= (3 + 1 − 6, 6 − 1 − 4, 3 + 2 + 2)
= (−2, 1, 7)
 →
−  √ √
 −
→ →
− 
Hence, 3 a + b − 2 c  = 4 + 1 + 49 = 54

Illustration 1.20: If a (1, 0, 0) + b (0, 1, 0) + c (2, −3, −7) = (0, 0, 0), where
a, b, c ∈ R, then find the values of a, b, and c.
Solution: Here

a (1, 0, 0) + b (0, 1, 0) + c (2, −3, −7) = (0, 0, 0)


1.13 Direction Cosines of a Vector 25

∴ a (a, 0, 0) + b (0, b, 0) + c (2c, −3c, −7c) = (0, 0, 0)


∴ (a + 0 + 2c, 0 + b − 3c, 0 + 0 − 7c) = (0, 0, 0)
∴ (a + 2c, b − 3c, −7c) = (0, 0, 0)
∴ a + 2c = 0, b − 3c = 0, −7c = 0
∴ c = 0, a = 0, b = 0 or a = 0, b = 0, c = 0.
Illustration 1.21: A space shuttle of 1000 tons hangs from two skyscrapers
using steel cables as shown in Figure 1.16. Find the forces or tensions in both
the cables attached with skyscrapers and also find their magnitude.

Figure 1.16 A space shuttle of 1000 tons weight hangs from two skyscrapers using steel
cables

Solution:

→ −

Let F1 and F2 be two forces or tensions on the steel cables respectively. First,

→ −

we represent F1 and F2 in terms of vertical and horizontal components.

→ −  − 
 →  →
F1 = − F1  cos50o î + F1  sin50o ĵ

→ −
− → −
 →

F2 = F2  cos32o î + F2  sin32o ĵ


The gravity force acting on the space shuttle is F = −mg ĵ =

− −

− (1000) (9.8) ĵ = −9800ĵ. Therefore, the counterbalance of F with F1


and F2 is given as
→ −
− → − → −
→ − → →

∴ F1 + F2 + F = 0 ⇒ F1 + F2 = − F = − (−9800) ĵ = 9800ĵ
26 Basic Concept of Vectors and Scalars

Thus,
 −  −  
 →  →
− F1  cos50o î + F1  sin50o ĵ +
−  −  
 →  →
F2  cos32o î + F2  sin32o ĵ = 9800ĵ
 −  −  
 →  →
∴ − F1  cos50o + F2  cos32o î+
−  −  
 →  →
F1  sin50o + F2  sin32o ĵ = 9800ĵ

Now, equation the components


−  −  −  − 
 →  →  →  →
− F1  cos50o + F2  cos32o = 0 ⇒ F1  cos50o = F2  cos32o
−  − 
 →  →
F1  sin50 + F2  sin32o = 9800
o



Solving for |F2 |, we get
− 
−   →
 →  1  cos50o
F
o
F1  sin50 + sin32o = 9800
cos32o
− 
 → 9800
∴ F1  = ≈ 8392 N
sin50o + tan32o cos50o
And − 
−   →
 → F1  cos50
o
 2
F = ≈ 6361 N
cos32o
Thus, the force vectors are

→ −

F1 ≈ −5394 î + 6429 ĵ and F2 ≈ 5394î + 3371ĵ.

1.14 Exercise
1. If −

x = (2, 1) and − →
y = (1, 3), then (i) find a unit vector in the direction
of 3 x − 2 y , (ii) find direction cosines of 3−

− →
− →x − 2− →
y.
 
Answer : (i) 45 î − 35 ĵ, (ii) 45 , − 35

2. If x (3, 2) + y (2, 3) = (17, 13), find the real values of x and y.


(Answer : (x, y) = (5, 1))
1.14 Exercise 27

3. Find a, b ∈ R such that (i) (4, 7) + (a, b) = (17, 13) (ii) (a, −8) −
2 (3, b) = (−4, 6).
(Answer : (i) (a, b) = (13, 20) , (ii) (a, b) = (2, −7))

4. If x̂ = (4, 7, 2) and ŷ = (−1, 3, 4), find the vectors 2x̂ + 4ŷ and 3x̂ − ŷ.
(Answer : (i) 2−→x + 4− →
y = (4, 26, 20) , (ii) 3−

x −− →y = (13, 18, 21))

5. If for real values of x, y, z, x (1, 2, 0) + y (0, 3, 1) + z (−1, 0, 1) =


(2, 1, 0), then find x, y, z.
(Answer : (x, y, z) = (5, −3, 3))

6. Find the vectors x, y ∈ R2 such that |x| = |y| = 1and |x − y| = 2.


(Answer : (1, 0) and (−1, 0))

7. If P (2, 4, −5) , and Q (1, 2, 3) are points in R2 , find the direction


cosines of P Q.
 
Answer : − √169 , − √269 , √869

8. If â = (3, −1, −4) , b̂ = (−2, 4, −3) , ĉ = (−1, 2, −1), then find the
direction cosines of the vector 3â − 2b̂ + 4ĉ.
 
9 3
Answer : √190 , − √190 , √−10
190

9. If â = (1,2, 3) , b̂ = (2, −2, −5) , ĉ = (3, −2, −1), then find (i) â +
 
2b̂ − ĉ (ii) â + b̂ + ĉ.
 √ √ 
Answer : (i) 68 (ii) 41
 
10. Show that â = (2, −3, 2) , b̂ = 1, − 32 , 1 are parallel vectors
2
Scalar and Vector Products

2.1 Scalar Product, or Dot Product, or Inner Product


(1) Algebraic definition: If − →
x = x1 î + x2 ĵ + x3 k̂ = (x1 , x2 , x3 ) and


y = y1 î + y2 ĵ + y3 k̂ = (y1 , y2 , y3 ) are two vectors in R3 , then their
dot product →

x ·− →y (read −→
x dot − →
y ) is defined as


x ·→

y = x1 y 1 + x 2 y 2 + x 3 y 3 (2.1)

Note that the result of a dot product is a scalar i.e., it is a real number.


(2) Geometric definition: If the angle between the vectors − →a and b is θ
as shown in Figure 2.1 then


− − 

→ →
− →
a . b = | a |  b  cos θ = ab cos θ, (2.2)

Figure 2.1 Represents a scalar or dot product

29
30 Scalar and Vector Products
− 
→
where, |−

a | = a and  b  = b.
 →
− →

Note: The angle θ is denoted by →−
a ,̂ b (read −

a cab b ) also.

2.2 The Measure of Angle Between two Vectors and


Projections

−  →

Let OA = −

a , OB = b , and −→
a ,̂ b = θ as shown in Figure 2.1.
Now,

→ → →−
− → OM
a · b =− a b cos θ = − →a (OB cosθ ) = − →
a OB × =− →a · OM
OB

− →

∴−→
a · b = |− →
a | (Projection
  of b on the direction of −

a)

− →
− →

  →
− →

Similarly, a · b =  b  (Projection of a on the direction of b )
→ −
− →−→
Also, the projection −
→a on the direction of b = a . b and the projection
−
→
b

− →
− −

of b on the direction of −

a = a.b

− .
| | a

− →

Thus, geometrically, a · b denoted the product of the modulus of one
vector and the projection of the second vector in the direction of the first
vector. →



a·b
An important result: From (2.2), we get cos θ = −→ → .

| a || b |

2.2.1 Properties of a Dot Product


(1) The scalar product of two vectors is commutative.
→ −
− → →
Thus, −

a · b = b ·− a


We have −→
a · b = x 1 y 1 + x 2 y 2 + x 3 y3
= y1 x1 + y2 x2 + y3 x3
→ →

= b ·− a
(2) If two non-zero vectors are perpendicular to each other then their dot
product is zero.


If −

a ⊥ b , then θ = 90◦ and cos θ = 0.


∴− →
a · b = ab cos θ = 0
2.2 The Measure of Angle Between two Vectors and Projections 31



Also, if −

a · b = 0, then ab cos θ = 0.
i.e., cosθ = 0 ⇒ θ = π2 .
(3) Scalar product of two like vectors or two opposite vectors:


(i) If −

a and b are of the same direction, then θ = 0 and cos θ =
cos θ = 1.

− →

∴−→
a · b = ab cos θ = − →a b = Product of their moduli


(ii) If −

a and b have opposite directions, then θ = p and cos θ =
cos π = −1.

− →

∴− →
a · b = ab cos θ = − →a b (−1)


= −− →
a b = −(Product of their moduli)


a = b , then −
From (i), if −
→ →
a ·−

a =−
→a−
→a =− →a = |− →
2 2
a|
(4) From unit vectors i, j, k in the directions of the axes:

î · î = ĵ · ĵ = k̂ · k̂ = 1,

î · ĵ = 0, ĵ · k̂ = 0, k̂ · î = 0,
Because î, ĵ, k̂ are mutually perpendicular.
(5) For p, q ∈ R,

−  →
− → →  −
− →
p−

a · p b = pq −→
a · b = (pq −

a)· b =−
a · pq b

(6) The scalar product of two vectors is distributive with respect to the vector
addition. −
→ → − → → −


−a · b +− c =→ a · b +− a ·→c
−→ → − → → −

Also, −
→a · b −− c =→ a · b −− a ·→ c
−→ →  −→ →
Note: If −→
a = θ and − →
a · b −− c = 0, then − →
a⊥ b −− c or
→ −

b −→ c = θ.
(7) The scalar product of two vectors is given by the sum of the products of
their corresponding elements.
32 Scalar and Vector Products


If →

a = (a1, a2 , a3 ) and b = (b1, b2 , b3 ), then

− →

a · b = (a1, a2 , a3 ) · (b1 , b2 , b3 )
= a1 b1 + a2 b2 + a3 b3 = Σai bi
 
Also −

a = |−

a | = a21 + a22 + a23 = Σa2i


→ −   
→
b =  b  = b21 + b22 + b23 = Σb2i


Now, −

a · b = ab cos θ gives

→ →

a.b
cos θ =  →

− −
| a | b 
a 1 b1 + a 2 b2 + a 3 b3
= 2 
a1 + a22 + a23 b21 + b22 + b23
Σai bi
= 
Σa2i Σb2i
Σai bi
=
Σa2i × Σb2i
⎧ ⎫
⎨ Σai bi ⎬
∴ θ = cos−1  
⎩ Σa2 Σb2 ⎭
i i

Note:

(1) If (l1 , m1 , n1 ) and (l2 , m2 , n2 ) are the direction cosines of vectors −



a


and b respectively. Then
cos θ = l1 l2 + m1 m2 + n1 n2 = Σl1 l2

(2)
sin2 θ = 1 − cos2 θ
 2  2
a · b a2 b2 − a · b
=1− =
a 2 b2 a 2 b2
2.2 The Measure of Angle Between two Vectors and Projections 33

 →2
− →2  →2
2 −  −
a 2 b2 − → −
a · b |−

a| b − − →a · b
∴ sin θ = −  = − 
→ →
|−
→a | b  |−
→a | b 

Remember the following two important results:


(1) −

x ·−

x = |−→
x | (2)−

x ·−→
y =0⇔− →
x ⊥− →
2
y

Illustration 2.1: If −

x = (1, 2, 3) and −

y = (2, 3, 4),then find (i) −

x ·−

y and

− →

(ii) ( x ,̂ y ).
Solution:
(i) →

x ·−

y = (1, 2, 3) · (2, 3, 4)
= (1) (2) + (2) (3) + (3) (4)
= 2 + 6 + 12 = 20
(ii) |→
− 2
x | = 1 + 4 + 9 = 14

∴ |−

x | = 14

|−
→ 2
y | = 4 + 9 + 16 = 29

∴ |−

y | = 29
Now,

−x .−→y 20
cos θ = cos(−x ,∧ −
→ →
y)= − → −
→ =√ √
| x || y | 14 29
20
∴ θ = (−
→x ,∧ −

y ) = cos−1 √ √
14 29
Illustration 2.2: If −
→x = î + 3ĵ + 2k̂ and − →
y = 4î − 2ĵ + k̂, then find (i)

− →
− →

x · y (ii) the angle between x and y .→

Solution:
(i) x · y = (1, 3, 2) · (4, −2, 1)
= (1) (4) + (3) (−2) + (2) (1)
=4−6+2=0
Hence, x and y are perpendicular.
34 Scalar and Vector Products

(ii) As x · y = 0, x⊥y.
∴The angle between x and y is 90◦ or π2 .

Illustration 2.3: If x = (1, −2, 2) and y = (0, 0, −1), then verify that

(i) |x.y| ≤ |x||y|


(ii) |x + y| ≤ |x| + |y|
(iii) |x + y| ≥ |x| + |y|

√ x = (1, −2,
Solution: Here, √2) and y = (0, 0,√−1).
∴ |x| = 1 + 4 + 4 = 9 = 3 and |y| = 0 + 0 + 1 = 1

(i)

x · y = (1, −2, 2) · (0, 0, −1)


= (1) (0) + (−2) (0) + (2)(−1)
= −2
∴ |x · y| = |−2| = 2 (2.3)

Also, |x| = 3 and |y| = 1

∴ |x| · |y| = 3 (2.4)

Hence, from (2.3) and (2.4) we get 2 < 3 ⇒ |x · y| ≤ |x||y|.


(ii) x + y = (1, −2, 2) + (0, 0, −1)

= (1 + 0, −2 + 0, 2 + (−1))
= (1, −2, 1)
√ √
∴ |x + y| = 1 + 4 + 1 = 6 (2.5)

Also, |x| = 3 and |y| = 1

∴ |x| + |y| = 3 + 1 = 4 (2.6)



Hence, from (2.5) and (2.6) we get 6 < 4 (∵ 6 < 16)

∴ |x + y| ≤ |x| + |y|.
2.2 The Measure of Angle Between two Vectors and Projections 35

(iii) x − y = (1, −2, 2) − (0, 0, −1)

= (1 − 0, −2 − 0, 2 − (−1))
= (1, −2, 3)
√ √
∴ |x − y| = 1 + 4 + 9 = 14 (2.7)

Also, |x| = 3 and |y| = 1

∴ |x| − |y| = 3 − 1 = 2 (2.8)



Hence, from (2.7) and (2.8) we get 14 > 2 (∵ 14 > 4)
Therefore, |x − y| ≥ |x| − |y|.

− →
− →

a , b ∈ R 3 , |−
Illustration 2.4: If for −
→ →
a | = | b |, then show that (−

a + b)


and (−→
a − b ) are perpendicular to each other.

− →

Solution: To prove (− →
a + b ) and
 (−

a − b )are perpendicular to each other,

− →

we have to show that − →a + b · − →a − b = 0.
 →  → −
− → → − → −
− → → − → − →
∴ →
−a + b · −a − b =− a ·→ a −−→
a · b + b ·− a + b · b

→ −→ → − →2
= |−
→a| −− →a · b + b ·−
2
a +b
→ −
− → −→2
(∵ −

a ·−

a = |−
→ 2
a | and b · b =  b  )
− 
→2 →

= |−
→ (∵ |−

2
a| −b a | = | b |)
=0
 → → −
− →
∴ −→
a + b ⊥ −
a − b



Illustration 2.5: If for −

a = (2, 2, −1) and b = (6, −3, 2) , then find the


angle between −→a and b .


Solution: We have − →a = (2, 2, −1) and b = (6, −3, 2) , then

→ →

a . b = (2, 2, −1) · (6, −3, 2) = 12 − 6 − 2 = 4
√ √ −  √
→ √
Also, |−

a | = 4 + 4 + 1 = 9 = 3,  b  = 36 + 9 + 4 = 49 = 7,
36 Scalar and Vector Products


If θ is the angle between −

a and b , then

→ →

a · b 4 4
cos θ = → = 3 × 7 = 21 = 0.1905



| a || b |
 
∴ sin 900 − θ = 0.1905
∴ 900 − θ = 100 58 (∵ Using trigonometric table)
∴ θ = 900 −100 58 = 790 2


Illustration 2.6: If −

a = 3î − 2ĵ + k̂ and b = î − 2ĵ + k̂, then find the


projection of −

a on b .


Solution: The projection of −

a on b is given as

→ →

a · b

= − →
b

(3, −2, 1) · (1, −2, 1)


= √
1+4+1

3+4+1 8 4 6
= √ = √ =
6 6 3

Illustration 2.7: Show that (−1, 6, 6) , (−4, 9, 6) , and (0, 7, 10) are posi-
tion vectors of the vertices of a right-angled triangle.
Solution: Let A (−1, 6, 6) , B (−4, 9, 6) , and C(0, 7, 10) be the vertices of
 ABC.
Now, AB = OB − OA = (−4, 9, 6) − (−1, 6, 6) = (−3, 3, 0)

BC = OC − OB = (0, 7, 10) − (−4, 9, 6) = (4, −2, 4)

And CA = OA − OC = (−1, 6, 6) − (0, 7, 10) = (−1, −1, −4)

AB · CA = (−3, 3, 0) · (−1, −1, −4) = 3 − 3 + 0 = 0

∴ AB⊥CA
∴  ABC is a right-angled triangle.
Illustration 2.7: Find the angle between the vectors 2î + ĵ + 4k̂ and î + ĵ + k̂.
2.3 Vector Product or Cross Product or Outer Product of Two Vectors 37

Solution: Let −

x = 2î + ĵ + 4k̂ = (2, 1, 4) and −
→y = î + ĵ + k̂ = (1, 1, 1)
then we have


x ·−
→y = (2, 1, 4) · (1, 1, 1) = 2 + 1 + 4 = 7 and if (x ,∧ y) = θ, then

− √
x ·− →
y 7 7 7
∴cos θ = − → →
− =√ √ =√ =
|x|| y | 21 3 63 3

−1 7
∴ θ = cos
3

Illustration 2.8: For what value of x, the vectors 2î−3ĵ +5k̂ and xî−6ĵ −8k̂
are perpendicular to each other?


Solution: Let −→a = 2î − 3ĵ + 5k̂ = (2, −3, 5) and b = xî − 6ĵ − 8k̂ =
(x, −6, −8).

− →

As −
→a⊥b ⇒→

a · b =0

∴ (2, −3, 5) · (x, −6, −8) = 0

∴ 2x + 18 − 40 = 0
∴ 2x = 22 ⇒ x = 11.

2.3 Vector Product or Cross Product or Outer Product of


Two Vectors


The vector product of two vectors −

a and b (See Figure 2.2) is denoted by

− →

a × b and it is defined as follow:


(1) Algebraic definition: If −

a = (a1 , a2 , a3 ) and b = (b1 , b2 , b3 ) then
⎡ ⎤

− î ĵ k̂

−a × b = ⎣ a1 a2 a3 ⎦ (2.9)
b 1 b 2 b3


→ →

a × b = (a2 b3 − a3 b2 ) î − (a1 b3 − a3 b1 ) ĵ + (a1 b2 − a2 b1 ) k̂

= (a2 b3 − a3 b2 , a3 b1 − a1 b3 , a1 b2 − a2 b1 )


Note: −

a × b is also a vector.
38 Scalar and Vector Products

Figure 2.2 Represents a vector or cross product

(2) Geometric definition:



− →

If −

a and b are given vectors, then their cross product −

a × b is defined
as − 

→ →
− →
a × b = |−

a |  b  sin θ n̂ (2.10)


where θ is an angle between −

a and b , n̂ is a unit vector perpendicular


to both −

a and b .


∴−→a × b = (ab sin θ ) n̂ (2.11)

2.4 Geometric Interpretation of a Vector Product



−  →
→ −
The vector product of two vectors →−
a × b has modulus − a × b  which is


twice the area of the triangle whose two consecutive sides are − →
a and b or
equals the area of a parallelogram whose two consecutive sides are − →a and


b. →
− →

Note that −→
a × b is a vector but |−
→a × b | is a scalar that gives the area
of the parallelogram ABCD as shown in text is present for Figure 2.3.

 →
→ −
Thus, −a × b  = |ab sinθ n̂|
= ab sinθ (∵ |n̂| = 1)
2.4 Geometric Interpretation of a Vector Product 39

Figure 2.3 Represents the geometric interpretation of a vector or cross product

= Area of the parallelogram ABCD


= 2 × Area of  ABC

2.4.1 Properties of a Vector Product


1. Vector product or cross product is non-commutative.
→ −
− → →
i.e., −

a × b = b ×− a.


Here if θ is the angle between directions of −

a and b , then the angle


between the direction of b and −
→a is −θ.


∴ −→
a × b = ab sin θ . n̂
→ →

and b × −
a = ab sin (−θ) .n̂ = −ab sin θ . n̂
(∵ sin (−θ) = −sin θ )


= −(−→a × b)
2. The vector product of two parallel vectors is a null vector.


If −

a  b , then θ = 0.


∴−

a × b = ab sin θ · n̂ = ab sin0 · n̂ = θ (∵ sin 0 = 0)

3. For the unit vectors î, ĵ, k̂ :


î × î = ĵ × ĵ = k̂ × k̂ = θ

and î × ĵ = k̂, ĵ × k̂ = î, k̂ × î = ĵ, ĵ × î = −k̂, k̂ × ĵ = −î,


î × k̂ = −ĵ.
40 Scalar and Vector Products

4. For any p, q ∈ R,

−  −
→ → →
− →

p−

a × q b = pq −→
a × b = (pq −

a)× b =−
a × (pq b )



Taking −

a = 1 and b = −1, we get
 −→ →
− →



a × − b = (−− →a ) × b = −(−

a × b)

5. Relation between cross product and dot product:


 →2
− →2  →2

− 2 −  → −
a · b = |−
→a |  b  − −
a × b

Or →2


→ − 2 −
→ →2
− 2 − 
a · b + →
a × b  = |−

a| b

6. Distributive law for vector product:


→− → − → → −

(i) −

a × b +− c =→ a × b +− a ×→
c
 →
−  → →

(ii) − →
a + b ×− →c =− →
a ×− →c + b ×−
c

7. Expression of a cross product in terms of unit vectors î, ĵ, k̂:




If −

a = a1 î + a2 ĵ + a3 k̂ = (a1 , a2 , a3 ) and b = b1 î + b2 ĵ + b3 k̂,
then  
 î ĵ k̂ 

− → 
− 
a × b =  a1 a2 a3 
 b 1 b 2 b3 

= (a2 b3 − a3 b2 ) î − (a1 b3 − a3 b1 ) ĵ + (a1 b2 − a2 b1 ) k̂

Remarks:
 → →

→ − |−

1. As −a × b  = absinθ, sinθ = a×b|


|−

a || b |



2. If the direction cosines of −

a and b are (l1 , m1 , n1 ) and (l2 , m2 , n2 )
respectively, then

sin θ = (m1 n2 − m2 n1 )2 + (l1 n2 − l2 n1 )2 + (l1 m2 − l2 m1 )2
2.4 Geometric Interpretation of a Vector Product 41

3. By definition of the cross product,



− →
− → → −
− →
a ⊥(−→
a × b ) and b ⊥(− a × b)
 →
− → → −
− →
∴−→
a · − →
a × b = 0 and b · − a × b = 0.

   
Illustration 2.9: Find the modulus of 2î − 3ĵ + k̂ × î − ĵ + 2k̂ .
  → 
− 
Solution: Consider −→a = 2î − 3ĵ + k̂ and b = î − ĵ + 2k̂
 
 

− →  î ĵ k̂
− 

∴ a × b =  2 −3 1 
 1 −1 2 
= (((−3) × 2) − ((−1) × 1))î − ((2 × 2) − (1 × 1))ĵ
+ ((2 × (−1)) − (1 × (−3)))k̂
= (−6 + 1) î − 3ĵ + k̂ = −5î − 3ĵ + k̂
 →
−  √ √
− 
∴ →a × b  = 25 + 9 + 1 = 35



Illustration 2.10: If −

a = 2î − ĵ, b = î + 3ĵ − 2k̂ then compute
 → → −
− →
 −
 →a + b × − a − b .



Solution: Here, −

a = 2î − ĵ = (2, −1, 0) and b = î + 3ĵ − 2k̂ = (1, 3, −2) .


∴ →−a + b = (2, −1, 0) + (1, 3, −2) = (3, 2, −2) and

− →

a − b = (2, −1, 0) − (1, 3, −2) = (1, −4, 2)
 
     î ĵ k̂ 

− →
− →
− →
−  
∴ a + b × a − b =  3 2 −2 

 1 −4 2 
= î (4 − 8) − ĵ (6 + 2) + k̂(−12 − 2)
= −4î − 8ĵ − 14k̂ = (−4, −8, −14)
   
→  √ √ √
 → − → −
∴ −a + b × −→
a − b  = 16 + 64 + 196 = 276 = 2 69
42 Scalar and Vector Products

Illustration 2.11: Simplify:


    
10î + 2ĵ + 3k̂ · î − 2ĵ + 2k̂ × (3î − 2ĵ − 2k̂) .



Solution: Consider −

a = 10î + 2ĵ + 3k̂ = (10, 2, 3) , b = î − 2ĵ + 2k̂ =
(1, −2, 2) and −

c = 3î − 2ĵ − 2k̂ = (3, −2, −2).
 
 î ĵ k̂ 
→ −
− →  
∴ b × c =  1 −2 2  = 8î + 8ĵ + 4k̂ = 4(2, 2, 1)

 3 −2 −2 

The given expression


−→ −

−a · b ×→ c = (10, 2, 3) · 4 (2, 2, 1) = 4 (20 + 4 + 3) = 108.

Illustration 2.12: Prove that


−→ → − →  →


−a × b +− c + b × (−

c +−

a)+−

c × −→
a + b = 0.

−
→ → − →  →

Solution: L.H.S. = − →
a × b +− c + b × (−→c +−→
a)+− →c × −→a + b
Using distributive law
→ → −
− → → −
− → → − →

=−→a × b +− a ×→ c + b ×−c + b ×− a +→ c ×−→
a +−→c × b
→ → →
− → → −
− → → − →

=−→a × b −− c ×− a −− →c × b −−
a × b +−c ×→ a +−
→c × b = 0 = R.H.S.

Illustration 2.13: Show that: (−



x −−→
y ) × (−

x +−

y ) = 2(−

x ×−

y)

− →
− →
− →

Solution: L.H.S. = ( x − y ) × ( x + y )
=−

x × (−

x +−

y)−−

y × (−

x +−

y)
=−
→x ×−

x +− →
x ×−
→y −−
→y ×−→
x −−
→y ×−

y
Now, we use −→x ×−→
x = θ, −

y ×−→
y = θ and −−

y ×−→
x =−→
x ×−

y in the
above expression
=θ+−

x ×−

y +−

x ×−

y − θ = 2 (−

x ×−

y ) = R.H.S

Illustration 2.14: If −

x = (3, −1, 2) and −→
y = (2, 1, −1) are given vectors.
Find unit perpendicular vector to the x and −

− →
y both.
2.4 Geometric Interpretation of a Vector Product 43

Solution: Given that −



x = (3, −1, 2) and −

y = (2, 1, −1)
 
 î ĵ k̂ 

− →
− 
∴ x × y =  3 −1 2  = (1 − 2) î − (−3 − 4) ĵ + (3 + 2) k̂

 2 1 −1 
= −î + 7ĵ + 5k̂ = (−1, 7, 5)

→ →
− √ √ √
| x × y | = 1 + 49 + 25 = 75 = 5 3
∴ Unit perpendicular vector to given vectors is −

x and −

y

−x × y→
− 1
= − = √ (−1, 7, 5) .
|→x ×− →
y| 5 3

Illustration 2.15: Find a unit vector of magnitude 10 which is perpendicular


to vectors 2î − ĵ − 2k̂ and 4î − 3ĵ − 5k̂.


Solution: Consider the vectors − →a = 2î − ĵ − 2k̂ = (2, −1, −2) and b =
4î − 3ĵ − 5k̂ = (4, −3,
 −5) 
 î ĵ k̂ 

− → 
− 
Now, a × b =  2 −1 −2 
 4 −3 −5 

= (5 − 6) î − (−10 + 8) ĵ + (−6 + 4) k̂

= −î + 2ĵ − 2k̂ = (−1, 2, −2)


 → √ √
→ −
∴ −a × b= 1+4+4= 9=3


Now, the unit perpendicular vector to −

a and b

− →

a × b 1
=

− → = 3 (−1, 2, −2)

|a × b|


A perpendicular vector with magnitude 10 to both −

a and b
10
= (−1, 2, −2) .
3

Illustration 2.16: If −

x = î + ĵ + k̂ and −

y = 2î − ĵ − k̂, then prove that −

x


and y are perpendicular to each other. Also, find a unit vector perpendicular
to the vectors −

x and −→y.
44 Scalar and Vector Products

Solution: Given that



−x = î + ĵ + k̂ = (1, 1, 1) and −
→y = 2î − ĵ − k̂ = (2, −1, −1)

− →

Now, x . y = (1,1, 1) . (2, −1, −1) → −
− →
 =2−1−1=0⇒ x⊥y.
 î ĵ k̂ 

− →
− 
Also, x × y =  1 1 1 
 2 −1 −1 

= (−1 + 1) î − (−1 − 2) ĵ + (−1 − 2) k̂

= 3ĵ − 3k̂ = (0, 3, −3)


√ √ √
|−

x ×−

y | = 0 + 9 + 9 = 18 = 3 2
Unit perpendicular vector to −

x and −

y


x ×−

y 1   1  
= → = √ 3ĵ − 3 k̂ = √ ĵ − k̂ .
|−
x ×−

y| 3 2 2

Illustration 2.17: Find a unit vector that makes an angle of 600 with the
vector î − k̂.
Solution: Let − →
x = î − k̂.

∴−

x = (1, 0, −1)

And suppose −

y = (y1 , y2 , y3 ) be the required vector.

∴ |−
→ 2
y | = y12 + y22 + y32 = 1 (2.12)

Now, (−

x ,̂−

y ) = 600

→x · −

y
cos 600 = cos (−

x ,̂ −

y)= −→ →

|x|| y |

(1, 0, −1). −→y


= √
2·1
1 y 1 − y3
∴ = √
2 2
1
∴ y1 − y3 = √
2
2.5 Application of Scalar and Vector Products 45

If we take y1 = 0, then y3 = − √12 . Then from (2.12), we get y22 = 1 − 1


2 =
1
2 ⇒ y = √1 . Thus, the required vector −
2 2
→y = (0, − √1 , √1 ).
2 2

 2.18: Show that the angle between the vectors î+2ĵ and î+ĵ+3k̂
Illustration
−1 46
is sin 55 .


Solution: Consider the vectors −

a = î + 2ĵ = (1, 2, 0) and b = î + ĵ + 3k̂ =
(1, 1, 3).
√ √ − → √ √
Also, |−

a | = 1 + 4 + 0 = 5,  b  = 1 + 1 + 9 = 11
 
 î ĵ k̂ 

− −
→  
a × b =  1 2 0  = (6 − 0) î − (3 − 0) ĵ + (1 − 2) k̂

 1 1 3 

= 6î − 3ĵ − k̂ = (6, −3, −1)


 → √ √
→ −
∴ −a × b  = 36 + 9 + 1 = 46
 →

Now, if →−
a 
, b = θ, then


− √
|−
→a × b| 46 46
sin θ = →
− =√ √ =
|−

a || b | 5 11 55

46
∴ θ = sin−1
55

2.5 Application of Scalar and Vector Products


The scalar and vector products are useful in many fields such as coordinate
geometry, solid geometry, trigonometry, mechanics, physics, computer sci-
ence, electric engineering, etc. In this section, we have discussed two basic
applications of scalar and vector products

(1) Work done by a force

(2) Moment of a force about a point.


46 Scalar and Vector Products

 on a particle
Figure 2.4 Represents work done by a force F

2.5.1 Work Done by a Force


The scalar product of a force applied to a particle and the displacement of the
particle in the direction of the force is known as work. Thus, work is a scalar
quantity.

− →

If F denotes the force applied to a particle and d denotes the displace-


ment of the particle in the direction of F , then work done (See Figure 2.4)


by the force F is given by
→ −
− →
W =F · d


Suppose a force F = (F1 , F2 , F3 ) acts on a particle at the point A(a1 , a2 , a3 )
and as a result, the particle moves to the point B (b1 , b2 , b3 ).


∴ The displacement d = AB = OB − OA = (b1 − a1 , b2 − a2 , b3 − a3 )


Now, work done by the force F is
 →
− 
W = Projection of F on AB × AB
→ −−→

F · AB
= AM × AB = × AB
AB
→ −−→

= F · AB

− −−→
= Dot product of F and AB
= (F1 , F2 , F3 ) · (b1 − a1 , b2 − a2 , b3 − a3 )
= F1 (b1 − a1 ) + F2 (b2 − a2 ) + F3 (b3 − a3 )
→ −
− → − → → −
− → − → − →
Note: If forces F1 , F2 , F3 , . . . act on the point A, then F = F1 +F2 +F3 +. . .

2.5.2 Moment of a Force About a Point




The moment of a force F about a point A is a vector and it is given by
−→ − → →

AP × F , where P is a point on the line of the force F .
2.5 Application of Scalar and Vector Products 47

Figure 2.5 Represents the moment of a force about a point



Thus, the moment of the force F about a point A (See Figure 2.5) is


a vector which is perpendicular to the plane containing F and A having a
modulus
−→ − → −  − 
 → →
= AP × F  = AP  F  sinθ = AN  F  (∵ AN = AP sinθ ) ,


where AN is the line perpendicular to the line of the force F .


Now let F = (F1 , F2 , F3 ) , A(a1 , a2 , a3 ) and P (p1 , p2 , p3 ). Then
−→ −−→ −→
AP = OP − OA = (p1 − a1 , p2 − a2 , p3 − a3 ).


Moment of F about the point A
−→ − →
= AP × F
 
 î ĵ k̂ 
 

=  p1 − a1 p2 − a2 p3 − a3 
 F1 F2 F3 
= {F3 (p2 − a2 ) − F2 (p3 − a3 )} î − {F3 (p1 − a1 ) − F1 (p3 − a3 )} ĵ
+ {F2 (p1 − a1 ) − F1 (p2 − a2 )} k̂

Note:


(1) The moment of the force F about a point on its line of action is always
−→
zero, because AP is a null vector.
→ −
− → − →
(2) If concurrent forces F1 , F2 , F3 , . . . act at the point P , then their moment
−→ − → → −
− → − → − →
about a point A = AP × F , where F = F1 + F2 + F3 + . . .
48 Scalar and Vector Products

Illustration 2.19: A particle moves from the point A (3, 2, −1) to the point


B (2, −1, 4) under the effect of force F = 4î − 3ĵ + 2k̂. Find the work done
by the force.


Solution: Here, the force vector is F = 4î − 3ĵ + 2k̂ = (4, −3, 2) and the
−−→
displacement is given by AB = (2, −1, 4) − (3, 2, −1) = (−1, −3, 5). So,
the work done is obtained as follow:
→ −
− → −−→
∴ Work done by the force F = F · AB = (4, −3, 2) · (−1, −3, 5)
= −4 + 9 + 10 = 15 units

Note: In this illustration, the measurement unit for the force and displacement
are not given so we have used a unit.
Illustration 2.20: The constant forces î + 2ĵ + 3k̂ and 3î + ĵ + k̂ act on a
particle. Under the action of these forces, a particle moves to 5î + ĵ + 2k̂ from
ĵ − 2k. Obtain total work done by these forces.

→ −

Solution: Here, F1 = î + 2ĵ + 3k̂ = (1, 2, 3) and F2 = 3î + ĵ + k̂ = (3, 1, 1)
are the forces acting on the particle. So, the total force or resultant force acting
on the particle is given by
→ −
− → − →
∴ Resultant force F = F1 + F2 = (1, 2, 3) + (3, 1, 1) = (4, 3, 4)
Let A represent the initial position of the particle and B represents the final
position of the particle after forces are applied. Therefore, A = ĵ − 2k̂ =
(0, 1, −2) and B = 5î + ĵ + 2k̂ = (5, 1, 2). Now, the displacement is given
by
−−→
∴ Displacement AB = (5, 1, 2) − (0, 1, −2) = (5, 0, 4)
As we have obtained the total forces acting on the particle and the displace-
ment of the particle, we calculate work done as follow:
→ −−→

∴ Work done = F · AB = (4, 3, 4) · (5, 0, 4)
=4×5+3×0+4×4
= 20 + 0 + 16 = 36
Thus, the total work done is by the forces is 36 units.
Illustration 2.21: Forces 3î − ĵ + 2k̂ and î + 3ĵ − k̂ act on a particle and the
particle moves from 2î + 3ĵ + k̂ to 5î + 2ĵ + k̂ under these forces. Find the
work done by these forces.
2.5 Application of Scalar and Vector Products 49



Solution: The given forces acting on a particle are F1 = 3î − ĵ + 2k̂ =


(3, −1, 2) and F2 = î + 3ĵ − k̂ = (1, 3, −1).
The total force acting on a particle is given by
→ −
− → − →
∴ Resultant force F = F1 + F2 = (3, −1, 2) + (1, 3, −1) = (4, 2, 1)

Let A represent the initial position of the particle then A = 2î + 3ĵ + k̂
= (2,3,1) and let B denote the final position of the particle after forces are
applied then B = 5î + 2ĵ + k̂ = (5, 2, 1). So, the displacement of the
particle from the point A to point B is given by
−−→
Displacement = AB = (5, 2, 1) − (2, 3, 1) = (3, −1, 0)
The work done by the forces is obtained as follow:
→ −−→

Work done = F · AB
= (4, 2, 1) · (3, −1, 0)
= 4 × 3 + 2 × (−1) + 1 × 0
= 12 − 2 + 0 = 10 units.

Illustration 2.22: A particle moves from point 3î−2ĵ + k̂ to point î+3ĵ −4k̂
under the effect of constant forces î − ĵ + k̂, î + ĵ − 3k̂, 4î + 5ĵ − 6k̂. Find
the work done by these forces on the particle.
Solution: Let A be the initial point of the position of the particle then A =
3î − 2ĵ + k̂ = (3, −2, 1) and let B be the final position of the particle then
B = î + 3ĵ − 4k̂ = (1, 3, −4). The displacement of the particle is obtained
as
−−→
Displacement = AB = (1, 3, −4) − (3, −2, 1) = (−2, 5, −5).


The constant forces acting on the particle are F1 = î− ĵ + k̂ = (1, −1, 1),

→ −

F2 = î + ĵ − 3k̂ = (1, 1, −3), and F3 = 4î + 5ĵ − 6k̂ = (4, 5, −6).
The resultant forces acting on the particle is given by
→ −
− → − → − →
∴ Resultant forces F = F1 + F2 + F3
= (1, −1, 1) + (1, 1, −3) + (4, 5, −6)
= (6, 5, −8)
→ −−→

∴ Work done = F · AB = (6, 5, −8) · (−2, 5, −5)
= −12 + 25 + 40 = 53 units
50 Scalar and Vector Products

Illustration 2.23: Three forces of magnitudes 2, 1, and 5 units in the direc-


tions (1, 2, 3), (−1, 2, 3), and (−1, 2, −3) are applied on a particle. If a
particle moves from the point (0, 1, −2) to the point (−1, 3, 2) under these
forces, find the work done.
Solution: Let − →
x = (1, 2, 3), −
→y = (−1, 2, 3), and −

z = (−1, 2, −3) are the
given directions, and to find the forces in these directions, we have to find the
unit vectors in these directions.
√ √ √ √
∴ |−

x | = 1 + 4 + 9 = 14, |− →
y | = 1 + 4 + 9 = 14,

and similarly |− →
z | = 14.


x 1
∴ The unit vector in the direction of →

x = →− = √ (1, 2, 3)
|x| 14


y 1
∴ The unit vector in the direction of −

y = −→ = √ (−1, 2, 3)
|y| 14


z 1
∴ The unit vector in the direction of −

z = − = √ (−1, 2, −3)
|→
z| 14


∴ F1 = Force of magnitude 2 in the direction of

− 2
x = √ (1, 2, 3)
14


∴ F2 = Force of magnitude 1 in the direction of

− 1
y = √ (−1, 2, 3)
14


∴ F3 = Force of magnitude 5 in the direction of

− 5
z = √ (−1, 2, −3)
14
→ −
− → − → − →
∴ The resultant force F = F1 + F2 + F3
1
= √ [2 (1, 2, 3) + (−1, 2, 3) + 5(−1, 2, −3)]
14
1
= √ [(2, 4, 6) + (−1, 2, 3) + (−5, 10, −15)]
14
1
= √ (−4, 16, −6)
14
2.5 Application of Scalar and Vector Products 51

Now, let A(0, 1, −2) be the initial position of the particle and B (−1, 3, 2) be
the final position of the particle then the displacement of the particle is given
by
−−→
Displacement = AB = (−1, 3, 2) − (0, 1, −2) = (−1, 2, 4).
Finally, the work done is given as
→ −−→
− 1
∴ Work done = F · AB = √ (−4, 16, −6) · (−1, 2, 4)
14
1 12
= √ (4 + 32 − 24) = √
14 14
Thus, the total work done is obtained as √12 units.
14
Illustration 2.24: Find the moment about the point (2, 3, −1) of the force
3î − k̂ acting through the point (1, −2, 1). Also, find the magnitude of the
moment.
Solution: Let A(2, 3, −1) and P (1, −2, 1) be the given points.


Let F = 3î − k̂ = (3, 0, −1) be the given force vector.
−→ −−→ −→
∴ AP = OP − OA = (1, −2, 1) − (2, 3, −1) = (−1, −5, 2)

− −→ − →
Moment of the force F about the point A = AP × F
 
 î ĵ k̂ 

=  −1 −5 2 
 3 0 −1 
= (5 − 0) î − (1 − 6) ĵ + (0 + 15) k̂
= 5î + 5ĵ + 15k̂ = (5, 5, 15)
√ √ √
∴ Magnitude of the moment = 25 + 25 + 225 = 275 = 5 11.

Illustration 2.25: Find the moment about the point (4, 0, 1) of the forces
2î + ĵ − 3k̂ and 2î − 2ĵ + k̂ acting through the point (−1, 3, −2).


Solution: The resultant force F is given by


F = 2î + ĵ − 3k̂ + 2î − 2ĵ + k̂ = (4, −1, −2).
Let A(4, 0, 1) and P (−1, 3, −2) be the given points.
−→
∴ AP = (−1, 3, −2) − (4, 0, 1) = (−5, 3, −3)
52 Scalar and Vector Products


The moment of the force F about the point A
 
 î ĵ k̂ 
−→ − →   
= AP × F =  −5 3 −3 
 4 −1 −2 
= (−6 − 3) î − (10 + 12) ĵ + (5 − 12) k̂
= −9î − 22ĵ − 7k̂

2.6 Exercise
   
1. Evaluate î + 2ĵ + k̂ · 3k̂ − 2ĵ + 4î .
(Answer : 3)


2. If −

a= î − ĵ + k̂, b = 2î − ĵ + k̂, and −

c = î + ĵ − 2k̂, then find

− → →

a · b +− c .
(Answer : 2)

3. For what value of p, the vectors 2î + 3ĵ − k̂ and pî − ĵ + 3k̂ are
perpendicular to each other?
(Answer :p= 3)


4. If the vectors −

a = mî − 2ĵ + k̂ and b = 2mî + mĵ − 4k̂ are
perpendicular to each other then find the value of m.
(Answer : m = −1, 2)

5. If vectors (m, 2m, 4) and (m, −3, 2) are mutually perpendicular then
find the value of m.
(Answer : m = 2, 4)

6. Find the angle between vectors (1, 2, 3) and (−2, 3, 1).


 
Answer : π3 or 60o

7. For which value of p vectors →



x = (1, −2, −3) and →

y = (2, p, 4) are
mutually perpendicular?
(Answer : p = −5)
2.6 Exercise 53



8. Find the value of q if vectors →

a = (q, 2, 1), and b = (2, q, −4) are
perpendicular to each other.
(Answer : q = 1)
 
9. Find the modulus of 2î − 3ĵ + k̂ × (î − ĵ + 2k̂).
 √ 
Answer : 35
10. Find a unit perpendicular vector to the given vector −

x = (1, 2, 3) and


y = (−2, 1, −2) .
 
Answer : 3√110 (−7, −4, 5)

11. Find a unit perpendicular vector to the given vector −



a = (1, −1, 1) and


b = (2, 3, −1) both.
 
Answer : √138 (−2, 3, 5)

12. Find a unit vector perpendicular to the plane containing the vectors −

a =


3î + ĵ + 2k̂ and b = 2î + ĵ − k̂.
 
Answer : √183 (−3, 7, 5)

13. Show  + k̂ is
that the angle between the vectors î + ĵ − k̂ and 2î − 2j
sin−1 26
27 .

14. Prove that the angle between two vectors 3î + ĵ + 2k̂ and 2î − 2ĵ + 4k̂
is sin−1 √27 .

15. Prove that |−


→ y | + |−
x +−
→ →
x −−

y | = 2(|−

x | + |−

2 2 2 2
y | ).
16. If −

x +−

y +→

z = θ, then prove that −

x ×−

y =−

y ×−

z =−

z ×−

x.
17. If the vectors î − 2ĵ − 3k̂ and î + 2k̂ represent two sides of the triangle
then find the area of the triangle.
 √ 
Answer : 12 45

18. Find the area of the parallelogram whose adjacent sides are 3î + ĵ − 2k̂
and î − 3ĵ + 4k̂.
 √ 
Answer : 10 3
54 Scalar and Vector Products

19. If |x − y| = 13 for the vectors →



x = (7, p + 1, 1) and →

y = (4, 1, 5),
then find the value of p.
(Answer : − 21)

20. Constant forces (3, −2, 1) and (−1, −1, 2) acts on the particle. Under
the effects of these forces, a particle is displaced from (2, 2, −3) to the
point (−1, 2, 4). Find the total work done by these forces.
(Answer : 15 unit)

21. Under the effect of two forces (4, 1, −3) and (3, 1, −1) particle is
displaced from origin to (1, 1, 1). Find the work done.
(Answer : 5 unit)

22. A particle is displaced from point (0, 1, −2) to the point (5, 1, 2) under
the effect of constant forces (1, 2, 3) and (3, 1, 1) then find the total work
done.
(Answer : 36 unit)

23. A particle moved from the point î − ĵ to point 3î + k̂ under the effect
of the two constant forces −2k̂ + î + ĵ and 2ĵ + 2î − 4k̂. Then find the
total work done.
(Answer : 3 unit)

24. Force ĵ + î + k̂ is action at −2î + 3ĵ + 4k̂ find the moment of inertia
and its magnitude above 2î + 3ĵ + 5k̂.
 √ 
Answer : (1, 3, −4) and 26

25. Force (1, 1, 1) is acting on B(1, 2, 3) find the magnitude of the moment
of inertia along A(−1, 2, 0).
 √ 
Answer : (−3, 1, 2) and 14
3
Vector Differential Calculus

3.1 Introduction
In this chapter, we study vector functions in three dimensions and the appli-
cations of differential calculus to them. Vectors simplify many calculations
considerably and help to visualize physical and geometrical quantities and
relations between them. Consequently, vector methods are used extensively
in applied mathematics and engineering. The impact of these methods on
the study of physical phenomena such as fluid flow, elasticity, heat flow,
electrostatics, electromagnetism, and waves in solids and fluids, which the
engineer must understand as the foundation for the design and construction
of systems such as aircraft, laser generators, robots, and thermo-dynamical
systems, is critical to the engineer.
Our goal is to acquaint readers with vector calculus, a branch of differen-
tial calculus that applies the basic notions of ordinary differential calculus to
vector functions. The gradient, divergence, and curl are three physically and
geometrically essential concepts connected to scalar and vector fields.

3.2 Vector and Scalar Functions and Fields


A variable quantity whose value at any point in a region of space depends
on the position of the point is called a point function. There are two types
of point functions. A point function whose values are vector is called vector
functions and is given as


v =−

v (P ) = (v1 (P ) , v2 (P ) , v3 (P ))

depending on the position of the points P in space. A point function whose


values are scalars at any point P is called scalar functions and is given as
f = f (P ) depending on the position of the point P in space.

55
56 Vector Differential Calculus

The domain of definition for such a function in applications is a region of


space, a surface in space, or a curve in space.

3.2.1 Scalar Function and Field


If to each point (x, y, z) of a region R in space corresponds a number or a
scalar f = f (x, y, z) then f is called a scalar function of position or scalar
(point) function and R is called a scalar field.
For example, scalar functions are defined as quantities that take dis-
tinct values at various sites, such as the temperature field of a body, the
pressure field of air in the earth’s atmosphere, density of a body, and poten-
tial. φ (x, y, z) = x2 − 2yz 3 defines a scalar field. A scalar field that is
independent of time is called a stationary or steady-state scalar field.

3.2.2 Vector Function and Field


If to each point (x, y, z) of a region R in space, there corresponds a vector

−v = − →v (x, y, z) = [v1 (x, y, z) , v2 (x, y, z) , v3 (x, y, z)] , then −→v is
called a vector function of position or vector (point) function and R is called
a vector field.
For example, vector functions include the velocity of a flowing fluid at any
given time, gravitational force, electric field intensity, magnetic field intensity,
and force. −→
v (x, y, z) = xy 2 î−2yz 2 ĵ +x2 k̂ defines a vector field. A vector
field that is independent of time is called a stationary or steady-state vector
field.

3.2.3 Level Surfaces


Let a scalar function f (x, y, z) be defined in a certain region R of space.
Consider those points of the field for which scalar function f has a fixed
value k. The totality of point satisfying the equation f (x, y, z) = k defines
in general a surface and is called a level surface of the function since at every
point of the surface f has a constant value k. For different values of k, we
have different level surfaces and there will be no intersection of two-level
surfaces. For example, if f (x, y, z) represents temperature in a medium,
then f (x, y, z) = k represents a surface on which the temperature is a
constant k. These surfaces are called isothermal surfaces.
For example, the level surfaces of the scalar fields in space defined by the
function f (x, y, z) = x2 + y 2 + z 2 is f (x, y, z) =√k i.e., x2 + y 2 + z 2 =
k. Therefore, the level surfaces are spheres of radius k.
3.3 Curve and Arc Length 57

3.3 Curve and Arc Length


First, we discuss the parametric representation of curves.

3.3.1 Parametric Representation of Curves


If x and y are given as continuous functions

x = f (t) , y = g (t)

over an interval I of t−values, then the set of points (x, y) = (f (t) , g (t))
defined by these equations is a curve in the coordinate plane. The equations
are parametric equations for the curve. The variable t is a parameter for the
curve and its domain I is the parameter interval. If I is a closed interval,
a ≤ t ≤ b, the point (f (a) , g (a)) is the initial point of the curve and
(f (b) , g (b)) is the terminal point of the curve. If the equations of a curve
are given in the plane in parametric equations and parametric interval then
it is called a parameterized curve. The equations and interval constitute a
parameterization of the curve.
A curve C in the two-dimensional xy-plane can be parametrized by

x = x (t) , y = y (t) , a ≤ t ≤ b.

Then, the position vector of a point P on the C can be written as




r (t) = x (t) î + y (t) ĵ (3.1)

Therefore, the position vector of a point on a curve defines a vector


function. Similarly, a three-dimensional curve or a space curve C can be
parametrized as


r (t) = x (t) î + y (t) ĵ + z (t) k̂, a = t = b (3.2)

Illustration 3.1: Find a parametrization for the line segment with endpoints
(−2, 1) and (3, 5) .
Solution:
Using (−2, 1), we crate the parametric equations x = −2 + at, y =
1 + bt.
These represent a line, as we see by solving each equation for t and
equating to obtain
x+2 y−1
=
a b
58 Vector Differential Calculus

This line goes through the point (−2, 1) when t = 0. We determine a and b
so that the line goes through (3, 5) when t = 1.

3 = −2 + a ⇒ a = 5x = 3 when t = 1
5 = 1 + b ⇒ b = 4y = 5 when t = 1

Therefore,

x = −2 + 5t, y = 1 + 4t, 0 = t = 1.

or


r (t) = (−2 + 5t) î + (1 + 4t) ĵ
is a parametrization of the line segment with initial point (−2, 1) and
terminal point (3, 5) .
Illustration 3.2: Find the parametric representation of the straight line
through the point P (1, 2, 3) and has the direction b = i + 2j + 2k.
Solution:
In this case, a = (1 − 0) i + (2 − 0) j + (3 − 0) k = i + 2j + 3k

∴ a1 = 1, a2 = 2, a3 = 3

from b = i + 2j + 2k, we have b1 = 1, b2 = 2, b3 = 2

∴−

r (t) = a + tb = (a1 + tb1 ) î + (a2 + tb2 ) ĵ + (a3 + tb3 ) k̂

gives −
→r (t) = (1 + t) î + (2 + 2t) ĵ + (3 + 2t) k̂
or −
→r (t) = (1 + t) î + 2 (1 + t) ĵ + (3 + 2t) k̂ are required for parametric
representation.

3.3.2 Curves with Tangent Vector


A curve is the locus of a point whose position vector − →r relative to a fixed
origin may be expressed as a function of a single variable parameter. Then its
Cartesian coordinates x, y, z are also functions of the same parameter.
Thus, the equation of the curve is usually expressed in parametric form

x = x (t) , y = y (t) , z = z (t) (3.3)

If P, Q are adjoining points (x, y, z) and (x + δx, y + δy, z + δz) , then


their position vectors are given by (Figure 3.1)
3.3 Curve and Arc Length 59

Figure 3.1 Represents a curve with a tangent vector



r = xî + y ĵ + z k̂ and −

r + δ−

r = (r δx) î + (y + δy) ĵ + (z + δz) k̂
∴ δ−

r = (δx) î + (δy) ĵ + (δz) k̂
∴ |d−
→ 2
r | = δr2 = δx2 + δy 2 + δz 2 (3.4)
If δs is the length of the are P Q and in the limit as δt → 0, chord
P Q (= δr) and are δs will be equal i.e., dr
ds = 1

d−
→ 2 2 2 2
r dx dy dz
∴ = + + =1 (3.5)
ds ds ds ds
and
d−
→r dx dy dz
= î + ĵ + k̂
ds ds ds ds

 −  
 d→r  dx 2
dy 2
dx 2
∴  =
 + + =1 (∵ from (3.4)) (3.6)
ds ds ds ds

3.3.2.1 Tangent Vector


Limiting position of chord P Q (i.e., d−
→r ) as δt → 0 (as Q → P ) is along
the tangent to the curve at P
 − 
d−
→r  d→
 r 
= T̂ = T̂ ( from (3.5))
ds ds 
where unit vector T̂ is along the tangent at P (t) .
dr dr ds ds
∴ = · = · T̂ (3.7)
dt ds dt dt
where, T̂ is a tangent vector.
60 Vector Differential Calculus

3.3.2.2 Important Concepts


In Figure 3.2, the limiting position of the P QR passing through neighboring
points P, Q, R on the curve and as Q, R approaches to P (i.e., the plane
containing two consecutive tangents and therefore three consecutive points
at P ) is called the osculating plane or the plane of curvature of the curve at
point P.
The binormal at the point P is denoted
 by B̂ and is a line normal to the
osculating plane at P i.e., B̂ ⊥ T̂ . And the principal normal at the point
P denoted by n̂ to the curve at point P is a line through the point P lying
in the osculating plane at P and it is perpendicular to the tangent line. N̂ ⊥T̂
and N̂ ⊥B̂.
The binormal is perpendicular to both T̂ and N̂ , it is parallel to T̂ × N̂
Hence trio unit vectors T̂ , N̂ , B̂ form a right-handed system of mutually
perpendicular unit vectors, and therefore connected by the relations

T̂ . T̂ = N̂ . B̂ = B̂ . T̂ = 0

and
T̂ × N̂ = B̂ , N̂ × B̂ = T̂ , B̂ × T̂ = N̂ ,
the cyclic order being preserved in the cross products.

(a) A plane through the point P , normal to B̂ is an osculating plane of the


curve at P .

(b) A plane through the point P normal to T̂ is known as the normal plane
of the curve at P .

(c) A plane through the point P normal to N̂ is known as rectifying plane


of the curve.

Figure 3.2 Represents the plane of curvature of the curve


3.3 Curve and Arc Length 61

3.3.3 Arc Length


The parameterization for a curve is a set of functions depending only on a
parameter t along with the bounds for the parameter. When we parameterize
a curve by taking values of t from some interval [a, b], the position vector

−r (t) of any point, t on the curve can be written as,


r (t) = x (t) î + y (t) ĵ + z (t) k̂
The tangent vector −

r  (t) is

− 
r (t) = x (t) î + y  (t) ĵ + z  (t) k̂
−   
∴ →r (t) = (x (t))2 + (y  (t))2 + (z  (t))2
The length of the curve is
 b 
l= −
→ 
r (t) dt
a
The arc length function or arc length of the curve is obtained by considering
a variable limit. i.e., a = t1 and b = t2 .
 t2
− 
l= → 
r (v) dv (3.8)
t1

3.3.3.1 Unit Tangent Vector


The unit tangent vector of a curve −

r (t) is

d−
→r d−
→r /dt v
T̂ = = = (3.9)
ds ds/dt |v|
Illustration 3.3: Find the length of the arc between points (a, 0, 0) and
(0, a, 12 πa tan α ) for the curve x = a cos θ , y = a sin θ , z = a θ tan α .
Solution:
Using result (3.8), we get
2 2 2 2
ds dx dy dz
= + +
dθ dθ dθ dθ
= a2 sin2 θ + a2 cos2 θ + a2 tan2 α
= a2 (1 + tan2 α) = a2 sec2 α
ds
∴ = a sec α

62 Vector Differential Calculus

For the given points the parameters are


θ = 0[a, 0, 0] and θ = π2 [0, a, 12 πa tan α]
π/2
∴ The required length = 0 a sec α · dθ = π
2 a sec α .
Illustration3.4: Prove that the length
 of the curve
−1

x = 2a sin t + t 1 − t , y = 2 at2 , z = 4 at
2

between the points, t = t1 and t = t2 are 4 2 (t2 − t1 ) a.
Solution:
Now,
!  "
dx 1 2
2t
= 2a √ + 1−t −1 − √
dt 1 − t2 2 1 − t2
! 2  " 
1−t
= 2a √ + 1 − t2 = 4a 1 − t2
1 − t2
dy dz
= 4 at, = 4a
dt dt
ds 2 dx 2 dy 2 dz 2
∴ = + +
dt dt dt dt
 
= 16a 1 − t + 16a t + 16a2
2 2 2 2
# 
= 162 1 − t2 + t2 + 1] = 32a2
ds √
∴ = 4 2a
dt
√  t2 √
∴ s = 4 2a dt = 4 2 a (t2 − t1 )
t1

Illustration 3.5: Find the length of the arc of the curve


y = log sec x from x = 0 to x = π/3
Solution:
Here,
dy 1
= · secx tanx = tanx
dx secx
!
 2 "
π/3 dy
Length of the arc S = 0 1 + dx dx
3.3 Curve and Arc Length 63

 π/3 
= 1 + tan2 x dx
0
 π/3
π/3
= sec x dx = [log(secx − tanx)]0
0
 π  π  
= log sec − tan − log (1 − 0)
√ 3 3
= log(2− 3) − 0

= log(2 − 3 )

Illustration 3.6: Find the arc length of the Helix traced by



−r (t) = a cost î + a sinĵ + ct k̂, a > 0, 0 ≤ t ≤ 2π .
Also, express the position vector −

r (t) in terms of the arc length s.
Solution:
we have x (t) = a cost , y (t) = a sint , z (t) = ct and
dx dy dz
= −a sint , = a cost , =c
dt dt dt
Therefore,
 2π
s = Arc length = [a2 sin2 t + a2 cos2 t + c2 ]1/2
0
 2π
 2 1/2  1/2
= a + c2 dt = 2π a2 + c2
0
Also, the arc length s is given as
 t
 2 1/2  1/2
s= a + c2 dt = t a2 + c2
0
or
s
t=
(a2 + c2 )1/2
Therefore,
 
s s
r (s) = a cos  i + a sin  j
(a2 + c2 ) (a2 + c2 )

s
+c  k.
(a2 + c2 )
64 Vector Differential Calculus
 
It can be verified that  dr 
ds = 1.
Illustration 3.7: Find the unit tangent vector to the unit circle


r (t) = (cos t) î + (sin t) ĵ

Solution:
Here, r (t) = (cos t) î + (sin t) ĵ
dr
∴−→
v = = (−sin t) î + (cos t) ĵ
√  dt
 − 
 →
v=1


v (−sin t ) î + (cos t) ĵ
∴ T̂ = →− = = (−sin t) î + (cos t) ĵ
|v| 1

3.4 Curvature and Torsion


In this section, we discuss curvature, torsion, and Serret (1851) – Frenet
(1852) Formulas. We may gather them together in the form.
(a) ddST̂ = k N̂ , (b) ddS

= −τ N̂ , (c) ddS

= τ B̂ − k N̂
These formulas give the derivatives of the direction cosines of the tangent,
the binomial, and the principal normal.

(I) Definition: Curvature k at point P on the curve is are the rate of rotation
of tangent T̂ at P

∴k= (i)
dS
where δΨ is the angle through which T̂ has turned when P moves to a
dT̂ dT̂ dΨ dT̂
point Q . dS = dΨ . ds = k dΨ (ii)
Now,
T̂ .T̂ = 1
dT̂ dT̂
∴ T̂ · = 0 ⇒ T̂ ⊥
dS dS
Since T̂ and T̂ + δ T̂ lie in the osculating plane, δ T̂ lies in the osculating
plane i.e., ddST̂ lies in an osculating plane and being perpendicular to, it is
3.4 Curvature and Torsion 65

Figure 3.3 Represents the arc rate of rotation of binormal

along N̂ at P .
  
 dT̂ 
dT̂  
∴ = (1) N̂ ∵  =1 (iii)
dψ  Dψ 

From (ii) and (iii) we get,

dT̂
= k N̂ (3.10)
dS

(II) Definition: Torsion (τ ) at point P on the curve is arc-rate of rotation of


binormal at P .
If δ∅ is the angle of rotation when P moves to point Q (in Figure 3.3).


∴τ = (i)
dS
dB̂ dB̂ dφ dB̂
= · = τ (ii)
dS dφ ds d∅
Now,B̂ · B̂ = 1, ∴ B̂ · dB̂
dφ =0

dB
∴ B̂⊥

Since B̂ and B̂ +δ B̂ (in limiting position as → 0 ) lie in a normal plane,


dB̂ dB̂
dφ lies in a normal plane and is perpendicular to B̂. Hence dφ is along
the normal N̂ at P .
66 Vector Differential Calculus

Figure 3.4  and N̂ orthogonal unit vectors


Represents B̂, T,

If point P moves in the increasing direction of s and the right-handed


screw is rotated in such a way as to advance in direction of s (i.e., T̂ ),
the vector B̂ is rotated in the clockwise direction and thus the direction
of ddsB̂ is opposite to that of N̂ .
  
 dB̂ 
dB̂  
∴ = −N̂ ∵  =1 (iii)
d∅  dφ 

From (ii) and (iii), we get

dB̂ dB̂
=τ = −τ N̂
ds dφ
Hence,
dB̂
= −τ N̂ (3.11)
ds
(III) Since B̂, T̂ , and N̂ are orthogonal unit vectors (in Figure 3.4),

N̂ = B̂ × T̂
dN̂ dT̂ dB̂
∴ = B̂ × + × T̂
ds ds ds
   
= B̂ × k N̂ + −τ N̂ × T̂
3.4 Curvature and Torsion 67

Using results (3.10) and (3.11), we get


   
= k B̂ × N̂ − τ N̂ × T̂
   
=k − − T̂ − τ −B̂
Hence,
dN̂
= τ B̂ − k T̂ (3.12)
ds

3.4.1 Formulas for Curvature and Torsion


The position vector −

r (t) of the point [x (t) , y (t) , z (t)] on the curve is
given by


r (t) = x (t) î + y (t) ĵ + z (t) k̂ (i)
where x = x (t) , y = y (t) , z = z (t) represents parametric equations of
the curve −

r =−→
r (t)

d−→r d−→r ds ds
∴−

r = = · = · T̂ (ii)
dt ds dt dt

→ d2 r d2 s ds dT̂
r¨ = 2 = 2 T̂ + dt . dt
dt dt
d2 s ds dT̂ ds
= 2 T̂ + .
dt dt ds dt
2
d2 s ds dT̂
= T̂ +
dt2 dt ds
d2 s ds 2
= T̂ + k N̂ (iii)
dt2 dt
! " $ 2 2
%

− →
−̈ ds d s ds
∴ r × r = T̂ × T̂ + k N̂
dt dt2 dt
ds 3    
= k T̂ × N̂ ∵ T̂ × T̂ = 0
dt
3
ds
= k B̂ (iv)
dt
68 Vector Differential Calculus
   3  
→  −̇ 
∴ −̇
r × −̈→r  = k dsdt and →r= ds
dt (From (ii))
Equating the magnitudes, we get
   3
−̇
→ →
−̈  −̇ 
 r × r  = k →r (v)

 
 
 −̇
→ →
−̈
r × r
∴ k =   3  (3.13)
 −̇  
 → r 
 
Formula (3.13) determines the curvature k or radius of curvature = k1 at
a point P on the curve.
From equation (iii), we get
$ %
...

− d3 −
→r d3 s d2 s dT̂ d ds 2 ds 2 dN̂
r = = T̂ + + k N̂ + k
dt3 dt3 dt2 dt dt dt dt dt
$ % $ %
d3 s d2 s dT̂ ds d ds 2 ds 2 dN̂ ds
= 3 T̂ + 2 · + k N̂ + k ·
dt dt ds dt dt dt dt ds dt

dT̂ dN̂
Substituting ds and ds from the results of the Serret-Frenet Formula,
we have
$ %


... d 3s d 2 s ds d ds 2
ds 3  
r = T̂ + k N̂ + k N̂ + k τ B̂ − k T̂
dt3 dt2 dt dt dt dt
$ % & $ %'
3 2
d2 s ds ds d 2s d ds
= − k2 T̂ + k + k N̂
dt3 dt dt dt2 dt dt
$ %
ds 3
+ kτ B̂
dt
$ %
...   ds 3 ...

− →
−̇
∴ r · r × r = →
−̈ k B̂ . − →
r (∵ From (iv))
dt
ds 6 2  
= k τ ∵ B̂ × T̂ = B̂ × N̂ = 0
dt
( )2
= −̇→r × −̈ →r τ (∵ From (v))
3.4 Curvature and Torsion 69

...  


r · −̇ →
r × −→r¨
∴ τ= ( )2 (3.14)

−̇
r ×−→

1
The formula (3.14) gives the torsion τ or radius of torsion σ = τ at a
point P on the curve.
2
Illustration 3.8: For the curve x = t, y = t2 , z = 3 , find (i) K and (ii) τ
at point  t .
Solution:

→ 2 3
r = t (i) î + t2 ĵ + t k̂
3
−̇
→r = î + 2tĵ + 2t2 k̂

−̈
r = 2ĵ + 4t k̂
−̈
→r = 4k̂
⎡ ⎤
î ĵ k̂  2

−̇
r × → −̈
r = ⎢ 2 ⎥
⎢ 1 2t 2t ⎥ = 4t î − (4t) ĵ + 2k̂
⎢ 0 2 4t ⎥
 2
Hence, −→ 2
r  = 1 + 4t2 + 4t4 = 1 + 2t2
( )2  2
and −̇

r × −̈ →r = 16t2 + 16t2 + 4 = 4 2t2 + 1
  , -
and r × r · −̈

−̇ →
−̈ →
r = 4t2 i − 4t j + 2kj . 4k = 8
( )    ... 
−̇
→r × −̈

r −̇
→r × −̈ →
r · − →r
Using formula k = ( )3 and τ = ( )2

−̇
r →
−̇ →
−̈
r ×r
we get
2(1+2t2 ) 2 8 2
k = (1+2t2 )3 = (1+2t2 )2
and τ = 4(1+2t2 )2
= (1+2t2 )2
Illustration 3.9: Show that for the curve x = a cos θ , y = a sin θ , z =
a θ cot β curvature and torsion are k = a1 sin2 β and τ = a1 sinβ cosβ .
Solution:
Here, −→
r = (a cos θ ) î + (a sin θ ) ĵ + (a θ cot β ) k̂
⎡ ⎤
î ĵ k̂
∴ −̇
→r × −̈
→r = ⎣ −a sin θ a cos θ a cot β ⎦
−a cos θ −a sin θ 0
70 Vector Differential Calculus

and
...


.

r = (a sin θ ) î + (−a cos θ ) ĵ + 0 k̂


( )2

−̇
r = a2 sin θ + a2 cos2 θ + a2 cot2 β
# .
= a2 1 + cot2 β = a2 cosec2 β
( )2  

−̇
r × −̈

r = a4 sin2 θ cot2 β + a2 cos2 θ cot2 β + a4 cot2 β + 1
= a4 cosec2 β

and
  ...     

−̇
r × −̈
→r · →

r = a2 sin θ cot β î + −a2 cos θ cot β ĵ + a2 k̂ ·
[(a sin θ ) î + (−a cos θ ) ĵ]
= a3 sin2 θ cot β + a3 cos2 θ cot β = a3 cot β
( )

−̇
r × →−̈
r a2 cosec β 1
∴k= ( )3 = 3 3
= sin2 β

−̇
r
a cosec β a

and
 

−̇
r × −̈

r · −̈
→r a3 cot β 1
τ= ( )2 = 4 2 β
= sin β cos β

−̇ →
−̈
r × r
a cosec a

3.5 Vector Differentiation


Next, we show the basic concepts of calculus, such as vector differentiation,
differentiability defined for scalar, and vector functions in a simple and
natural way.
The derivative of a vector function −→v (t) with respect to the scalar,
variable t is given as

d→
−v →

v (t + Δt) − −

v (t)
=−
→ 
v (t) = lim (3.15)
dt Δt→0 Δt
provided the limit exists.
The vector −→v  (t) is called the derivative of →

v  (t). (see Figure 3.5).
3.5 Vector Differentiation 71

Figure 3.5 Represents the derivative of the vector v (t)

In terms of components with respect to a given Cartesian coordinate


system, − →v (t) is differentiable at a point t if and only if its three components
v1 (t), v2 (t), and v3 (t) are differentiable at t, and then the derivative −

v  (t) is
obtained by differentiating each component separately,

−   
v (t) = v1 , v2 , v3 (3.16)

Most of the familiar rules of differentiation yield corresponding rules for


differentiating vector functions provided the order of factors in vector product
is maintained.

1. The derivative of a constant vector is zero.


2. (c−

v) =c−


v  where c is constant
3. (→

u ±−

v) =−

u ±−→
  
v
4. (→

u ·−

v) =→

u ·−
 →
v +−

u ·−


v
5. (→

u ×−v) =−
→ →
u ×− →v +−
→u ×−→
 
v
 
6. (−

u ·−

v ·−

w) = − →
u ·−
 → −
v ·→w + (−

u ·−

v ·−

w ) + (−

u ·−

v ·→


w )


− →

dF
Illustration 3.10: If F (t) has a constant magnitude, then prove that dt is


perpendicular to F (t).
Solution:


F (t)
− hasa constant magnitude
→ 
∴  F (t) = constant
72 Vector Differential Calculus


− →
− − 
→  2
∴ F (t) · F (t) =  F (t) = constant

d −→ − →
∴ F ·F =0
dt

− →

→ dF
− dF − →
⇒ F · + ·F =0
dt dt


→ dF

⇒ 2F · =0
dt


→ dF

⇒ F · =0
dt

− →

Since, F · dF
= 0, dF
dt dt is perpendicular to F (t). Hence proved.


Illustration 3.10: If F (t) has a constant direction, then prove that


→ dF

F × = 0.
dt

−
Solution: 
→  →
− →

Let  F (t) = φ (t). Let G (t) be a unit vector in the direction of F (t) so

− →

that F (t) = φ (t) G (t).

− →

dF d G dφ − →
∴ =φ + G (i)
dt dt dt

− →
− →

If F (t) has constant direction, so has G (t). Thus G (t) is a constant vector


dG
and dt =0
From (i).


dF dφ −

= G
dt dt




→ dF →
− dφ −

∴F × = φG × G
dt dt
−
→ − → dφ
=φ G ×G =0
dt
Illustration 3.11: Find the angle between the tangents to the curve x =
t2 , y = 2t, z = −t3 at the points t = 1 and t = −1.
3.6 Gradient of a Scalar Field and Directional Derivative 73

Solution:
Let −→
r be the position vector of any point (x, y, z) on the curve, then


r (t) = (x (t) , y (t) , z (t)) = x (t) î + y (t) î + z (t) k̂
∴−→r = t2 î + 2tĵ − t2 k̂
→ d→
− −
r
∴ T = = 2tî + 2ĵ − 3t2 k̂
dt
is a vector along the tangent at any point t.

− →

∴ T 1 = 2î + 2ĵ − 3k̂ and T 2 = −2î + 2ĵ − 3k̂ are the vectors along
the tangents at t = 1 and t = −1 respectively.

− →

If θ be the angle between T 1 and T 2 , Then
→ −
− →
T1· T2 2 (−2) + 2 (2) − 3 (−3) 9
cos θ = −
→  −
→  = √4 + 4 + 9 √4 + 4 + 9 = 17
 T 1  T 2
9
∴ θ = cos−1 .
17

3.6 Gradient of a Scalar Field and Directional Derivative


The vector differential operator is denoted by ∇ (read as del or nabla) and
defined as
∂ ∂ ∂
∇ = î + ĵ + k̂
∂x ∂y ∂z
and this vector operator has properties analogous to those of ordinary vectors.
It is very useful in defining three important quantities gradient, divergence,
and curl.

3.6.1 Gradient of a Scalar Field


For a given scalar function f (x, y, z) the gradient of f , is written as grad f
or ∇f is the vector function defined by
∂f ∂f ∂f
grad f = ∇f = î + ĵ + k̂
∂x ∂y ∂z

3.6.1.1 Properties of Gradient


1. The projection of ∇f in any direction is equal to the derivative of f in
that direction.
74 Vector Differential Calculus

Figure 3.6 Represents the directional derivative

2. The gradient of f is a vector normal to the surface f (x, y, z) = c =


constant. So, the angle between any two surfaces f (x, y, z) = c1 and
g (x, y, z) = c2 is the angle between their corresponding normal given
by ∇f and ∇g respectively.

3. The gradient at any point P gives the maximum rate of change of f in


the direction of maximum increase of f at the point P .

3.6.2 Directional Derivative


If f = f (x, y, z) then ∂f ∂f ∂f
∂x , ∂y , ∂z are the derivatives (rate of change) of
f in the “direction” of the coordinate axes OX, OY , and OZ respectively.
This concept can be extended to define a derivative of f in a given direction
−−→
P Q (see Figure 3.6)


Let P be a point in space and b be a unit vector from P in the given
direction. Let s be the arc length measured from P to another point Q along


the ray C in the direction b . Then the directional derivative of f at the point

− df
P in the direction of b is denoted D− → f or
b ds and defined by (see Figure 3.6)

df f (Q) − f (P )
D−
→f =
b
= lim (s = distance between P and Q)
ds s→0 s


where Q is a variable point on the ray C in the direction of b and the ray C
is given by

−r (s) = x (s) î + y (s) ĵ + z (s) k̂ (s ≥ 0)


Next, we use Cartesian coordinates, and for b a unit vector. Now consider
the function

f (s) = f (x, y, z) = f (x (s) , y (s) , z(s))


3.6 Gradient of a Scalar Field and Directional Derivative 75

and it’s derivative with respect to the arc length s of C. Hence, assuming that
f has continuous partial derivatives and applying the chain rule, we obtain
df ∂f ∂x ∂f ∂y ∂f ∂z
D−
→f =
b
= + +
ds ∂x ∂s ∂y ∂s ∂z ∂s
∂f ∂f ∂f dx dy dz
= î + ĵ + k̂ · î + ĵ + k̂
∂x ∂y ∂z ds ds ds
! "
∂ ∂ ∂ →

= î + ĵ + k̂ f · b
∂x ∂y ∂z
d−→r dx dy dz
∵ = î + ĵ + k̂
ds ds ds ds
df → −
− →
∴ D−→f =
b
= ∇f · b = b · grad f
ds
Thus, the directional derivative of f at any point P is the dot product of


unit vector b and grad f .
Note that the directional derivative of the vector −

a of any length (= 0) is

df −

a 1 →
Dâ f = = grad f · = ∇f · → −
a = ∇f · â
ds →

|a| −
|a|


where â = a

− is the unit vector in the direction of the vector −

a.
| |
a

3.6.2.1 Properties of Gradient


1. The function f increases most rapidly when cosθ = 1 (i.e., when θ =
0) or when −

a is the direction of ∇f . The derivative in this direction is

a f = |∇f | cos(0) = |∇f | .


D−

2. Similarly, f decreases most rapidly when in the direction of −∇f . The


derivative in this direction is

a f = |∇f | cos(π) = − |∇f | .


D−

3. Any direction −

a orthogonal to a gradient ∇f = 0 is a direction of zero
change in f because θ then equals π/2 and
π 
D a f = |∇f | cos

− = |∇f | · 0 = 0
2
76 Vector Differential Calculus

These properties hold in both two as well as three dimensions.


Illustration 3.12: (a) Gradient as a Surface normal vector,  find a unit normal
vector n of the cone of revolution z 2 = 4 x2 + y 2 at the point P :
(1, 0, 2) . (b) If ∅ = 3x2 y − y 3 z 2 , find grad ∅ at the point (1, −2, −1).
Solution:

(a) The cone is a level surface


 
f = 0 for f (x, y, z) = 4 x2 + y 2 − z 2 .
Thus, ∇f = grand f = 8x î + 8y ĵ − 2z k̂ and at P , grad f = 8î − 4k̂.
Hence, a unit normal vector of the cone at P is

1 8î − 4k̂
n̂ = grad f = 
|grad f |
(8)2 + (−4)2

8î − 4k̂ 2 1
√ = = √ î − √ k̂.
80 5 5
 
(b) Here, ∇φ = 6xy î + 3x2 − 3y 2 z 2 ĵ − 2y 3 z k̂

∇at(1, −2, 1) = −12î − 9ĵ − 16k̂.

Illustration
 3.13: Find the derivative of f (x, y) = x2 sin2y at the point
1, π2 in the direction of −

v = 3î − 4ĵ.
Solution:
Consider,  
∇f = (2xsin2y ) î + 2x2 cos2y ĵ
and  π
(∇f ) 1, = 2sinπ î + (2cosπ ) ĵ
2
= 0 î − 2ĵ = −2ĵ
The direction of −

v is the unit vector obtained by dividing →

v by its length:


v 3 4
v̂ = →− = î − ĵ
|v| 5 5
3.6 Gradient of a Scalar Field and Directional Derivative 77

 
The derivative of at 1, π2 in the direction of v is therefore
  3 4
∇f |(1, π ) · v̂ = −2ĵ · î − ĵ
2 5 5
8 8
=0+ =
5 5

Illustration 3.14: (a) Find the derivative of f = (x, y, z) = x3 − xy 2 − z at


P0 (1, 1, 0) in the direction of −→
v = 2î − 3ĵ + 6k̂. (b) In what directions
does f change most rapidly at p0 , and what are the rates of change in these
directions?
Solution:

(a) (∇f ) P0 (1, 1, 0) = 2î − 2ĵ − k̂


The direction of −

v is


v 2î − 3ĵ + 6k̂ 2 3 6
û = → = = î − ĵ + k̂
|−
v| 7 7 7 7

The derivative of f and P0 in the direction of −



v is
  2 3 6
∇f |(1,1,0) · û = 2î − 2ĵ − k̂ · î − ĵ + k̂
7 7 7
4 6 6 4
= + − =
7 7 7 7

(b) The function increases most rapidly in the direction of ∇f = 2î −


2ĵ − k̂ and decreases most rapidly in the direction of −∇f . The rates of
change in the directions are, respectively
 √
|∇f | = (2)2 + (−2)2 + (−1)2 = 9 = 3 and − |∇f | = −3.

Illustration 3.15: If −

r = xî + y ĵ + z k̂, Show that

(i) ∇r = r̂

(ii) ∇rn = nrn−2 −



r
78 Vector Differential Calculus

Solution:

(i) grad r = ∇r = î ∂x
∂r
+ ĵ ∂r
∂y + k̂ ∂r
∂z
x y  z 
= î + ĵ + k̂
r r r
xî + y ĵ + z k̂ →

r
= = =− →
r
r r


r
∵ − = r̂ unit vector
|→r|

(ii) grad rn = ∇rn


∂ n ∂ n ∂ n
= î r + ĵ r + k̂ r
∂x ∂y ∂z
∂r ∂r ∂r
= î nrn−1 + ĵ nrn−1 + k̂ nrn−1
∂x ∂y ∂z
 x   y   z
= î nrn−1 . + ĵ nrn−1 . + k̂ nrn−1 .
 r  r r
n−2 n−2 −

= nr xî + y ĵ + z k̂ = nr r

Illustration 3.16: Find thegradient of


f (x, y, z) = 2z 3 − 3 x2 + y 2 z + tan−1 (xz) at (1, 1, 1).
Solution:
! "
1
∇f = î −6xz + · z + ĵ [−6yz]
1 + x2 z 2
! "
2
 2 2
 1
+ k̂ 6z − 3 x + y + ·x
1 + x2 y 2
! " ! "
1 1
∇f(1, 1, 1) = î −6 + + ĵ [−6] + k 6 − 6 +
2 2
! "
−11 1
= î − 6ĵ + k̂
2 2
 
∴ The gradient of f at (1, 1, 1) = 12 −11î − 12ĵ + k̂
     
Illustration 3.17: If ∇φ = y 2 î + 2xy + z 3 ĵ + 3yz 2 k̂, determine φ.
3.6 Gradient of a Scalar Field and Directional Derivative 79

Solution:
∂φ ∂φ ∂φ
∇φ = î + ĵ + k̂
∂x ∂y ∂z
 2    
= î y + ĵ 2xy + z + k̂ 3yz 2
3

∂φ ∂φ   ∂φ
∴ = y 2 (i) , = 2xy + z 3 (ii), = 3yz 2 (iii)
∂x ∂y ∂z
Integrating (i), (ii) and (iii) partially with respect to x, y, z respectively,

φ = xy 2 + f1 (y, z)
φ = xy 2 + yz 3 + f2 (z, x)
φ = yz 2 + f1 (x, y)
φ = xy 2 + yz 3 + c (c is an arbitrary constant)

Illustration 3.18: Find the magnitude and the direction of the greatest change
of u = xyz 2 at (1, 0, 3).
Solution:
∂u ∂u ∂u
∇u = î + ĵ + k̂
∂x ∂y ∂z
 2  2
= î yz + ĵ xz + k̂ (2xyz)
∴ ∇u at (1, 0, 3) = 9ĵ
∴ The magnitude of the greatest change of u = |∇u| = 9 and its
direction is along the y-axis.
Illustration 3.19: The temperature at any point in space is given by T =
xy + yz + zx. Determine the derivative of T in the direction of the vector
3î − 4k̂ at the point (1, 1, 1).
Solution:
The derivative of T in the direction of the vector 3î − 4k̂ at the point
P (1, 1, 1) is given by (∇T )p · −

a ; where −

a = 3î − 4k̂.
Now,
∂ ∂
∇T = î (xy + yz + zx) + ĵ (xy + yz + zx)
∂x ∂y

+ k̂ (xy + yz + zx)
∂z
80 Vector Differential Calculus

= (y + z) î + (z + x) ĵ + (x + y) k̂
∴ (∇T )p(1, 1, 1) = 2î + 2ĵ + 2k̂
 
∴ Directional Derivative of T = (∇T )P · â = 2î + 2ĵ + 2k̂ ·
 
3î − 4k̂

9 + 16
2 4
= (6 − 8) = −
5 5

Illustration 3.20: Find the directional derivative of g (x, y, z) =




3ex cos (yz) at P0 (0, 0, 0) in the direction of A = 2î + ĵ − 2k̂.
Solution:
∂g ∂g ∂g
∇g = î + ĵ + k̂
∂x ∂y ∂z
= î [3 ex cos (yz) ] + ĵ [−3 ex sin (yz) · z] + k̂ [−3 ex sin (yz) · y]

∴ ∇g at p0 (0, 0, 0) = î (3) + 0 + 0 = 3î

2î + ĵ − 2k̂ 2î + ĵ − 2k̂


 = √ =
4+1+4 3

∴ Directional derivative = (∇g)p0 · Â



2î + ĵ − 2k̂
= 3î ·
3
6+0−0
= =2
3

Illustration 3.21: Find the derivative of f (x, y) = x ey + cos (xy) at the


point (2, 0) in the direction of A = 3î − 4ĵ.
Solution:
3î − 4ĵ 3î − 4ĵ
 = √ =
9 + 16 5
∂f ∂f
∂x = [ey − ysin (xy) ] and ∂y = [x ey − xsin (xy) ]
3.6 Gradient of a Scalar Field and Directional Derivative 81

∴ ∂f
∂x at (2, 0) = e0 − 0 = 1 and ∂f
∂y at (2, 0) = 2 e0 − 2.0 = 2

∇f(2,0) = î (1) + ĵ (2) = î + 2ĵ

The directional derivative of f at (2, 0) in the direction  is therefore


 
  3î − 4ĵ 3−8 −5
(∇f )(2,0) · Â = î + 2ĵ · = = = −1
5 5 5
x2 y2
Illustration 3.22: Find the direction in which f (x, y) = 2 + 2

(a) increases most rapidly and


(b) decreases most rapidly at the point (1, 1).
(c) What are the directions of zero change in f at (1, 1)?

Solution:
(a) The function increases most rapidly in direction of ∇f at (1, 1)
The gradient is
 
(∇f )(1, 1) = xî + y ĵ (1, 1) = î + ĵ

Its direction is


u î + ĵ 1 1
û = →− =  = √ î + √ ĵ
|u|
(1)2 + (1)2 2 2

(b) The function decreases most rapidly in the direction of −∇f at (1, 1)
which is
1 1
−−

u = − √ î − √ ĵ
2 2
(c) The directions of zero change at (1, 1) are the directions orthogonal to
∇f :
1 1
n̂ = − √ î + √ ĵ
2 2
and
1 1
−n̂ = √ î − √ ĵ
2 2
Since û · n̂ = 0 and −û · (−n̂) = 0.
82 Vector Differential Calculus

Illustration 3.22: Find the directional derivative of


φ = 4xz 3 − 3x2 y 2 z at the point (2, −1, 2)

(i) in the direction 2î + 3ĵ + 6k̂

(ii) towards the point (1, 1, −1) or vector î + ĵ − k̂

(iii) along a line equally inclined with coordinate axes.

(iv) along the tangent to the curve


x = et cost, y = et sint, z = et at t = 0

(v) along the direction normal to the surface


x2 + y 2 + z 2 = 9 at (1, 2, 3)

(vi) along the z-axis.

Solution:
∂φ ∂φ ∂φ
= 4z 3 − 6xy 2 z , = −6x2 yz, = 12xz 2 − 3x2 y 2
∂x ∂y ∂z
   
∴ ∇φ = 4z 2 − 6xy 2 z î − 6x2 yz ĵ + 12xz 2 − 3x2 y 2 k̂
∇φ at (2, −1, 2) = 8î + 48ĵ + 84k̂

(i) →

a = 2î + 3ĵ + 6k̂

2î + 3ĵ + 6k̂ 2î + 3ĵ + 6k̂


∴ â = √ =
4 + 9 + 36 7
Directional derivative = ∇φ · â
  2î + 3ĵ + 6k̂
= 8î + 48ĵ + 84k̂ ·
7
16 + 144 + 504 664
= =
7 7
(ii) an along the line joining say P (2, −1, 2) and Q (1, 1, −1) is


a = (1 − 2) î + (1 + 1) ĵ + (−1 − 2) k̂ = −î + 2ĵ − 3k̂
3.6 Gradient of a Scalar Field and Directional Derivative 83

−î + 2ĵ − 3k̂


∴ â = √
14
Directional derivative = ∇φ · â
 
  −î + 2ĵ − 3k̂
= 8î + 48ĵ + 84k̂ · √
14
8 (−1) + 48 (2) + 84 (−3) −164
= √ = √
14 14

(iii) →

a = î + ĵ + k̂
î + ĵ + k̂
∴ â = √
3
Directional derivative = ∇φ .â
 
  î + ĵ + k̂
= 8î + 48ĵ + 84k̂ · √
3
140
= √
3
dy
(iv) dx
dt = et (cost − sint), dt = et (sint + cost ), dz
dt = et

→ d→
− −
r dx dy dz
∴ T = = î + ĵ + k̂
dt dt dt dt
∴ Tat t=0 = î + ĵ + k̂
î + ĵ + k̂
∴ T̂ = √
3

∴ Directional derivative ∇φ · T̂
 
 
î + ĵ + k̂
= 8î + 48ĵ + 84k̂ · √
3
140
= √
3
84 Vector Differential Calculus

(v) Let f (x, y, z) = x2 + y 2 + z 2 = c

∴ ∇f = 2xî + 2y ĵ + 2z k̂

Let −

a = ∇fat(1, 2, 2) = 2î + 4ĵ + 4k̂

2î + 4ĵ + 4k̂ î + 2ĵ + 2k̂


∴ â = √ =
4 + 16 + 16 3
∴ Directional derivative = ∇φ · â
  î + 2ĵ + 2k̂
= 8î + 48ĵ + 84k̂ .
3

8 + 96 + 168 272
= =
3 3
(vi) Directional derivative ∇φ · â = ∇φ · k̂
 
= 8î + 48ĵ + 84k̂ · k̂
= 84

3.6.3 Equations of Tangent and Normal to the Level Curves


At every point (x0 , y0 ) in the domain of f (x, y), the gradient of f is normal
to the level curve through (x0 , y0 ).
∴ Equation of the tangent at (x0 , y0 ) to the level curve, f (x, y) is
(x − x0 ) fx (x0 , y0 ) + (y − y0 ) fy (x0 , y0 ) = 0 (3.17)
and the equation of the normal is
(x − x0 ) (y − y0 )
= (3.18)
fx (x0 , y0 ) fy (x0 , y0 )

Illustration 3.23: Find equations of the tangent and normal to the ellipse
x2 2
4 + y = 2 at the point (−2, 1).
Solution: The ellipse is a level curve of the function
x2
f (x, y) = + y2.
4
3.6 Gradient of a Scalar Field and Directional Derivative 85

The gradient f at (−2, 1) is


x 
∇fat(−2, 1) = î + 2y ĵ = −î + 2ĵ
2 at(−2, 1)

The tangent is the line

(x + 2) (−1) + (y − 1) (2) = 0.

or
x − 2y = −4
Equation of the normal
x+2 y−1
=
−1 2
or
2x + y = −3

3.6.4 Equation of the Tangent Planes and Normal Lines to the


Surfaces
The equation of the tangent plane at P (x0 , y0 , z0 ) to the surface z =
f (x, y) or F (x, y, z) = f (x, y) − z is

(x − x0 ) Fx (x0 , y0 , z0 ) + (y − y0 ) Fy (x0 , y0 , z0 )
+ (z − z0 ) Fz (x0 , y0 , z0 ) = 0 (3.19)

and the equations of the normal lines to the surface through P (x0 , y0 , z0 )are
x − x0 y − y0 z − z0
= =
Fx (x0 , y0 , z0 ) Fy (x0 , y0 , z0 ) Fz (x0 , y0 , z0 )

Illustration 3.24: Find equations for the (a) tangent plane and (b) normal line
at the point P0 (1, 1, 1) on the surface x2 + y 2 + z 2 = 3.
Solution:

F (x, y, z) = x2 + y 2 + z 2 − 3
∴ ∇F = 2xi + 2yj + 2zk
∴ ∇FatP 0 (1, 1, 1) = 2i + 1j + 2k
86 Vector Differential Calculus

∴ The equation of the tangent plane is


2 (x − 1) + 2 (y − 1) + 2 (z − 1) = 0
or 2x + 2y + 2z = 6 or x + y + z = 3
Normal line
x−1 y−1 z−1
= = =t
2 2 2
or x = 2t + 1, y = 2t + 1, z = 2t + 1

3.7 Divergence and Curl of a Vector Field


3.7.1 Divergence of a Vector Field


Let V (x, y, z) = V1 î+V2 ĵ +V3 k̂ be a differentiable vector function, then the

− →
− →

divergence of V is denoted by div V or ∇ · V is the scalar function defined
by

− →
− ∂ ∂ ∂  
div V = ∇ · V = î + ĵ + k̂ · V1 î + V2 ĵ + V3 k̂
∂x ∂y ∂z
∂V1 ∂V2 ∂V3
= + +
∂x ∂y ∂z

− →

For example, if V = 3xz î + 2xy ĵ − yz 2 k̂, then div V = 3z + 2x − 2yz.

3.7.1.1 Physical Interpretation of Divergence




Consider the motion of a fluid with velocity V = Vx î + Vy ĵ + Vz k̂ at a
point A (x, y, z). Consider a small parallelepiped having one of its corners
at A (x, y, z) and edges parallel to the coordinate axes and having magnitude
δx, δy, δz respectively. (See Figure 3.7)
The mass of the fluid entering through the face ABCD per unit time
is Vy δx δz and  that flowing out through the opposite face EFGH is
∂V
Vy + δy δx δz = Vy + ∂yy δy δx δz by using Taylor’s series.
Thus, the rate at which fluidflows out from the elementary volume along
∂V ∂V
the y-direction is Vy + ∂yy δy δx δz − V, δx δz = ∂yy δx δy δz.
Similarly, the rate of outward flow along x and z directions will be given
by ∂V x ∂Vz
∂x δx δy δz and ∂z δx δz respectively.
 Thus, the rate at  which fluid flows out of the volume per unit time is
∂Vx ∂Vy ∂Vz
∂x + ∂y + ∂z δx δy δz.
3.7 Divergence and Curl of a Vector Field 87

Figure 3.7 Represents the parallelopiped

Dividing this by the volume δx, δy, δz of the parallelepiped, we have


the rate of outflow (or rate of loss of fluid) per unit volume per unit time is
∂Vx ∂Vy ∂Vz →

+ + = div V .
∂x ∂y ∂z


Hence, the divergence of V gives the rate of outward flow per unit volume at
a point of the fluid.
From this, we have the following observations.

− →

(1) If there is no gain of the fluid anywhere, then div V = 0 i.e., ∇ · V =
0. This is called the continuity equation for an incompressible fluid or
condition of incompressibility.

(2) Since the fluid is neither created nor destroyed at any point, it is said to
have no sources or sinks.

(3) If the flux entering any element of the space is the same as the leaving it,


i.e., if div V = 0 everywhere then such a vector point function is called
a solenoidal vector function or solenoidal


(4) If V represents an electric flux, div V is the amount of flux that diverges
per unit volume.

− →

(5) If V represents heat flux, div V is the rate at which heat flows from a
point per unit volume.

Illustration 3.25: If −

r = xî + y ĵ + z k̂, show that div r = 3.
88 Vector Differential Calculus

Solution: We know that



− →
− ∂ ∂ ∂  
div V = ∇ · V = î + ĵ + k̂ · V1 î + V2 ĵ + V3 k̂
∂x ∂y ∂z
∂V1 ∂V2 ∂V3
= + +
∂x ∂y ∂z

∴ div −

r = ∇·−

r = ∂
∂x (x) + ∂
∂y (y) + ∂
∂z (z)

=1+1+1=3

− →

Illustration 3.26: If F = x2 z î − 2y 3 z 2 ĵ + xy 2 z k̂ then find div F at the
point (1, −1, 1) .
Solution: We know that

− →
− ∂ ∂ ∂  
div V = ∇ · V = î + ĵ + k̂ · V1 î + V2 ĵ + V3 k̂
∂x ∂y ∂z
∂V1 ∂V2 ∂V3
= + +
∂x ∂y ∂z

− →
−  2     
∴ div F = ∇· F = ∂
∂x

x z + ∂y −2y 3 z 2 + ∂
∂z xy 2 z

= 2xz − 6y 2 z 2 + xy 2

at point (1, −1, 1)




∴ ∇· F (1,−1,1) = 2 (1) (1) − 6(−1)2 (1)2 + (1) (−1)2 = −3.

Illustration 3.27: Show that the vector




V = (x + 3y) î + (y − 2z) ĵ + (x − 2z) k̂ is solenoidal.
Solution: A vector V is solenoidal if its divergence is zero.

− ∂ ∂ ∂
∇· V = (x + 3y) + (y − 2z) + (x − 2z)
∂x ∂y ∂z
= 1 + 1 − 2 = 0.

→ →

Since ∇ · V = 0 therefore vector V is solenoidal.
3.7 Divergence and Curl of a Vector Field 89

3.7.2 Curl of a Vector Field




Let V = (x, y, z) = V1 î + V2 ĵ + V3 k̂ be a differentiable vector function,

− →
− →

then the curl or rotation of V , written curl V or ∇ × V is the vector function
defined by  
 î ĵ k̂ 

− →  ∂
− ∂ 
curl V = ∇ × V =  ∂x ∂y ∂
∂z 
 V V V 
1 2 3

∂V3 ∂V2 ∂V1 ∂V3 ∂V2 ∂V1


= î − + ĵ − + k̂ −
∂y ∂z ∂z ∂x ∂x ∂y

3.7.2.1 Physical Interpretation of Curl


To understand the curl of a vector physically, consider the motion of a rigid
body rotating about a fixed axis passing through O. Let −→
ω = ω1 î+ω2 ĵ +ω3 k̂


be the angular velocity of the rigid body, V the linear velocity of any point
P (−
→r ) on the body, where −
→r = xî + y ĵ + z k̂, then


V =→

ω ×−

r
 
 î ĵ k̂ 
→ −
− → →
−  

∴ V = ω × r =  ω1 ω2 ω3 
 x y z 
= î (ω2 z − ω3 y) + ĵ (ω3 x − ω1 z) + k̂(ω1 y − ω2 x)

Now,
 
 î ĵ k̂ 

− → 
− 

curl V = ∇ × V =  ∂
∂x

∂y

∂z 
 ω z−ω y ω x−ω z ω y−ω x 
2 3 3 1 1 2

= (ω1 + ω1 ) î + (ω2 + ω2 ) ĵ + (ω3 + ω3 ) k̂


 
= 2 ω1 î + ω2 ĵ + ω3 k̂

− 1 →

∴ curl V = 2 −

ω ⇒−

ω = curl V
2
Thus, the angular velocity of rotation at any point is equal to half the curl
of the linear velocity at that point of the body.
This indicates that the curl of a vector field has something to do with the


rotational properties of the field. If the field V is that due to a moving fluid,
90 Vector Differential Calculus

for example, then a paddlewheel placed at various points in the field would

− →

tend to rotate in regions where curl V = 0, while if curl V =0 in the region


there would be no rotation and the field V is then called irrotational. A field
that is not irrotational is sometimes called a vortex field.
Illustration 3.28: If −→r = xî + y î + z k̂. show that curl −

r = 0.
Solution:  
 î ĵ k̂ 

− →
−  ∂
curl r = ∇ × r =  ∂x ∂
∂y
∂ 
∂z 
 x y z 
∂z ∂y ∂ ∂ ∂y ∂x
= î − + ĵ x− z + k̂ −
∂y ∂z ∂z ∂x ∂x ∂y
= î (0) + ĵ (0) + k̂ (0) = 0

− →

Illustration 3.29: If F = xz 3 î − 2x2 yz ĵ + 2yz 4 k̂, find curl F at the point
(1, 1, 1).
Solution:
 
 î ĵ k̂ 

− 
∇ × F =  ∂x ∂ ∂
∂y

∂z 

 xz 3 −2x2 yz 2yz 4 
! " ! "

− ∂  4
 ∂  2
 ∂  3 ∂  4

∴∇ × F = î 2yz − −2x yz + ĵ xz − 2yz
∂y ∂z ∂z ∂x
! "
∂  2
 ∂  3
+ k̂ −2x yz − xz
∂z ∂y
 
= 2x2 + 2x2 y î + 3xz 2 ĵ − 4xyz k̂


∴ curl F at (1, −1, 1) = 3ĵ + 4k̂.
Illustration 3.30: Find constants a, b, c so that


V = (x + 2y + az) î + (bx − 3y − z) ĵ + (4x + cy + 2z) k̂ is irrota-
tional.
Solution:  →
− 
 →
− −
→ 
 i j k 

− →

curl V = ∇ × V =  ∂
∂x

∂y

∂z


 x + 2y + az bx − 3y − z 4x + cy + 2z 

= (c + 1) î + (a − 4) ĵ + (b − 2) k̂
3.7 Divergence and Curl of a Vector Field 91

This equals zero when a = 4, b = 2, c = −1 and




V = (x + 2y + 4z) î + (2x − 3y − z) ĵ + (4x − y + 2z) k̂


− →

Illustration 3.31: If a vector F is irrotational then show that F = ∇φ =

− →

grad φ, where φ is a scalar point function of F or scalar potential of F .

− →

Solution: Let curl F = 0 where F = F1 î + F2 ĵ + F3 k̂
 
 î ĵ k̂ 
 ∂
∴  ∂x ∂y∂ ∂ 
∂z 
=0
 F F F 
1 2 3

i.e.,
∂F3 ∂F2 ∂F1 ∂F3 ∂F2 ∂F1
î − + ĵ − + k̂ − =0
∂y ∂z ∂z ∂x ∂x ∂y
∂F3 ∂F2 ∂F1 ∂F3 ∂F2 ∂F1
∴ − = 0, − = 0, − =0
∂y ∂z ∂z ∂x ∂x ∂y
i.e.,
∂F3 ∂F2 ∂F1 ∂F3 ∂F2 ∂F1
= , = , =
∂y ∂z ∂z ∂x ∂x ∂y
These three conditions are satisfied when
∂φ ∂φ ∂φ
F1 = , F2 = , F3 = ,
∂x ∂y ∂z
where φ is a function of x, y, z

− ∂φ ∂φ ∂φ
∴ F = î + ĵ + k̂ = ∇φ
∂x ∂y ∂z


φ is known as the scalar point function of F . A vector field F which can be


derived from a scalar field φ so that F = ∇φ is called a conservative vector
field and φ is called the scalar potential.

− →

Note that conversely if F = ∇φ, then ∇ × F = 0.


Illustration 3.32: Prove that F = (y 2 cosx+z 3 ) î+(2y sinx−4)ĵ +3xz 2 k̂
is irrotational and find its scalar potential.
92 Vector Differential Calculus

Solution:
 
 î ĵ 

→ 
− 

∇ ×F = ∂
∂x

∂y

∂z  = 0
 y 2 cosx + z 3 2ysinx − 4 3xz 
2

on simplification
Hence,

→ ∂φ ∂φ ∂φ
F = ∇φ = î + ĵ + k̂
∂x ∂y ∂z
∂φ ∂φ ∂φ
∴ (i) = y 2 cosx + z 3 , (ii) = 2ysinx − 4 , (iii) = 3xz 2
∂x ∂y ∂z
∴ from (i), φ = y 2 sinx + xz 3 + f1 (y, z) .
from (ii), φ = y 2 sinx − 4y + f2 (z, x) .
from (iii), φ = xz 3 + f3 (x, y).
∴ φ = y 2 sinx + xz 3 − 4y + c , where c is an arbitrary constant.
Illustration 3.33: A fluid motion is given by

→  
V = (ysinz − sinx ) î + (xsinz + 2yz) ĵ + xycosz + y 2 k̂.

is motion irrotational? If so, find the velocity potential.


Solution:
 
 î ĵ k̂ 
→ 
− ∂ ∂ ∂


Curl V =  ∂x ∂y ∂z 
 ysinz − sinx xsinz + 2yz xycosz + y 2 

= î [x cosz + 2y − x cosz − 2y] + ĵ [ y cosz − y cosz]


+ k̂ [sinz − sinz]
= î (0) + ĵ (0) + k̂ (0) = 0


Since curl V = 0, the motion is irrotational.


Hence V = ∇φ = î ∂φ ∂φ
∂x + ĵ ∂y + k̂ ∂z
∂φ

∂φ
∂x = y sinz − sinx (i)
∂φ
∂y = y sinz + 2yz (ii)
∂φ
∂z = xy cosz + y 2 (iii)
From (i), φ = xy sinz + cosx + f1 (y, z)
3.7 Divergence and Curl of a Vector Field 93

From (ii), φ = xy sinz + y 2 z + f2 (z, x)


From (iii), φ = xy sinz + zy 2 + f3 (x, y)
∴ φ = xy sinz +y 2 z +cosx+c, where c an arbitrary constant is required
velocity potential.

− →

Illustration 3.34: If A = 2yz î − x2 y ĵ + xz 2 k̂, B = x2 î + yz ĵ − xy k̂, and
φ = 2x2 yz 3 , find
−
→ 
(i) A · ∇ φ


(ii) A · ∇φ and compare them.
−
→ 
(iii) A × ∇ φ


(iv) A × ∇φ and compare them.

Solution:

(i)
!  "
2 2 ∂ ∂ ∂
(A · ∇) φ = 2yz î − x y ĵ + xz k̂ · î + ĵ + k̂ φ
∂x ∂y ∂z
∂ ∂ ∂  2 3
= 2yz − x2 y + xz 2 2x yz
∂x ∂y ∂z
∂  2 3 ∂  2 3 ∂  2 3
= 2yz 2x yz − x2 y 2x yz + xz 2 2x yz
∂x ∂y ∂z
       
= (2yz) 4xyz 3 − x2 y 2x2 z 3 + xz 2 6x2 yz 2
= 8xy 2 z 4 − 2x4 yz 3 + 6x3 yz 4

(ii)

→   ∂φ ∂φ ∂φ
A · ∇φ = 2yz î − x2 y ĵ + xz 2 k̂ · î + ĵ + k̂
∂x ∂y ∂z
   
= 2yz î − x2 y ĵ + xz 2 k̂ · 4xyz 3 î + 2x2 z 3 ĵ + 6x2 yz 2 k̂
= 8xy 2 z 4 − 2x4 yz 3 + 6x3 yz 4
−
→ 
Comparison with (i) illustrates the result A · ∇ φ = A · ∇φ.
94 Vector Differential Calculus

(iii)
!  "

→ 2 2 ∂ ∂ ∂
( A × ∇)φ = 2yz î − x y ĵ + xz k̂ × î + ĵ + k̂ φ
∂x ∂y ∂z
 
 î ĵ k̂ 

=  2yz −x2 y xz 2  φ
 ∂ ∂ ∂ 
∂x ∂y ∂z
$
∂ ∂ ∂ ∂
= î −x2 y − xz 2 + ĵ xz 2 − 2yz
∂z ∂y ∂x ∂z
%
∂ ∂
+ k̂ 2yz + x2 y φ
∂y ∂x
∂φ ∂φ ∂φ ∂φ
= − x2 y + xz 2 î + xz 2 − 2yz ĵ
∂z ∂y ∂x ∂z
∂φ ∂φ
+ 2yz − x2 y k̂
∂y ∂x
−
→     
A × ∇ φ = − 6x4 y 2 z 2 + 2x3 z 5 î + 4x2 yz 5 î
   
+ 4x2 yz 5 − 12x2 y 2 z 3 ĵ + 4x2 yz 4 + 4x3 y 2 z 3 k̂

(iv)
−
→    ∂φ ∂φ ∂φ
A × ∇ φ = 2yz î − x2 y ĵ + xz 2 k̂ × î + ĵ + k̂
∂x ∂y ∂z
 
 î ĵ k̂ 

=  2yz −x y xz 2 
 2
 ∂φ ∂φ ∂φ 
∂x ∂y ∂z
∂φ ∂φ ∂φ ∂φ
= −x2 y − xz 2 î + xz 2 − 2yz ĵ
∂z ∂y ∂x ∂z
∂φ ∂φ
+ 2yz + x2 y k̂
∂y ∂x
   
= − 6x4 y 2 z 2 + 2x3 z 5 î + 4x2 yz 5 − 12x2 y 2 z 3 ĵ
 
+ 4x2 y 3 z 4 + 4x3 y 2 z 3 k̂
−→  →

Comparison with (iii) illustrates the result A × ∇ φ = A × ∇φ.
3.7 Divergence and Curl of a Vector Field 95

Illustration 3.35: Find the scalar potential function f for




A = y 2 î + 2xy ĵ − z 2 k̂ .

Solution:  
 î ĵ 


− →  ∂
− 
Curl A = ∇ × A =  ∂x ∂ ∂ 
∂y ∂z 
 y2 2xy −z 2 

= î (0) + ĵ (0) + k̂ (2y − 2y) = 0



− →
− →

Since and A = 0, A = ∇φ, where φ is a scalar potential function of A

−   ∂φ ∂φ ∂φ
A = ∇φ =⇒ y 2 î + 2xy ĵ − z 2 ĵ = î + ĵ + k̂
∂x ∂y ∂z
∂φ
∂x = y 2 (i)
∂φ
∂y = 2xy (ii)
∂φ
∂z = −z 2 (iii)
from (i) φ = xy 2 + f1 (y, z)
from (ii) φ = 2xy 2/2 + f2 (z, x) = xy 2 + f2 (y, z)
from (iii) φ = −23 /3 + f3 (x, y)
∴ φ = xy 2 − z 3 /3 + C. where C is an arbitrary constant.
Illustration 3.36: Prove that rn −
→r is irrotational.
Solution:
To prove rn −

r irrotational, find ∇ × (rn −

r ) i.e., curl (rn −

r ).
/ ∂ n −
curl (rn −
→r)= î × (r , →r)
0 ∂x 1
/ x
= î × nrn−1 r̂ + rn î
/  r 
= {nr n−2
× î × −→r + rn (î × î)}
/  
= n rn−2 x y k̂ − z ĵ + 0
0      1
= n rn−2 x y k̂ − z ĵ + y z î − xk̂ + z xĵ − y î
=0
∴ rn →

r is irrotational.
96 Vector Differential Calculus

−  
Illustration 3.37: Show that F = 2xy z î + x2 z + 2y ĵ + x2 y k̂ is


irrotational and find a scalar function φ such that F = grad ∅ .
Solution:  
 î
 ĵ k̂ 

− ∂ 
∇ × F =  ∂x ∂ ∂
∂y ∂z 
 2xyz x2 z + 2y x2 y 
 
= î x2 − x2 + ĵ (2xy − 2xy) + k̂ (2xz − 2xz) = 0


∴ F is irrotational
F = grad φ =⇒ (i) ∂φ ∂φ 2 ∂φ 2
∂x = 2xyz, (ii) ∂y = x z + 2y, (iii) ∂z = x y
Integrating (i), (ii) and (iii) w.r.t. x, y, and z respectively, we get

φ = x2 yz + f1 (y, z)
φ = x2 yz + y 2 + f (z, x)
φ = x2 yz + y 2 + f3 (x, y)
∴ φ = x2 yz + y 2 + c

3.7.3 Formulae for grad, div, curl Involving Operator ∇


3.7.3.1 Formulae for grad, div, curl Involving Operator ∇ Once
1. If −

u and −

v are vector point functions and f , a scalar point function.

(i) div (−
→u +− →
v ) = div −

u + div −
→v
or ∇ · ( u + v ) = ∇ · u + ∇ · −

− →
− →
− →v
(ii) div (f u ) = f (div u ) + (grad f )· −

− →
− →
u
or f (∇ · −

u ) + (∇f ) · −

u.

2. (i) ∇ × (f →−u ) = (∇f ) × −→


u + f (∇ × −→
u)
or curl (f u ) = (grad f ) × u + f curl −

− →
− →
u
(ii) ∇ × ( u × v ) = (∇ × v ) u − (∇ · −

− →
− →
− →
− →u) →

v + (→

v · ∇) →

u −


( u · ∇) v →

3. If u and v are scalar point functions

(i) grad (u + v) = grad u+ grad v


(ii) grad (uv) = u (grad v ) + (grad u ) v
The proofs for (3) are very simple so left to the reader.
3.7 Divergence and Curl of a Vector Field 97

Proof:
 
1. (i) ∇ · (−

u +−
→ ∂
v ) = î ∂x + ĵ ∂
∂y

+ k̂ ∂z · (−

u +−

v)
/ ∂
= î · (−

u +−→v)
∂x
/ ∂ / ∂
= î · −→u + î · −

v
∂x ∂x
∂ ∂ ∂
= î + ĵ + k̂ ·u
∂x ∂y ∂z
∂ ∂ ∂
+ î + ĵ + k̂ ·−
→v
∂x ∂y ∂z
=∇·− →u +∇·→ −
v
→∂

(ii) ∇ · (f −
→ ∂
u ) = î ∂x · (f −
→ ∂
u ) + ĵ ∂y · (f −

u ) + k ∂z · (f −
→u)
∂ ∂ ∂
= î· (f −
→u )+ĵ· (f − →
u )+ k̂· (f →

u)
∂x ∂y ∂z
/ ! ∂f ∂−→u
"
= î →

u +f
∂x ∂x
/ ∂f / ∂−
→u
= î · −
→u +f î ·
∂x ∂x
= (∇f ) · − →u + f (∇ · −→
u)
 
2. (i) ∇ × (f →
− ∂
u ) = î ∂x + ĵ ∂y ∂ ∂
+ k̂ ∂z × f−→
u
/ ∂
= î × f−→u
∂x
/ ∂
= î × (f −

u)
∂x
/ ∂f −
→ ∂− →u
= î × u +f
∂x ∂x
/ ∂f −→
/ ∂− →
u
= î × u + î × f
∂x ∂x
/ ∂f / ∂− →u
= î ×− →u +f î
∂x ∂x


= (∇f ) × u + f (∇ × u )→

98 Vector Differential Calculus
2
(ii) ∇ × (→

u ×−

v)= î ∂
∂x × (−

u ×−

v)
/ ∂ −
= î × (→
u ×−→
v)
∂x
/ ∂−
→u ∂−→v
= î × ×−
→v +−→
u ×
∂x ∂x
/ →

∂u / ∂−

v
= î × ×−
→v + î × →

u ×
∂x ∂x

Now
/ ∂−→
u / 3  ∂−→
u ∂−
→u −
4
î × →

× v = →

î · v − î · →
v
∂x ∂x ∂x
−
→ →  → → −
−  
→ →

∵−→
a × b ×− c = b (− a ·→
c)− − →
a · b − c
/  ∂→
−u   ∂−

u   ∂−→
u   ∂−→
u
î · −

v = î · −

v + ĵ · − →v + k̂ · −

v
∂x ∂x ∂y ∂z
∂u ∂u ∂u
= v1 + v2 + v3
∂x ∂y ∂z

where −

v = v1 î + v2 ĵ + v3 k̂

∂ ∂ ∂ →

= v1 + v2 + v3 u
∂x ∂y ∂u
/  ∂− → 3   4

− u ∂ ∂ ∂ →

î · v = v1 î + v2 ĵ + v3 k̂ · î + ĵ + k̂ u
∂x ∂x ∂y ∂z
= (−

v · ∇) − →
u
/ →

∂u → / ∂−→
u −
î · −v = î · →v
∂x ∂x
/ ∂
= î ·−

u − →
v
∂x
3 4
∂ ∂ ∂ →

= î + ĵ + k̂ · u − →
v
∂x ∂y ∂z
= (∇ · − →u) − →v
/ ∂−→
u
∴ î × ×−
→v = (− →v · ∇) −→
u − (∇ · −

u) − →
v
∂x
3.7 Divergence and Curl of a Vector Field 99

Also
/ ∂−→
v / ∂−

v
î × →−u × =− î × × −

u
∂x ∂x
= − (→−
u · ∇) −

v + (∇ · −

v) −

u

(Similar to the previous result)


Hence

∇ × (−

u ×− →v ) = (−

v · ∇) −
→u − (∇ · −

u) −
→v − (−

u · ∇) −→
v + (∇ · −

v) −

u

− →
− →
− →
− →
− →
− →
− →

= (∇ · v ) u − (∇ · u ) v + ( v · ∇) u − ( u · ∇) v


Illustration 3.38: Prove that V = rn −

r is irrotational. Find n when it is also
solenoidal.
Solution:


∇ × V = ∇ × (rn − →
r)
= ∇ (rn ) × −

r + rn (∇ × −
→r)
−
→ 
using ∇ × (φ −

u) = V φ ×− →u + φ (∇ × −

u)

/ ∂ / ∂−
→r
∇ (rn ) = î rn = î nrn−1
∂x
x ∂x
/
= î nr n−1
since r = x + y 2 + z 2
2 2

/ r
=n î rn−2 x
  /  
∴ ∇ r2 = nrn−2 î x = nrn−2 xî + y î + z k̂
= nrn−2 − →r

Since ∇ × −

r =0


Hence ∇ × V = nrn−2 −

r ×−

r + rn (0)

= 0, since −

r ×−

r =0

Hence the vector is irrotational




∇ · V = ∇ · (rn −

r)

= rn (∇ · −

r ) + ∇ (rn ) ·−

r
100 Vector Differential Calculus

using ∇ · (φ −
→u ) = φ (∇ · −

u ) + ∇φ · −

u


since ∇ · r = 3 and ∇r = nr
n n−2 →

r
Hence


∇ · V = 3rn + nrn−2 −

r ·−

r
= 3rn + nrn−2 r2
= (n + 3) rn

− →

If the vector V is solenoidal ∇ · V = 0
∴ (n + 3) rn = 0, i.e., n = −3.

3.7.3.2 Formulae for grad, div, curl Involving Operator ∇ Twice




Given a scalar function f and a vector function V , the following combina-
tions of ∇ twice are possible:

∂2f ∂2f ∂2f


(i) div (grad f) = ∇ · ∇f = ∂x2
+ ∂y 2
+ ∂z 2

− →

(ii) (∇ · ∇) V =∇2 V

−  →

(iii) grad (div V ) = ∇ ∇ · V

(iv) curl (grad f ) = ∇ × ∇f = 0



−  →

(v) div curl V = ∇ · ∇ × V = 0


−  →
−  →
− →

(vi) curl curl V = ∇ × ∇ × V = ∇ ∇ · V − ∇2 V .

Proof:
   
∂f
(i) ∇ · ∇φ = î ∂x

+ ĵ ∂
∂y

+ k̂ ∂z · î ∂x + ĵ ∂f ∂f
∂y + k̂ ∂z

∂2f ∂2f ∂2f


∴ ∇ · ∇φ = ∇2 φ = + +
∂x2 ∂y 2 ∂z 2
2 2 2
The operator ∇2 = ∂x
∂ ∂ ∂
2 + ∂y2 + ∂z2 is called Laplace’s operator or Lapla-

cian and ∇2 f = 0 is called Laplace’s equation. which frequently occurs


in physical and engineering problems.
3.7 Divergence and Curl of a Vector Field 101


− → 2−
− → 2−
→ 2−

(ii) (∇ · ∇) V = ∇2 V = ∂∂xV2 + ∂∂yV2 + ∂∂zV2
 
(iii) ∇ (∇ · ∇) = ∇ ∂V ∂x
1
+ ∂V2
∂y + ∂V3
∂z = which is a vector
 
 î k̂ 
 ĵ
 ∂ ∂ ∂ 
(iv) curl grad f = ∇ × ∇f =  ∂x ∂z  = 0 on simplification
 ∂f ∂y
∂f 
 ∂x ∂f
∂y ∂z


(v) div curl V = ∇ · (∇ × V )
 
 î ĵ k̂ 
→  ∂
− ∂ 
 →

∇ × V =  ∂x ∂y ∂z  , where V = V1 î + V2 ĵ + V3 k̂

 V V V 
1 2 3


− ∂V3 ∂V2 ∂V1 ∂V3 ∂V2 ∂V1
∴ ∇ × V = î − + ĵ − + k̂ −
∂y ∂z ∂z ∂x ∂x ∂y

−  →

∴ div curl V = ∇ · ∇ × V

∂ ∂V3 ∂V2 ∂ ∂V1 ∂V3 ∂ ∂V2 ∂V1


= − + − + −
∂x ∂y ∂z ∂y ∂z ∂x ∂z ∂x ∂y
2
∂ V3 2
∂ V2 ∂ 2 V1 ∂ 2 V3 2
∂ V2 2
∂ V1
= − + − + −
∂x∂y ∂x∂y ∂y∂z ∂y∂x ∂z∂x ∂z∂y
=0
−
→ −  −
→ → −  → →

(vi) We have −

a × b ×→
c = b (−
a ·→
c)− −→
a · b −
c
Treating ∇ as a vector if we take

→ →
− →

a = ∇ , b = ∇, and − →c = V , we have
 →

∇ × ∇× V =∇ (∇· ∇) − (∇· ∇) V
 −
→ →

∴ ∇ × (∇ × V ) = ∇ ∇ · V − ∇2 V


− →
− →

i.e., curl curl V = grad div V − ∇2 V
102 Vector Differential Calculus

− →

Illustration 3.39: If ρ E = ∇φ, where ρ, φ are scalar fields and E is a vector

− →

field, prove that E · curl E = 0.
Solution:

− 1
E = ∇φ
ρ

− →
− 1
curl E = ∇ × E = ∇ × ∇φ
ρ
1 1
=∇ × ∇φ + ∇ × (∇φ)
ρ ρ
[∵ ∇ × (f u ) =∇f ×−

− →u +f (∇×−

u)
1
=∇ × ∇φ + 0 [∵ ∇ × ∇f = 0
ρ

− → −
− →    
∴ E curl E = E × ∇ ρ1 × ∇φ

1  →

=∇ · ∇φ × E
ρ
−
→ → − → → −
[∵ −

a · b ×− c = b · (−
c ×→
a)
1  → −
− → 
=∇ · ρE×E
ρ
→ −
− →
= 0 since E × E = 0
2
Illustration 3.40: Prove that ∇2 f (r) = f  (r) + r f  (r).
Solution:
∇2 f (r) = ∇ · {∇ f (r)} =div {grad f (r)}
∂2 ∂2 ∂2
= + + f (r)
∂x2 ∂y 2 ∂z 2
Now ∂
∂x f (r) = ∂
∂x f (r) · ∂r
∂x = f  (r) · x
r since r2 = x2 + y 2 + z 2
∂2 ∂ #  .
∴ f (r) = f (r) · x · r−1
∂x2 ∂x
x
= f (r) r · 1 + f  (r) · · x r−1 − f  (r) x2 r−3
 −1
r
f  (r) x2 f  (r) x2 f  (r)
= + −
r r2 r3
3.7 Divergence and Curl of a Vector Field 103

Similarly,

∂2 f  (r) y 2 f  (r) y 2 f  (r)


f (r) = + −
∂y 2 r r2 r3
and
∂2 f  (r) z 2 f  (r) z 2 f  (r)
f (r) = + −
∂z 2 r r2 r3
Adding, we have

∂2 ∂2 ∂2 3f  (r) f  (r)
+ + f (r) = + f  (r) −
∂x2 ∂y 2 ∂z 2 r r

2f  (r)
= + f  (r)
r
2f  (r)
∴ ∇2 f (r) = div grad f (r) = r + f  (r).
Illustration 3.41: Maxwell’s equations of the electromagnetic theory are

→ →


→ →
− →
− ∂H → ∂E

∇ · E = 0, ∇ · H = 0, ∇ × E = − , ∇×H =
∂t ∂t

− →

Show that E and H satisfy wave equations
→ 2−
− → → 2−
− →
(i) ∇2 E = ∂∂tE
2 and (ii) ∇2 H = ∂∂tH
2

Solution:
 →

−
→ ∂H ∂  →

∇× ∇× E =∇× − =− ∇ ×H
∂t ∂t

∂ ∂E ∂2E
=− =− (i)
∂t ∂t ∂t2
But
 →
−  →
− →
− →

∇ × ∇ × E = ∇ ∇ · E − ∇2 E = −∇2 E (ii)

Then from (i) are (ii), we have




2−
→ ∂2 E
∇ E =
∂t2
104 Vector Differential Calculus

Now
 →

−
→ ∂E ∂  →

∇× ∇×H =∇× = ∇× E
∂t ∂t

−  →

∂ ∂H ∂2 E
= − =− 2 (iii)
∂t ∂t ∂t
but
 →
−  →
− →
− →

∇ × ∇ × H = ∇ ∇ · H − ∇2 H = −∇2 H (iv)

from (iii) and (iv), we have




∂2 H
2−

∇ H= 2
∂t
2
The equation ∇2 u = ∂∂t2u
2 2 2 ∂2u
i.e., ∂∂xu2 + ∂∂yu2 + ∂∂zu2 = ∂t2
is called the wave equation.

3.8 Exercise
1. Determine k and τ for the following curve:
(i) x = t cos t , y = t sin t , z =λt at t = 0 
2 3λ
Answer: k = 1+λ2
, τ= 2(1+λ2 )

(ii) x = 3t, y = 3t2 ,z = 2t3 


3
 2
Answer: = σ = 2 1 + 2t2 , = k1 , σ = τ1
 
(iii) x = a 3t − t3 , y = 3 at2 , z = a(3t + t3 )
  2 
Answer: = σ = 3a 1 + t2

2. For the curve x = 3t, y = 3t2 , z = 2t2 at t = 1 Prove that = σ=


3
 
2 2.
2 1 + 2t

3. For the curve x = 2 log t, y = 4t, z = 2t2 + 1 at t = 1 prove that


= σ = 9.
4. Find curvature and torsion for the curve −

r = cos t î + sin t ĵ + tk̂.
2 2
Also, prove that 2 κ + τ = 1.
 
Answer: k = 12 , τ = 12
3.8 Exercise 105

5. Find curvature and torsion for the curve x = t cos t , y = t sin t , z =


λt at t = 0.  
2 3λ
Answer: k = 1+λ 2 , τ = 2(1+λ2 )

6. For the curve x = a cos θ , y = a sin θ , z = a θtan α , find ρ.


 
Answer:ρ = asec2 α

7. For the curve x = a cos θ , y = a sin θ , z = λθ, find k and τ .


 
a λ
Answer: k = a2 +λ 2 , τ = a2 +λ2

8. Show that for the curve


z 2x2
x2 − y 2 = c2 , y = xtanh , = σ= .
c c
 
9. For the curve x = tan−1 s , y = √1 log s2 + 1 , z = s − tan−1 s
2

2
show that k = τ = s2 +1
.
   
10. For the curve x = a 3u − u3 , y = 3au2 , z = a 3u + u3
1
show that k = τ = 3a(1+u2 )2
.

11. For the curve x = 4a cos3 u , y = 4a sin3 u , z = 3c cos2u


a
prove that k = 6(a2 +c2 )sin2u
.

12. Find the length of the curve x = et cost , y = et sint , z =


et between t = t1 and t = t2 .
 √  
Answer: 3 et2 − et1

13. Find the length of the arc of the curve x = 3t, y = 3t2 , z = 2t3
between t = 0 and t = 1.
(Answer: 5)
3 3 3a 2
 x = a cos t, y = a sin t , z = 2 cos t
14. Find the lengthof the curve
3a
from the point a, 0, 2 to the point (0, a, 0).
 
3a
Answer: √ 2
106 Vector Differential Calculus

15. Find the length of the curves −



r (t) = 2tî + 3sin2t ĵ + 3cos2t k̂ on the
interval 0 ≤ t ≤ 2π.
 √ 
Answer: 2 10t
16. Find the magnitude of the velocity and acceleration of a particle which
moves along the curve x = 2 sin3t , y = 2 cos3t , z = 8t at any time
t > 0. Find the unit tangent vector to the curve.
  
1
Answer: 10, 18, 10 (6 cos3t ) î − (6 sin3t ) ĵ + 8k̂

17. A particle moves along a plane curve such that its linear velocity is
perpendicular to the radius vector. Show that the path of the particle
is a circle.
18. Find the magnitude of the tangential components of acceleration at any
time t of a particle whose position at any time t is given by x = cost +
tsint , y= sint − tcost .
(Answer: 1)
19. Show that the length of the √ curve 2x = a (cos 3θ + cos θ ) , 2y =
a
 (sin 3θ +
√  sin θ ) z = 3 a cos θ measured from the point
a, 0, 3 a in 2aθ.
 
d2 V d3 V
d
20. Prove that dt V. dV
dt × dt 2 dt × dt3 .
= V . dV

21. Show that r = e−1 (a cos2t + b sin 2t) , where a and b are constant
2
vectors, is a solution to the differential equation ddt2r + 2 dr
dt + 5r = 0.

22. What is the greatest rate of increase of u = x2 + yz 2 at the point


(1, −1, 3)?
 √ 
Answer: 89
23. Find grad φ at the point (1, −2, 1) when φ is given by φ = 3x2 y − y 3 z 2 .
 
Answer: − 12î − 9ĵ − 16k̂

24. If φ = x3 + y 3 + z 3 − 3xyz, show that −



r · ∇φ = 3φ.
25. If u = x + y + z, v = x2 + y 2 + z 2 , w = yz + zx + xy then

a. Prove that (∇ u) · [∇v × ∇w] = 0.


3.8 Exercise 107

b. Show that ∇u, ∇v, ∇w are coplanar.


   
26. If ∇φ = (2xyz) î + x2 z ĵ + x2 y k̂, determine φ.
 
Answer: φ = x2 yz + c

−  − → − →
27. If A is a constant vector then prove that ∇ r · A = A .

Find a unit vector normal to the surface x2 + y 2 + z 2 = a2 at


28. 
√a , √a , √a .
3 3 3
 
Answer: î+√ĵ+
3

29. Find a unit vector normal to the surface x2 + y 2 − z = 1 at the point


(1, 1, 1).
 
Answer: 2î−23ĵ−k̂
 
30. Find a unit outward drawn normal to the surface x2 − 1 + y 2 +
(z + 2)2 = 9 at the point (3, 1, −4).
 
Answer: 2î+ĵ−2
3

31. The temperature at a point (x, y, z) in space is given by T (x, y, z) =


x2 + y 2 − z. A mosquito located at (1, 1, 2) desires to fly in such a
direction that it will get warm as soon as possible. In what direction
should it fly?
  
Answer: 13 2î + 2ĵ − k̂

32. What is the angle between the normals to the surface xy = z 2 at the
points (1, 9, −3) and (−2, −2, 2)?
  
Answer: cos−1 √11118

33. Show that ∇φ is a vector perpendicular to the surface φ (x, y, z) = 0.

34. Find the directional derivative of φ (x, y, z) = xy 2 + yz 3 at the point


(2, −1, 1) in the direction of the vector î + 2ĵ + 2k̂.
 
Answer: − 11
3
108 Vector Differential Calculus

35. Find the directional derivative of φ (x, y, z) = xy 2 + yz 2 at the point


(2, −1, 1) in the direction of the vector î + 2ĵ + 2k̂.
(Answer: − 3)

36. If the directional derivative of φ = axy + byz + czx at (1, 1, 1)


has maximum magnitude 4 in a direction parallel to the x-axis, find the
values of a, b, c.
(Answer: a = 2, b = −2, c = 2 )

37. Prove that the angle between the surface x2 + y 2 + z 2 = 9 and


 
x2 + y 2 − z = 3 at the point (2, −1, 2) is cos−1 3√821 .

38. Find the angle between the tangent planes to the surfaces xlogz = y 2 −
1 and x2 y = 2 − z at the point (1, 1, 1).
  
Answer: cos−1 √130

39. In what direction form (3, 1, −2) is the directional derivative of φ =


x2 y 2 z 4 maximum, and what is its magnitude?
   
1
Answer: 13 4î + 3ĵ − 12k̂ , 1

40. Is there a direction u in which the rate of change of f (x, y) = x2 −


3xy + 4y 2 at P (1, 2) equals 14? Give reasons for your answer.
 √ 
Answer : No, the maximum rate of change is 185 < 14
 
41. Given −→u = xyz î + 2x2 z − y 2 x ĵ + xz 3 k̂, − →
v = x2 î + 2yz ĵ +
(1 + 2z) k̂, f = xy + yz + z 2 then find (i) ∇ · − →v (ii) ∇ × v (iii)
∇ · (f −

u ) (iv) ∇ × (f −
→v ) at (1, 0, −1)
 
Answer: (i) 3, (ii) 0, (iii) 5, (iv) − 2ĵ

42. If ω is a constant vector and −



v =−

ω ×−

r prove that div −

v = 0.

− xî+y ĵ
43. Prove that F = x2 +y 2
is solenoidal.
→ 
− 
44. If F = y 2 − z 2 + 3yz − 2x î+(3xz + 2xy) ĵ+(3xy − 2xz + 2z) k̂,


Show that F is both solenoidal and irrotational.
3.8 Exercise 109

→ 
− 
45. If F = z 2 + 2x + 3y î + (3x + 2y + z) ĵ + (y + 2xz) k̂, Show that
F is irrotational but not solenoidal.
→ 
−     
46. Show that F = 6xy + z 3 î + 3x2 − z ĵ + 3xz 2 − y k̂


is irrotational. Find scalar φ such that F = ∇φ.

47. In each case the velocity vector − →v of a steady fluid motion is given, Find


curl v . Is the motion incompressible?
(i) −

v = z 2 ĵ , (ii) −

v = y î − xĵ , (iii) −

v = xî + y ĵ
⎛ ⎞
Answer: (i) curl − →v = −2z ĵ, incompressible
⎝ (ii) curl − →v = −2k̂, incompressible ⎠

− →

(iii) curl v = 0, div v = 2, compressible

− →

48. If the vector product of the vectors A and B be the curl of a third vector,

− →
− →

prove that A · curl = B · curl A .

− →

49. Prove that div (f curl F ) = (grad f ) · curl F .

50. If φ, ψ satisfy Laplace equation, prove that the vector (φ∇ψ − ψ∇φ) is
solenoidal.

51. Show that


#  .
(i.) ∇2 ∇. rr2 = r24
   
(ii.) ∇ × a × ∇ 1r + ∇ a . ∇ 1r = 0.

d−
→ d−

52. If u
dt =−

w ×−

u and v
dt =−

w ×−

v , then prove that

d −
(→
u ×−

v)=−

w × (−

u ×−

v ).
dt
  d−

53. If −

r = t3 î + 2t3 − 1
5t2
ĵ, then show that r̂ × r
dt = k̂.
4
Vector Integral Calculus

4.1 Introduction
Vector calculus deals with the differentiation and integration of vector func-
tions. We have learned about the derivative of a vector function, gradient,
divergence, and curl in vector differential calculus. In vector integral calculus,
we learn about line integral, surface integral, and volume integral. It plays an
important role in differential geometry and the study of partial differential
equations. It is useful in the study of rigid dynamics, fluid dynamics, heat
transfer, electromagnetism, theory of relativity, etc.

4.2 Line Integrals


b
The line integral is a simple generalization of a definite integral a f (x)dx
which is integrated from x = a (point A) to x = b (point B) along the x-axis.
In a line integral, the integration is done along a curve C in space.
→→

Let F (− r ) be a vector function defined at every point of a curve C in
Figure 4.1. If −→r is the position vector of a point P (x, y, z) on the curve C,
→→

then the line integral of F (− r ) over a curve C is defined by

Figure 4.1 Representation of a vector function defined at every point of a curve C

111
112 Vector Integral Calculus

 
− −

F (→
r ) · d−

r = F1 dx + F2 dy + F3 dz
C C


where, F = F1 î + F2 ĵ + F3 k̂ and −
→r = xî + y ĵ + k̂
If the curve C is represented by a parametric representation


r (t) = xî + y ĵ + z k̂,
then the line integral along the curve C from t = a to t = b is
  b
→ −
− → →
− → d−
− →r
F (r)·dr = F · dt
C a dt
 b
dx dy dz
= F1 + F2 + F3 dt
a dt dt dt

9If C is a closed curve, then the symbol of the line integral C is replaced by
C.


Two other types of line integrals are C F × d− →
r and C φd−

r , (where φ
is a scalar point function) which are both vectors.
Note:
1. The curve C is called the path of integration, the points −

r (a) and −

r (b)
are called initial and terminal points respectively.
2. The direction from A and B along which t increases is called positive
direction on C.

4.2.1 Circulation


If F is the velocity of a fluid particle and C is a closed curve, then the line
9 −→ → →

integral C F · d− r represents the circulation of F around the curve C.
Note:


1. If the circulation of F around every closed curve C in the region R is

− 9B−→ → →

zero, then F is irrotational, i.e., if A F · d−
r = 0, F is irrotational.

4.2.2 Work Done by a Force




If F is the force acting on a particle moving along the arc AB of the curve
B−
→ →
C, then the line integral A F · d− r represents the work done in displacing
the particle from the point A to the point B.
4.3 Path Independence of Line Integrals 113

4.3 Path Independence of Line Integrals


4.3.1 Theorem: Independent of Path
B−→ →
The necessary and sufficient condition that the line integral A F · d− r be


independent of the path is that F is the gradient of some scalar function φ.


Proof: Let F = F1 î + F2 ĵ + F3 k̂ be the gradient of a scalar function φ (i.e.,

− →

F is conservative) i.e., F = ∇φ in components
∂φ ∂φ ∂φ
F1 = , F2 = , F3 =
∂x ∂y ∂z
where φ is a scalar potential, then the line integral along the curve C from the
point A to B is
  B

− − →
F ·dr = ∇φ · d− →
r
C A
 B
∂φ ∂φ ∂φ
= dx + dy + dz
A ∂x ∂y ∂z
 B
= dφ
A
= φ (B) − φ(A)
where φ (A) and φ(B) are the values of φ at A and B respectively, That
means the line integral depends only on the start and end values of the scalar
potential, not on the path of the curve. Hence the condition is necessary. We
show below that the condition is sufficient.
B− → →
Let A F · d− r depend on the end values A and B and not on the path of
integration.
 B
→ −

∴ F · d→r =φ (B) − φ (A) = [φ]B A
A
→ −
− ∂φ ∂φ ∂φ
∴ F · d→ r = dφ = dx + dy + dz = ∇φ · d− →r
∂x ∂y ∂z


Since this is true for all the curves between A and B, F = ∇φ.


In Mechanics, F is called a conservative field and φ is the scalar potential,

− →

and in Potential theory, if φ is potential and F is a force then F = grad φ.


Thus, the line integral is independent of the path in C iff F is the gradient of
potential in C.
114 Vector Integral Calculus


Corollary 4.3.1.1: If F = ∇φ, we have


curl F = curl grad φ = ∇ × ∇φ = 0
→ →

Hence, the necessary and sufficient condition that C F · d− r be independent

− →

of the path is the curl F vanishes identically and hence F is irrotational.
→ →

Corollary 4.3.1.2: If C F · d− r is independent of the path of integration,
9 −→ − →
then F · d r along any closed path is zero.
C


Corollary 4.3.1.3: Let F = îP + ĵQ, C (P dx + Qdy) is independent of
∂Q
its path, if ∂P
∂y = ∂x .
Note:

1. As shown in Figure 4.2,




if F is conservative and curve C is closed, then
:
→ −

F · d→r = φ (A) − φ (A) = 0.
C

2. The work done in moving a particle from points A to B under a


conservative force field is

Work done = φ (B) − φ(A)

Figure 4.2 Representation of a closed curve C

→ →

Illustration 4.1: Evaluate C F · d−r along the parabola y 2 = x between the


points (0, 0) and (1, 1) where F = x2 î + xy ĵ.
4.3 Path Independence of Line Integrals 115

Solution:
Let −

r = xî + y ĵ
∴ d−

r = îdx + ĵdy


Here, F = x2 î + xy ĵ
→ →  2
−   
∴ F · d−
r = x î + xy ĵ · îdx + ĵdy
= x2 dx + xydy (4.1)
In the above expression, we can see that
→ −

F · d→ r contains both the variables x and y.
To apply line integral in Equation (4.1), we require the expression either in
terms of x or in terms of y variable. As we have discussed that the line integral
is independent of the path, we can consider the expression in Equation (4.1)
in terms of any one variable. In this illustration, we represent the Equation
(4.1) in terms of y variable and as shown in Figure 4.3, the path of integration
is the parabola
x = y 2 ⇒ dx = 2ydy
Substituting in the Equation (4.1) and integrating between the limits y =
0 to y = 1, we get
  1
→ −
− →
F ·dr = (y 4 · 2ydy + y 2 · ydy)
C 0
 1
 5 
= 2y + y 3 dy
0
 6 
4 1
 y y 1 1 7
= 2 +  = + =
6 4 0 3 4 12

Figure 4.3 Representation of parabola x = y 2


116 Vector Integral Calculus
→ →
− →

Illustration 4.2: Prove that C F · d− r = 3π, where F = z î + xĵ + y k̂ and
C is the arc of the curve −

r = cost î + sint ĵ + tk̂ from t = 0 to t = 2π.
Solution: Here, we have − →r = cost î + sint ĵ + tk̂

∴ x = cost, y = sint , z = t
∴ dx = −sint dt, dy = cost dt, dz = dt
→ → 
−   
∴ F · d−
r = z î + xĵ + y k̂ · îdx + ĵdy + k̂dz
= zdx + xdy + ydz
= t (−sint ) dt + cost · cost dt + sint dt
 
= −tsint + cos2 t + sint dt

Now, the path of integration is the arc of the curve −



r = cost î + sint ĵ + tk̂
from t = 0 to t = 2π.

→ −

∴ F · d→ r
C
 2π
 
= −tsint + cos2 t + sint dt
0
 2π
2π (1 + cos2t )
= −|t (−cost ) − (−sint )|0 + dt + |−cost |2π
0
0 2
 2π
t sin2t 
= − (−2π) +  + − (cos2π − cos0 )
2 4 0

= 2π + = 3π
2


Illustration 4.3 If F = (2x − y + 2z) î+(x + y − z) ĵ +(3x − 2y − 5z) k̂,


calculate the circulation of F along the circle in the xy-plane of 2 units radius
and center at the origin.
Solution:
The circulation is given by :
− −

F · d→
r
C

Let →

r = xî + y ĵ + z k̂ then

d−

r = îdx + ĵdy + k̂dz
4.3 Path Independence of Line Integrals 117

→ →

Now, F · d−
r
 
= (2x − y + 2z)î + (x + y − z)ĵ + (3x − 2y − 5z)k̂ ·(îdx + ĵdy + k̂dz)
= (2x − y + 2z) dx + (x + y − z) dy + (3x − 2y − 5z) dz

Next, the path of integration is the circle in xy-plane of radius 2 units and
center at the origin, i.e., x2 + y 2 = 4 and in xy-plane z = 0. Therefore, the
parametric equation of the circle is

x = 2cosθ , y = 2 sinθ

dx = −2sinθ dθ , dy = 2cosθ dθ
For the complete circle, θ varies from 0 to 2π.
→ →

Substituting in F · d−
r and integrating between the limits θ = 0 to θ = 2π,
 2π
∴ Circulation = [(2 · 2 cos θ − 2 sin θ) (−2 sin θdθ)
0
+ (2 cos θ + 2 sin θ) (2 cos θ dθ)]
 2π
 
=4 −2cosθ sinθ + sin2 θ + cos2 θ + cosθ sinθ dθ
0
 2π  2π
sin2θ  cos2θ 
=4 1− dθ = 4θ +  = 8π
0 2 4 0

→ →
− → 
− 
Illustration 4.4 Evaluate C F · d− r where F = x2 + y 2 î − 2xy ĵ and C
is the rectangle in the xy-plane bounded by y = 0, x = a, y = b, x = 0.
Solution:

Figure 4.4 Representation of the rectangle in xy-plane bounded by lines


118 Vector Integral Calculus

Consider, −

r = xî + y ĵ
∴ d− →r = îdx + ĵdy
→ →
−    
Now, F · d−
r = x2 + y 2 î − 2xy ĵ · (îdx + ĵdy)
 
= x2 + y 2 dx − 2xydy
Next, the path of integration is the rectangle OABD (See Figure 4.4)
bounded by the four lines y = 0, x = a, y = b, x = 0.
    
→ −
− → → −
− → → −
− → → −
− → → −

∴ F ·dr = F ·dr + F ·dr + F ·dr + F · d→r
C OA AB BD DO
(4.2)

(i) Along OA : y = 0 ⇒ dy = 0 and x varies from 0 to a.


   3 a
− −
→ a x  a3
F · d→
r = x dx =   =
2
OA 0 3 0 3
(ii) Along AB : x = a ⇒ dx = 0 and y varies from 0 to b.
  b
→ −
− →  b
F ·dr = −2aydy = −ay 2 0 = −ab2
AB 0

(iii) Along BD : y = b ⇒ dy = 0 and x varies from a to 0.


  0  3 0
→ →
− − x  a3
F ·dr = 2 2  2 
(x + y )dx =  + b x = − + b2 a
BD a 3 a 3

(iv) Along DO : x = 0 ⇒ dx = 0 and y varies from b to 0.


  0
→ −
− →
F ·dr = 0dy = 0
DO b

Substituting in the Equation (4.2), we get



→ −
− a3 a3
F · d→
r = − ab2 − − b2 a = −2ab2
C 3 3
→ →
− →
−  
Illustration 4.5 Evaluate C F · d− r where F = 3x2 + 6y î − 14yz ĵ +
20xz 2 k̂ and C is the straight line joining the points (0, 0, 0) to (1, 1, 1).
4.3 Path Independence of Line Integrals 119

Solution:
Consider, −

r = xî + y ĵ + z k̂ then d−

r = îdx + ĵdy + k̂dz
→  2
− 
Here, we have F = 3x + 6y î − 14yz ĵ + 20xz 2 k̂
→ →  2
−  
∴ F · d−r = 3x + 6y î − 14yz ĵ + 20xz 2 k̂ · (îdx + ĵdy + k̂dz)

Next, the path of integration is the straight line joining the points
A (0, 0, 0) to B (1, 1, 1). The equation of the line AB is
x−0 y−0 z−0
= =
0−1 0−1 0−1
∴x=y=z
∴ dx = dy = dz

− −
Substituting in F ·d→
r and integrating between the limits x = 0 to x = 1,
we get
  1
→ →
− − # 2  .
F ·dr = 3x + 6x dx − 14x2 dx + 20x3 dx
C 0
 1
= (20x3 − 11x2 + 6x)dx
0
 1
 x4 x3 x2  13

= 20 − 11 + 6  =
4 3 2 0 3


−  
Illustration 4.6 If F = 2xyz î + x2 z + 2y ĵ + x2 y k̂, then


(i) If F is conservative, then find its scalar potential φ.

(ii) Find the work done in moving a particle under this force field
from (0, 1, 1) to (1, 2, 0).

Solution:

− →

(i) Since F is conservative, we have F = ∇φ
  ∂φ ∂φ ∂φ
∴ 2xyz î + x2 z + 2y ĵ + x2 y k̂ = î + ĵ + k̂
∂x ∂y ∂z
120 Vector Integral Calculus

Now, comparing the coefficients of î, ĵ and k̂ on both sides, we have


∂φ ∂φ ∂φ
= 2xyz, = x2 z + 2y, = x2 y
∂x ∂y ∂z
Also,
∂φ ∂φ ∂φ
dφ = dx + dy + dz
∂x ∂y ∂z
 
= 2xyzdx + x2 z + 2y dy + x2 ydz
Integrating both sides,
  
dφ = y, z 2xyzdx + x, z (x2 z + 2y)dy
constant constant

+ x, y (x2 y)dz
constant

Considering only those terms on the right-hand side of the integral which
have not appeared in the previous integral, i.e., omitting the x2 yz terms
in second and third integral, we get
φ = x2 yz + y 2 + c
where c is the integrating constant.


(ii) F is conservation and hence the work done is independent of the path.

→ −

∴ Work done = F · d→
r
C
 (1,2,0)
(1,2,0)
= dφ = |φ|(0,1,1)
(0,1,1)
 (1,2,0)
= x2 yz + y 2 + c (0,1,1)
=3


−    
Illustration 4.7 If F = (x2 − yz)î + y 2 − zx ĵ + z 2 − xy k̂, then


(i) If F is conservative, then find its scalar potential φ.
4.3 Path Independence of Line Integrals 121

(ii) Find the work done in moving a particle under this force field
from (1, 1, 0) to (2, 0, 1).

Solution:

− →

(i) Since F is conservative, we have F = ∇φ
    ∂φ ∂φ ∂φ
∴ (x2 − yz)î + y 2 − zx ĵ + z 2 − xy k̂ = î + ĵ + k̂
∂x ∂y ∂z

Now, comparing the coefficients of î, ĵ and k̂ on both sides, we have


∂φ ∂φ  2  ∂φ  2 
= (x2 − yz), = y − zx , = z − xy
∂x ∂y ∂z
Also,
∂φ ∂φ ∂φ
dφ = dx + dy + dz
∂x ∂y ∂z
   
= (x2 − yz)dx + y 2 − zx dy + z 2 − xy dz

Integrating both sides,


  
dφ = y, z 2xyzdx + x, z (x2 z + 2y)dy
constant constant

+ x, y (x2 y)dz
constant

Considering only those terms on the right-hand side of the integral which
have not appeared in the previous integral, i.e., omitting the xyz terms
in second and third integral, we get

x3 y3 z3
φ= − xyz + + +c
3 3 3
where c is the integrating constant.


(ii) F is conservation and hence the work done is independent of the path.
122 Vector Integral Calculus

− −

∴ Work done = F · d→
r
C
 (2,0,1)
(2,0,1)
= dφ = |φ|(1,1,0)
(1,1,0)
 3 (2,0,1)
x y3 z3 

=  − xyz + + + c
3 3 3 (1,1,0)
7
=
3

4.4 Surface Integrals


The surface integral over a curved surface S is the generalization of a double
integral over a plane region R given in Figure 4.5.


Let F = F1 î + F2 ĵ + F3 k̂ be a continuous vector point function defined
over a two-sided surface S. Divide S into a finite number of subsurfaces
S1 , S2 , S3 , . . . , Sm with surface areas δS1 , δS2 , δS3 , . . . , δSm . Let δSr be
the surface area of Sr and n̂r be the limit vector at some point Pr (in Sr ) in
the direction of the outward normal to Sr . If we increase the number of sub-
surfaces, then the surface area δSr of each sub-surface will decrease. Thus,
as m → ∞, δSr → 0 then,
/
m 

− →

lim F (Pr ) · n̂r δSr = F · n̂ dS
m→∞ S
r=1

Figure 4.5 Representation of curved surface S and a plane region R


4.4 Surface Integrals 123



This is called the surface integral of F over the surface S.
The surface integral can also be written as

→ −
− → →

F · d S , Where d S = n̂dS
S
∇φ
If the equation of surface S is φ (x, y, z) = 0, then n̂ = |∇φ| .

4.4.1 Flux


If F represents the velocity of the fluid at any point P on a closed surface S,

− →

then surface integral S F · n̂ dS represents the flux of F over S, i.e., the
volume of the fluid flowing out from S per unit time.
Note:

→ →

If S F · n̂ dS = 0, then F is called a solenoidal vector point function.

4.4.2 Evaluation of Surface Integral


A surface integral is evaluated by expressing it as a double integral over
the region R. The region R is the orthogonal projection of S on one of the
coordinate planes (xy, yz or zx) . Let R be the orthogonal projection of S
on the xy−plane and cosα, cosβ, cosγ are the direction cosines of n̂.
Then
n̂ = cosα î + cosβ ĵ + cosγ k̂
dxdy = Projection of dS on xy − plane = dScosγ
dxdy dxdy
dS = =  

cosγ n̂ · k̂ 
Hence,  

→ −
→ dxdy
F · n̂ dS = F · n̂  

S R n̂ · k̂ 
Similarly, taking projection on yz and zx-plane,
 

− →
− dydz
F · n̂ dS = F · n̂  

S R n̂ · î
and  

→ −
→ dzdx
F · n̂ dS = F · n̂  

S R n̂ · ĵ 
124 Vector Integral Calculus

4.4.2.1 Component form of Surface Integral


   


F · n̂ dS = (F1 î + F2 ĵ + F3 k̂)· cosα î + cosβ ĵ + cosγ k̂ dS
S
 S
= F1 dScosα + F2 dScosβ + F3 dScosγ
 S
= F1 dydz + F2 dzdx + F3 dxdy
S


− −

Illustration 4.8: Evaluate S F · n̂ dS, where F = 18z î − 12ĵ + 3y k̂ and
S is the part of the plane 2x + 3y + 6z = 12 in the first octant.
Solution:

(i) The given surface is the plane 2x + 3y + 6z = 12 in the first octant (See
Figure 4.6(a)).
Let φ = 2x + 3y + 6z

∇φ 2î + 3ĵ + 6k̂ 2î + 3ĵ + 6k̂


∴ n̂ = = √ =
|∇φ| 4 + 9 + 36 7

(ii) Let R be the projection of the plane 2x + 3y + 4z = 12 (in the first


octant) on the xy-plane, which is a triangle OAB bounded by the lines
y = 0, x = 0 and 2x + 3y = 12 (See Figure 4.6(b)).
(iii) Next, we find
dxdy 7
dS =   = dxdy

n̂ · k̂  6

Figure 4.6 Representation of the projection of the plane in the first octant
4.4 Surface Integrals 125

12−2x
(iv) Along the vertical strip P Q, y varies from 0 to 3 , and in the region
R, x varies from 0 to 6.



∴ F · n̂ dS

S

  2î + 3ĵ + 6k̂ 7
= 18z î − 12ĵ + 3y k̂ · dxdy
R 7 6

1
= (36z − 36 + 18y) dxdy
6
 ! R
"
12 − 2x − 3y
=3 2 − 2 + y dxdy
R 6
 6  12−2x−3y
6
= (6 − 2x)dydx
0 0
 6 12−2x
=2 (3 − x) |y|0 3 dx
0
 6
(12 − 2x)
=2 (3 − x) dx
0 3

4 6 2
= (x − 9x + 18)dx
3 0
 6
4  x3 9x2 
=  − + 18x
3 3 2 0
4
= (72 − 162 + 108) = 24
3

Illustration 4.9: Evaluate S (yzdydz + xzdzdx + xydxdy) over the sur-


face of the sphere x2 + y 2 + z 2 = 1 in the positive octant.
Solution:


(i) S F · n̂ dS = yzdydz + xzdzdx + xydxdy


∴ F = yz î + xz ĵ + xy k̂

(ii) The given surface is the sphere x2 + y 2 + z 2 = 1.


Let φ = x2 + y 2 + z 2
∇φ 2xî + 2y ĵ + 2z k̂
∴ n̂ = = = xî + y ĵ + z k̂
|∇φ| 4x2 + 4y 2 + 4z 2
126 Vector Integral Calculus

Figure 4.7 Representation of the positive octant of the sphere

(∵ x2 + y 2 + z 2 = 1)

(iii) Let R be the projection of the sphere x2 + y 2 + z 2 = 1 (in the positive


octant) on the xy−plane (z = 0), which is the part of the circle x2 +y 2 =
1 in the first quadrant (See Figure 4.7).
dxdy dxdy
(iv) dS = =
|n̂·k̂| z



(v) S F · n̂ dS

= yzdydz + xzdzdx + xydxdy
 S
   dxdy
= yz î + xz ĵ + xy k̂ · xî + y ĵ + z k̂
z
 R
dxdy
= (3xyz)
z
R
=3 xydxdy
R

Substituting x = rcosθ , y = rsinθ , the equation of the circle x2 +y 2 =


1 reduces to r = 1 and dxdy = rdrdθ.
Along the radius vector OP , r varies from 0 to 1, and in the first quadrant
of the circle, θ varies from 0 to π2 .

∴ yzdydz + xzdzdx + xydxdy
S
4.4 Surface Integrals 127

 π  1
2
=3 rcosθ · rsinθ · r drdθ
0 0
 π  1
sin2θ2
=3 dθ · r3 dr
0 2 0
  π  1
3  −cos2θ  2  r4  3 3
=   ·  = (−cosπ + cos0 ) =
2 2 0 4 0 16 8



Illustration 4.10: Find the flux of F = î − ĵ + xyz k̂ through the circular
region S obtained by cutting the sphere x2 +y 2 +z 2 = a2 with a plane y = x.
Solution:
We know that 


Flux = F · n̂ dS
S
(i) Here, the surface S (See Figure 4.8) is the intersection of the sphere
x2 +y 2 +z 2 = a2 with a plane y = x, which is an ellipse 2x2 +z 2 = a2 .
(ii) Normal to the ellipse 2x2 + z 2 = a2 is also normal to the plane y = x.
Let φ = x − y
∇φ î − ĵ
n̂ = = √
|∇φ| 2

Figure 4.8 Representation of the sphere


128 Vector Integral Calculus

(iii) Let R be the projection of the surface S on the xz-plane, which is an


ellipse 2x2 + z 2 = a2

(iv) dS = dxdz = 2 dxdz
|n̂·ĵ |
→ →
−    √
(v) S F · − n dS = R î − ĵ + xyz k̂ · î−√ ĵ
2
2 dxdz

= 2dxdz
R

Substituting x = √a rcosθ
2
, z = arsinθ , the equation of the ellipse 2x2 +
2
z 2 = a2 reduces to r = 1 and dxdz = √ a
2
rdr dθ .
Along the radius vector OP , r varies from 0 to 1, and for a complete
ellipse, θ varies from 0 to 2π.
  2π  1 2
→ −
− a
F ·→n dS = 2 √ rdr dθ
S 0 0 2
 2 1
2
2a r  √ 2 1
= √   |θ|2π 0 = 2 a · · 2π
2 2 0 2

= 2 πa2

Aliter,
 
− −

F ·→
n dS = 2 dxdz
S
⎡ R

⎢ x2 y2 ⎥
= 2 ⎣Area of the ellipse  2 + 2 = 1⎦
√a
a
2
a
= 2 · π√ · a
2
√ 2
= 2 πa


Hence, flux = 2 πa2 .
4.5 Volume Integrals 129

4.5 Volume Integrals


If V be a region in space bounded by a closed surface S, then the volume

− →

integral of a vector point function F is V F dV .

4.5.1 Component Form of Volume Integral




If F = F1 î + F2 ĵ + F3 k̂ then
   


F dV = F1 î + F2 ĵ + F3 k̂ dxdydz
V
V  
= î F1 dxdydz + ĵ F2 dxdydz + k̂ F3 dxdydz

Another type of volume integral is V φ dV , where φ is a scalar function.



− −

Illustration 4.11: Evaluate V F dV where F = xî + y ĵ + 2z k̂ and V is
the volume enclosed by the planes x = 0, y = a, z = b2 and the surface
z = x2 .
Solution:
(i) V is the volume of the cylinder in positive octant with the base as OAB
and bounded between the planes y = 0 and y = a. y varies from 0 to a
(See Figure 4.9).
(ii) Along the vertical strip P Q, z varies from x2 to b2 , and in the region
OAB, x varies from 0 to b.
  b  b2  a  


F dV = xî + y ĵ + 2z k̂ dxdydz
V x=0 z=x2 y=0

Figure 4.9 Representation of the cylinder in positive octant


130 Vector Integral Calculus
 b b2
 2 a
y 
= xî|y|a0 + ĵ   + 2z k̂|y|a0 dzdx
0 x2 2 0
 b b2
a2
= îxa + ĵ + k̂2za dzdx
0 x2 2
 b 2 a2 2  b 2
= îxa|z|bx2 + ĵ |z|bx2 + k̂az 2 x2 dx
0 2
 b
a2 2
= îxa(b2 − x2 ) + ĵ (b − x2 ) + k̂a(b4 − x4 ) dx
0 2
 b
 b2 x2 x4 a2 x3 x5 
= îa − + ĵ b2 x − + k̂a b4 x −
2 4 2 3 5 0
b4 b4 a2 b3 b5
= îa − + ĵ b3 − + k̂a b5 −
2 4 2 3 5
3
b4 a2 b 4ab5
= a î + ĵ + k̂.
4 3 5
 −
→ → 
− 
Illustration 4.12: Evaluate V ∇ × F dV , where F = 2x2 − 3z î −
2xy ĵ − 4xk̂ and V is the closed region bounded by the planes x = 0, y = 0,
z = 0 and 2x + 2y + z = 4.
Solution:

(i)
 
 
 î ĵ k̂ 
→ 
− 

∇× F = ∂
∂x

∂y

∂z 
 
 2x2 − 3z −2xy −4x 
= (0 − 0) î − (−4 + 3) ĵ + (−2y − 0) k̂
= ĵ − 2y k̂

(ii) Along the elementary volume P Q, z varies from 0 to 4−2x−2y. Along


the vertical strip P  Q , y varies from 0 to 2−x, and in the region, x varies
from 0 to 2.
   2  2−x  4−2x−2y 
→
− 
∇ × F dV = ĵ − 2y k̂ dxdydz
V 0 0 0
4.6 Exercise 131

 2  2−x  
= ĵ − 2y k̂ |z|4−2x−2y
0 dydx
0 0
 2  2−x  
= ĵ − 2y k̂ (4 − 2x − 2y) dydx
0 0
 2  2−x  
= (4 − 2x − 2y) ĵ − 2 (4 − 2x) y k̂ + 4y 2 k̂ dydx
0 0
2 $ 0
 2 2−x 1
= (4 − 2x) |y|2−x − y  ĵ
0 0
0
&  3 2−x ' %
 2 2−x y 
− 2 (2 − x) y 0 − 4   k̂ dx
3 0
 2! "
2 2 3 8 8 8
= (2 − x) ĵ − (2 − x) k̂ dx = ĵ − k̂ = (ĵ − k̂)
0 3 3 3 3

4.6 Exercise

− →
1. Evaluate C F · d− r along the curve x2 + y 2 = 1, z = 1 in the positive


direction from (0, 1, 1) to (1, 0, 1), where F = (yz + 2x) î + xz ĵ +
(xy + 2z) k̂.
(Answer: 1)
→ →
− →

2. Evaluate C F · d− r over the circular path x2 + y 2 = a2 where F =
siny î + x (1 + cosy ) ĵ.
(Answer : πa2 )


3. Find the work done in moving a particle in the force field F = 3x2 î +
(2xz − y) ĵ + z k̂ along the curve x2 = 4y and 3x3 = 8z from x = 0 to
x = 2.
(Answer: 16)

4. Find the work done in moving a particle from A(1, 0, 1) to B(2, 1, 2)




along the straight line AB in the force field F = x2 î + (x − y) ĵ +
(y + z) k̂.
 
Answer : 16
3
132 Vector Integral Calculus

5. Find the work done in moving a particle along the straight-line segments
joining the points (0, 0, 0) to (1, 0, 0) , then to (1, 1, 0) , and finally to
→ 
− 
(1, 1, 1) under the force field F = 3x2 + 6y î − 14yz ĵ + 20xz 2 k̂.
 
Answer : 23
3


6. Find the work done by the force F = xî − z ĵ + 2y k̂ in displacing the
particle along the triangle OAB, where

OA : 0 ≤ x ≤ 1, y = x, z = 0
AB : 0 ≤ z ≤ 1, x = 1, y = 1
BO : 0 ≤ x ≤ 1, y=z=x
 3

Answer : 2

−  
7. Find the work done by the force F = 16y î + 3x2 + 2 ĵ in moving
a particle once round the right half of the ellipse x2 + a2 y 2 = a2 from
(0, 1) to (0, −1).
 
Answer : 8aπ − 4a2 − 4

− → →

8. Evaluate C F · d− r , where F = 2xî + 4y ĵ − 3z k̂ and C is the curve


r = cost î + sint ĵ + t k̂ from t = 0 to t = π.
 2

Answer : −3π
2



9. Find the circulation of F = (x − 3y) î + (y − 2x) ĵ around the ellipse
in the xy− plane with the origin as a center and 2 and 3 as semi-major
and semi-minor axes respectively.
(Answer : 6π)


10. Find the circulation of F = y î + z ĵ + xk̂ around the curve x2 + y 2 =
1, z = 0.
(Answer : − π)

−  
11. If F = 2xy + z 3 î + x2 ĵ + 3xz 2 k̂ is conservative then (i) find its
scalar potential φ, (ii) find the work done in moving a particle under this
force field from (1, −2, 1) to (3, 1, 4).
 
Answer : (i) φ = x2 y + xz 3 + c, (ii) 202
4.6 Exercise 133



12. If F = 3x2 y î + (x3 − 2yz 2 )ĵ + (3z 2 − 2y 2 z)k̂ is conservative then (i)
find its scalar potential φ, (ii) find the work done in moving a particle
under this force field from (2, 1, 1) to (2, 0, 1).
 
Answer : (i) φ = x3 y + z 3 − y 2 z 2 + c, (ii) − 7


13. If F = 2xyez î + x2 ez ĵ + x2 yez k̂ is conservative then (i) find its scalar
potential φ, (ii) find the work done in moving a particle under this force
field from (0, 0, 0) to (1, 1, 1).
 
Answer : (i) φ = x2 yez + c, (ii) e

− →

14. Evaluate S F · n̂dS where F = 3y î + 2z ĵ + x2 yz k̂ and S is the
surface y 2 = 5x in the positive octant bounded by the planes x = 3 and
z = 4.
(Answer : − 42)

− →

15. Evaluate S F · n̂dS where F = (x + y 2 )î − 2xĵ + 2yz k̂ and S is the
surface 2x + y + 2z = 6 in the first octant.
(Answer : 81)

− →

16. Evaluate S ∇ × F · n̂dS where F = y 2 î + y ĵ − xz k̂ and S is the
upper half of the sphere x2 + y 2 + z 2 = a2 .
(Answer : 0)


17. Find the flux of the vector field F through the portion√ of the sphere

x2 + y 2 + z 2 = 36 lying between the planes z = 11 and z = 20


where F = xî + y ĵ + z k̂.
 √ √ 
Answer : 72π 20 − 11

− 
18. Find the flux of the vector field F = xî +
 y ĵ + x2 + y 2 − 1 k̂ through
the outer side of the hyperboloid
√ z = x + y 2 − 1 bounded by the
2

planes z = 0 and z = 3.
 √ 
Answer : 2 3π


19. Find the flux of the vector field F = 2y î−z ĵ +x2 k̂ across the surface of
the parabolic cylinder y 2 = 8x in the first octant bounded by the planes
y = 4 and z = 6.
(Answer : 132)
134 Vector Integral Calculus
 →
− →

20. Evaluate V ∇ · F dV where F = 2x2 y î − y 2 ĵ + 4xz 2 k̂ and V is
the region in the first octant bounded by the cylinder y 2 + z 2 = 9 and
the plane z = 2.
(Answer : 180)
 →
−  →

21. Evaluate V ∇ · F dV where F = 2xz î − xĵ + y 2 k̂ and V is the
region bounded by the surfaces x = 0, y = 0, y = 6, z = x2 , z = 4.
 
Answer : 128î − 24ĵ + 384k̂

22. Evaluate V f dV where f = 45x2 y and V is the region bounded by


the planes 4x + 2y + z = 8, x = 0, y = 0, z = 0.
(Answer : 128)
 →
− →

23. Evaluate V ∇ × F dV , where F = (x + 2y) î−3z ĵ+xk̂ and V is
the closed region in the first octant bounded by the plane 2x+2y+z = 4.
  
Answer : 83 3î − ĵ + 2k̂
5
Green’s Theorem, Stokes’ Theorem,
and Gauss’ Theorem

5.1 Green’s Theorem (in the Plane)


Double integrals over a plane region could also be transformed into line
integrals over the boundary of the region and conversely. This is of practical
interest because it’s going to help to form the evaluation of an integral easier.
The transformation can be computed using a theorem known as Green’s
theorem.
Statement: If M (x, y), N (x, y) and their partial derivatives ∂M ∂N
∂y , ∂x are
continuous in some region R of xy-plane bounded by a closed curve C, then
: 
∂N ∂M
(M dx + N dy) = − dxdy
C R ∂x ∂y

Proof: Let R be the bounded region by the curve C as shown in Figure 5.1.
Let the curve C be divided into two parts, the curves EAB and BDE.

Figure 5.1 Represents the region R bounded by the curve C

135
136 Green’s Theorem, Stokes’ Theorem, and Gauss’ Theorem

Let the equations of the curves EAB and BDE are x = f1 (y), x = f2 (y)
respectively and are bounded between the lines y = c and y = d.
Consider,
  d $ f2 (y) %
∂N ∂N
dxdy = dx dy
R ∂x c f1 (y) ∂x
 d
f (y)
= |N (x, y)|f21 (y) dy
c
 d
= [N (f2 , y) − N (f1 , y)] dy
c
 d  d
= N (f2 , y) dy + N (f1 , y) dy
c

c

= N (x, y) dy + N (x, y) dy
BDE EAB
:
= N (x, y)dy
C
 :
∂N
∴ dxdy = N (x, y)dy (5.1)
R ∂x C

Similarly, let the curve C be divided into two parts, the curves ABD and
DEA. Let the equation of the curves ABD and DEA are y = g1 (x) , y =
g2 (x) respectively and are bounded between the lines x = a and x = b.
Now, consider
  b $ g2 (x) %
∂M ∂M
dxdy = dy dx
R ∂y a g1 (x) ∂y
 b
g (x)
= |M (x, y)|g21 (x) dx
a
 b
= [M (x, g 2 ) − M (x, g 1 )] dx
a
 b  b
=− M (x, g 2 ) dx − M (x, g 1 ) dx
!
a a
 "
=− M (x, y) dx + M (x, y) dx
DEA ABD
5.1 Green’s Theorem (in the Plane) 137
:
=− M (x, y)dx
 C
:
∂M
∴ − dxdy = M (x, y) dx (5.2)
R ∂y C

Adding Equations (5.1) and (5.2), we get


: 
∂N ∂M
(M (x, y) dx + N (x, y) dy) = − dxdy
C R ∂x ∂y
or : 
∂N ∂M
(M dx + N dy) = − dxdy.
C R ∂x ∂y

− → →

Note: Let F = M î + N ĵ and −

r = xî + y ĵ, then F · d−
r = M dx + N dy.
Also,  
 î ĵ k̂ 

−  ∂ → ∂N
− ∂M
curl F =  ∂x ∂y ∂ ∂ 
∂z 
= k −
 M N O  ∂x ∂y


The component of the curl F which is normal to a region R in xy-plane is
 → −
− → ∂N ∂M
∇× F · k = −
∂x ∂y
Hence, the vector form of Green’s theorem is given as
:  
→ −
− → → −
− →
F ·dr = ∇ × F · k dxdy
C R


where, F = M î + N ĵ, −

r = xî + y ĵ, k̂ is the unit vector along z-axis.

5.1.1 Area of the Plane Region


Let A be the area of the plane region R bounded by a closed curve C.
Let M = −y, N = x ⇒ ∂M ∂y = −1, ∂x = 1
∂N

Using Green’s theorem,


:  
(−ydx + xdy) = (1 + 1)dxdy = 2 dxdy = 2A
C R R
:
1
∴A= (xdy − ydx)
2 C
138 Green’s Theorem, Stokes’ Theorem, and Gauss’ Theorem

Note: In polar coordinates,

x = rcosθ , y = rsinθ

dx = cosθ dr − rsinθ dθ, dy = sinθ dr + rcosθ dθ


:
1
∴A= [rcosθ (sinθ dr + rcosθ dθ) − rsinθ (cosθ dr − rsinθ dθ)]
2 C
:
1
∴A= r2 dθ
2 C

Illustration 5.1: Using Green’s theorem, evaluate


:
 2 
3x − 8y 2 dx + (4y − 6xy)dy
C

where C is the boundary of the region bounded by x = y 2 and y = x2 .


Solution:
Here, C is the boundary of the region bounded by x = y 2 and y = x2 (See
Figure 5.2). Therefore, solving both the parabolas, we get

∴ x4 − x = 0
 
∴ x x3 − 1 = 0
∴ x = 0 or x3 = 1

Therefore, for x = 0 ⇒ y = 0 and for x = 1 ⇒ y = 1. So, both the


parabolas are intersecting at (0, 0) and (1,1).

Figure 5.2 Represents the region R bounded two parabolas


5.1 Green’s Theorem (in the Plane) 139

Next,

M = 3x2 − 8y 2 , N = 4y − 6xy
∂M ∂N
∴ = −16y, = −6y
∂y ∂x
: 
∂N ∂M
∴ M dx + N dy = − dxdy
C R ∂x ∂y
 1  √x
= 10y dxdy
0 x2
 1
  √x
= 5 y 2 x2 dx
0
 1
 
=5 x − x4 dx
0
1
x2 x5 1 1 3
=5 − =5 − =
2 5 0 2 5 2

Illustration 5.2: Applying Green’s theorem, evaluate


:
[(y − sinx ) dx + cosx dy]
C

where C is the plane triangle enclosed by the lines y = 0, x = π


2 and y = π2 x
(See Figure 5.3).
Solution:

Figure 5.3 Represents the plane triangle enclosed by given lines


140 Green’s Theorem, Stokes’ Theorem, and Gauss’ Theorem

Here, M = y − sinx , N = cosx


∴ ∂M
∂y = 1, ∂x = −sinx and we know that
∂N

: 
∂N ∂M
M dx + N dy = − dxdy
∂x ∂y
C
 R

= (−sinx − 1 ) dydx

R
π
$ 2x %
2 π
= (−sinx − 1)dy dx
x=0 y=0
 π
2x
2
= [−ysinx − y]0π dx
0
 π

2 2π
= sinx −
− dx
0 x x
π
! 2 " π2
2 x 2 π
= − [−xcosx + sinx ]0 −
2
=− −
π π 0 π 4



Illustration 5.3: Verify Green’s theorem for the function F = (x + y) î +
2xy ĵ and C is the rectangle in the xy-plane bounded by x = 0, y = 0, x = a,
y = b (See Figure 5.4).
Solution:

Figure 5.4 Represents the rectangle in the xy-plane bounded by lines


5.1 Green’s Theorem (in the Plane) 141

We know that
: :
− −

M dx + N dy = F · d→
r
C :C
= (x + y) dx + 2xy dy
C

Along OA, y = 0 ⇒ dy = 0
  a
a2
∴ M dx + N dy = xdx = (i)
OA 0 2
Along AB, x = a ⇒ dx = 0
  b
2ab2
∴ M dx + N dy = 2ay dy = = ab2 (ii)
AB 0 2
Along BC, y = b ⇒ dy = 0
  0
a2
∴ M dx + N dy = (x + b)dx = − − ab (iii)
AB a 2
Along CO, x = 0 ⇒ dx = 0.
  0
∴ M dx + N dy = 0 dy = 0 (iv)
CO b

Adding (i), (ii), (iii), and (iv), we get


:
M dx + N dy = ab2 − ab (v)
C

Now,
  a b
∂N ∂M
− dxdy = (2y − 1) dxdy
R ∂x ∂y 0 0

2y 2 a b
= − y dx
0 2 0
 a
 2 
= b − b dx
0

 2  a
= b −b dx
 2  0 a
= b − b (x)0 = ab2 − ab (vi)
142 Green’s Theorem, Stokes’ Theorem, and Gauss’ Theorem

From (v) and (vi) we see that


: 
∂N ∂M
M dx + N dy = − dxdy
C R ∂x ∂y
Hence, Green’s theorem is verified.
Illustration 5.4: Use Green’s theorem to evaluate the integral
:
 2 
y dx + x2 dy
C

where, C: The triangle bounded by x = 0, x + y = 1, y = 0 (See Figure 5.5).


Solution:
Here, M = y 2 and N = x2 then
∂M ∂N
= 2y, = 2x
∂y ∂x
From Green’s theorem
:   1  1−y
∂N ∂M
M dx + N dy = − dxdy = 2 (x − y)dxdy
C R ∂x ∂y 0 0
 1! 2 "1−y  1 $ %
x (1 − y)2
=2 − xy dy = 2 − y(1 − y) dy
0 2 0 0 2
! " ! "
1 2 1 3 1 1 1
=2 y−y + y =2 −1+ =0
2 2 0 2 2

Figure 5.5 Represents the triangle in the xy-plane bounded by lines


5.1 Green’s Theorem (in the Plane) 143



Illustration 5.5: Verify Green’s theorem for the function F = (x − y) î+xĵ
and the region R bounded by the unit circle
C :−

r (t) = (cost ) î + (sint ) ĵ, 0 ≤ t ≤ 2π



Solution: Consider, −

r = xî+y ĵ ⇒ d−

r = îdx+ ĵdy and F = (x−y)î+xĵ
→ →

∴ F · d−
r = (x − y) dx + xdy
Now,
M = x − y, N = x
∂M ∂N
∴ = −1, =1
∂y ∂x
:  ! "
∂N ∂M
∴ M dx + N dy = − dxdy
R ∂x ∂y
C

= [1 + 1] dxdy
R

=2 dxdy = 2π
: : R

− −
F · d→
r = (x − y) dx + xdy
C C
: 2π
# .
= (cost − sint ) (−sint ) + cos2 t dt
0
 2π
# .
= −sint cost + sin2 t + cos2 t dt
0
 2π
= [−sint cost + 1] dt
0
 2π ! "
sin2t
= − + 1 dt
0 2
1 cos2t 2π
= + |t|2π
0
2 2 0
1
= [cos4π − cos0 ] + 2π = 2π
: 4 !
 "

− → − ∂N ∂M
∴ F ·dr = − dxdy = 2π
C R ∂x ∂y
144 Green’s Theorem, Stokes’ Theorem, and Gauss’ Theorem

Hence, the theorem is verified.


 
1 1
Illustration 5.6: Verify Green’s theorem for C + x dy y dx
where C is

the boundary of the region bounded by the parabola y = x and the lines
x = 1, x = 4, y = 1 (See Figure 5.6).
Solution:

(i) The point of intersection of the


√ √
(a) parabola y = x and the line x = 1 is obtained as y = 1 = 1.
Hence, A(1, 1) is the point of intersection.
√ √
(b) parabola y = x and line x = 4 is obtained as y = 4 = 2.
Hence, D(4, 2) is the point of intersection.

(ii) M = y1 , N= 1
x

∂M 1 ∂N 1
= − 2, =− 2
∂y y ∂x x
9
(iii) C (M dx + N dy) = AB (M dx + N dy) + BD (M dx + N dy)
+ DQA (M dx + N dy) (i)

(a) Along AB : y = 1 ⇒ dy = 0 and x varies from 1 to 4.


 
1 1
(M dx + N dy) = dx + dy
AB AB y x

Figure 5.6 Represents the region bounded by the parabola and lines
5.1 Green’s Theorem (in the Plane) 145

 4
= dx = |x|41 = 3
1
(b) Along BD : x = 4 ⇒ dx = 0 and y varies from 1 to 2.
 
1 1
(M dx + N dy) = dx + dy
BD BD y x
 2
1 1 1
= dy = |y|21 =
1 4 4 4

(c) Along DQA : y = x ⇒ dy = 2√1 x dx and x vary from 4 to 1.
 
1 1
(M dx + N dy) = dx + dy
DQA DQA y x
 1
1 1 1
= √ dx + · √ dx
4 x x 2 x
 1
 √ 1 
= 2 x − √ 
x 4
1 5
=2−1−4+ =−
2 2
Substituting in equation (i), we get

1 5 3
(M dx + N dy) = 3 + − = (ii)
C 4 2 4

(iv) Let R be the region bounded by the parabola y = x and the √ lines
x = 1, x = 4, y = 1. Along the vertical strip, y varies from 1 to x and
in the region R, x varies from 1 to 4.
  4  √x
∂N ∂M 1 1
− dxdy = − 2 + 2 dxdy
R ∂x ∂y 1 1 x y
 4 √ x
 1 1 
= − · y −  dx
 x2 y 1
1
 4 
3 1
= −x− 2 − x− 2 + x−2 + 1 dx
1
 4
 −1 1 1 
= 2x − 2x − + x
 2 2
x 1
146 Green’s Theorem, Stokes’ Theorem, and Gauss’ Theorem

1
=1−4− +4−2+2+1−1
4
3
= (iii)
4

From equations (ii) and (iii), we see that


: 
∂N ∂M
M dx + N dy = − dxdy
C R ∂x ∂y
Hence, Green’s theorem is verified.

5.2 Stokes’ Theorem

Statement: If S be an open surface bounded by a closed curve C (See




Figure 5.7) and F be a continuous and differentiable vector function, then
: 
→ −
− → →

F ·dr = ∇ × F · n̂ dS
C S

where n̂ is the unit outward normal at any point of the surface S.


Proof:


Let F = F1 î + F2 ĵ + F3 k̂ and −

r = xî + y ĵ + z k̂

   


∇ × F · n̂ dS = ∇ × F1 î + F2 ĵ + F3 k̂ · n̂ dS
S S

Figure 5.7 Represents an open surface bounded by a closed curve C


5.2 Stokes’ Theorem 147
     
= ∇ × F1 î · n̂ dS + ∇ × F2 ĵ · n̂ dS

S
 
S

+ ∇ × F3 k̂ · n̂ dS (5.3)
S

Consider,
    ! "
∂ ∂ ∂
∇ × F1 î · n̂ dS = î + ĵ + k̂ × F1 î · n̂ dS
S ∂x ∂y ∂z
 S
∂F1 ∂F1
= −k̂ + ĵ · n̂ dS
S ∂y ∂z

∂F1 ∂F1
= ĵ · n̂ − k̂ · n̂ dS (5.4)
∂y ∂z

Let the equation of the surface S be z = f (x, y),


Then,


r = xî + y ĵ + z k̂ ⇒ →

r = xî + y ĵ + f (x, y)k̂

Differentiating partially with respect to y, we get

∂−

r ∂f
= ĵ + k̂
∂y ∂y
Taking dot product with n̂,

∂−

r ∂f
· n̂ = ĵ · n̂ + k̂ · n̂ (5.5)
∂y ∂y
∂−

r
∂y is tangential and n̂ is normal to the surface S.

∂−

r
· n̂ = 0
∂y
Substituting in the Equation (5.5),
∂f
0 = ĵ · n̂ + k̂ · n̂
∂y
∂f ∂z
ĵ · n̂ = − k̂ · n̂ = − k̂ · n̂ (∵ z = f (x, y))
∂y ∂y
148 Green’s Theorem, Stokes’ Theorem, and Gauss’ Theorem

Substituting in the Equation (5.4),


    ! "
∂F1 ∂z ∂F1
∇ × F1 î · n̂ dS = − ĵ · n̂ − k̂ · n̂ dS
S S ∂z ∂y ∂y

∂F1 ∂z ∂F1
=− · + k̂ · n̂ dS (5.6)
S ∂z ∂y ∂y

The equation of the surface is z = f (x, y).

F1 (x, y, z) = F1 [x, y, f (x, y)] = G(x, y)

Differentiating partially with respect to y,


∂G ∂F1 ∂F1 ∂z
= + ·
∂y ∂y ∂z ∂y
Substituting in the Equation (5.6),
   
∂G
∇ × F1 î · n̂ dS = − k̂ · n̂ dS
S S ∂y

Let R is the projection of S on the xy-plane and dxdy is the projection of dS


on the xy-plane, then k̂ · n̂ dS = dxdy
Thus,
    :
∂G
∇ × F1 î · n̂ dS = − dxdy = Gdx
S R ∂y C1

(∵ Using Green s theorem)


Since the value of G at each point (x, y) of C1 is the same as the value of
F1 at each point (x, y, z) of C and dx is same for both the curves C1 and C,
we get   :

∇ × F1 î · n̂ dS = F1 dx (5.7)
S C
Similarly, by projecting the surface S on to yz and zx planes,
   :
∇ × F2 ĵ · n̂ dS = F2 dy (5.8)
S C

and    :
∇ × F3 k̂ · n̂ dS = F3 dz (5.9)
S C
5.2 Stokes’ Theorem 149

Substituting the Equations (5.7), (5.8), and (5.9) in (5.3),


  : : 
→
− − −
→ 
∇ × F · n̂ dS = (F1 dx + F2 dy + F3 dz) = F · d→
r .
S C C

Note:
If surfaces S1 and S2 have the same bounding curve C, then
    : 
→
− →
− → −
− 
∇ × F · n̂ dS = ∇ × F · n̂ dS = F · d→
r .
S1 S2 C


−  
Illustration 5.7: Verify Stokes’s theorem for F = x2 + y 2 î + 2xy ĵ,
taken round the rectangle bounded by the lines x = ±a, y = 0, y = b (See
Figure 5.8).
Solution:
Consider, −→
r = xî + y ĵ ⇒ d−

r = îdx + ĵdy and let ABCD be the given
rectangle then
    
→ −
− → → −
− → → −
− → → −
− → → −

F ·dr = F ·dr+ F ·dr+ F ·dr+ F · d→r
ABCD AB BC CD DA
and
− −
→     
→ 2 2
F · d r = x + y î + 2xy ĵ · îdx + ĵdy
 
= x2 + y 2 dx + 2xydy
Along AB, x = a ⇒ dx = 0 and y varies from 0 to b
  b
→ −
− → 1
F · d r = −2a ydy = −2a · b2 = −ab2
AB 0 2

Figure 5.8 Represents the rectangle bounded by the lines


150 Green’s Theorem, Stokes’ Theorem, and Gauss’ Theorem

Similarly,
 

− − a   2
F · d→
r = x2 + b2 dx = − · a2 − 2ab2
BC −a 3
  0

− −
F · d→
r = 2a ydy = −ab2
CD b

and  
− −
→ a
2a3
F · d→
r = x2 dx =
DA −a 3
Thus, 
− −

F · d→
r = −4ab2 .
ABCD


Also, since curl · F = −4y k̂,
  b

− a
∴ curl · F dS = −4y k̂ · k̂ dxdy
S 0 −a
 b a
= −4 ydxdy
0 −a
 b
= −4 |x|a−a ydy
0
 2 b
y 
= −8a  = −4ab2
2 0
Hence, the theorem is proved.


Illustration 5.8: Verify Stokes’s theorem when F = (2x − y) î − yz 2 ĵ −
y 2 z k̂, where S is the upper half surface of the sphere x2 + y 2 + z 2 = 1 and
C is the boundary.
Solution: Stokes’s theorem is given as
 
→ −
− → →

F ·dr = curl F · n̂ dS
C S
 
 î ĵ k̂ 
→ 
− 
 = k̂
∴ curl F =  ∂/∂x ∂/∂y ∂/∂z 
 2x − y −yz 2 −y 2 z 
5.2 Stokes’ Theorem 151

Let n̂ = sinθ cosφ î + sinθ sinφ ĵ + cosθ k̂


 →

∴ curl F · n̂ = cosθ
  π  2π

− 2
∴ curl F · n̂ dS = cosθ sinθ dθ dφ = π (i)
S θ=0 φ=0

Again C is the unit circle x2 + y 2 = 1, z = 0

∴ x = cosφ , y = sinφ , z = 0
∴ dx = −sinφ dφ, dy = cosφ dφ, dz = 0
→ → 
−   
∴ F · d−
r = (2x − y) î − yz 2 ĵ − y 2 z k̂ · îdx + ĵdy + k̂dz
= (2x − y) dx − yz 2 dy − y 2 zdz
= − (2cosφ − sinφ ) sinφ dφ
  2π
→ −
− →
∴ F ·dr =− (2cosφ − sinφ ) sinφ dφ
C 0
 2π
 
=− 2cosφ sinφ − sin2 φ dφ
0
=π (ii)

From (i) and (ii), the theorem is verified.




Illustration 5.9: Verify Stokes’s theorem for F = xy 2 î + y ĵ + z 2 xk̂, for
the surface of a rectangular lamina bounded by x = 0, y = 0, x = 1, y =
2, z = 0.


Solution: Here, we have z = 0 ⇒ F = xy 2 î + y ĵ
: :
→ −
− → # 2 .
F ·dr = xy dx + ydy
C C

where C is the path OABCO as shown in Figure 5.9.


Along OA, y = 0 ⇒ dy = 0
Along AB, x = 1 ⇒ dx = 0
Along BC, y = 2 ⇒ dy = 0
Along CO, x = 0 ⇒ dx = 0
:    
→ −
− → 2 2
∴ F ·dr = xy dx + ydy + xy dx + ydy
C OA AB BC CO
152 Green’s Theorem, Stokes’ Theorem, and Gauss’ Theorem

Figure 5.9 Represents the surface of a rectangular lamina bounded by the lines

 2  0  0
=0+ ydy + 4xdx + ydy
0 1 2
 2 0  2
x2
= ydy + 4 − ydy = −2
0 2 1 0
Now,
 
 î ĵ k̂ 

→  ∂
∇ × F =  ∂x ∂
∂y
∂ 
∂z  = î (0) + ĵ (0) + k̂ (−2xy) = −2xy k̂
 xy 2 y 0 

Normal to the surface, n̂ = k̂. dS = dxdy


  
→

∇ × F · n̂ dS = (−2xy) k̂ · k̂dxdy
S S
 1  2
= −2 xy dxdy
x=0 y=0
 1 ! "
y2
= −2 x dx
0 2
 1
4
= −2 x dx
0 2
1
x2
= −4 = −2
2 0
5.2 Stokes’ Theorem 153

Thus, :  
− −
→ →

F · d→
r = ∇ × F · n̂ dS = −2
C S
which verifies Stokes’s theorem.
Illustration 5.10: Use Stokes’s theorem to evaluate

− − → − →
C F · d r if F = (x + y) î + (2x − z) ĵ + (y + z) k̂ and C is the boundary
of the triangle (2, 0, 0) , (0, 3, 0) and (0, 0, 6) (See Figure 5.10).
Solution:
 
 î ĵ k̂ 
→  ∂
− ∂ ∂

 = 2î + k̂
curl F =  ∂x ∂y ∂z 
 x + y 2x − z y + z 
Let φ = 3x + 2y + z = 6

∇φ 3î + 2ĵ + k̂
∴ n̂ = = √
|∇φ| 14

−   3î + 2ĵ + k̂ 7
curl F · n̂ = 2î + k̂ · √ =√
14 14
Projection on xy-plane:

dxdy 14
dS =  
 = 1 dxdy
n̂ · k̂ 

Figure 5.10 Represents the boundary of the triangle


154 Green’s Theorem, Stokes’ Theorem, and Gauss’ Theorem

∴ By Stokes’s theorem we have,


:   √
→ →
− − →
− 7 14
F ·dr = curl F · n̂ dS = √ · dxdy
14 1
C
 S S

= 7dxdy
S
= 7 × Area of the triangle on xy-plane
1
= 7 × × 3 × 2 = 21
2

5.3 Gauss’ Divergence Theorem




Statement: If F be a vector point function having continuous partial
derivatives in the region bounded by a closed surface S, then
 

− −

∇ · F dV = F · n̂ dS
V S

where n̂ is the unit outward normal at any point of the surface S.




Proof: Let F = F1 î + F2 ĵ + F3 k̂,
 

− ∂ ∂ ∂  
∇ · F dV = î + ĵ + k̂ F1 î + F2 ĵ + F3 k̂ dxdydz
V ∂x ∂y ∂z
 V
∂F1 ∂F2 ∂F3
= + + dxdydz (5.10)
V ∂x ∂y ∂z
Assume a closed surface S such that any line parallel to the coordinate
axes intersects S at most at two points. Divide the surface S into two parts:
S1 − the lower and S2 − the upper part. Let z = f1 (x, y) and z = f2 (x, y) be
the equations and n̂1 and n̂2 be normal to the surfaces S1 and S2 respectively
(See Figure 5.11). Let R be the projection of the surface S on the xy-plane.
  $ f2 (x,y) %
∂F3 ∂F3
dxdydz = dz dxdy
V ∂z R f1 (x,y) ∂z

= F3 (x, y, z) |ff21 dxdy
 R

= [F3 (x, y, f2 ) − F3 (x, y, f1 )] dxdy


R
5.3 Gauss’ Divergence Theorem 155

Figure 5.11 Represents the region bounded by a closed surface S

 
= F3 (x, y, f2 ) dxdy − F3 (x, y, f1 ) dxdy (5.11)
R R

Now, dxdy = Projection of dS on xy-plane = n̂ · k̂ dS


For surface S2 : z = f2 (x, y) ⇒ dxdy = n̂2 · k̂ dS2
For surface S1 : z = f1 (x, y) ⇒ dxdy = −n̂1 · k̂ dS1
Substituting in the Equation (5.11),
  
∂F3
dxdydz = F3 n̂2 · k̂ dS2 − F3 (− n̂1 · k̂) dS1
V ∂z S2 S1
 
= F3 n̂2 · k̂ dS2 + F3 n̂1 · k̂ dS1
S2 S1

= F3 n̂ · k̂ dS (5.12)
S

Similarly, projecting the surface S on yz and zx− planes, we get


 
∂F1
dxdydz = F1 n̂ · î dS (5.13)
V ∂x S

and  
∂F2
dxdydz = F2 n̂ · ĵ dS (5.14)
V ∂y S
Substituting Equations (5.12), (5.13), and (5.14) in the Equation (5.10),
   


∇ · F dV = F1 n̂ · î dS + F2 n̂ · ĵ dS + F3 n̂ · k̂ dS
V S S S
156 Green’s Theorem, Stokes’ Theorem, and Gauss’ Theorem
  
= F1 î · n̂ + F2 ĵ · n̂ + F3 k̂ · n̂ dS
S
   


= F1 î + F2 ĵ + F3 k̂ · n̂ dS = F · n̂ dS
S S

→ −

Hence, V ∇ · F dV = S F · n̂ dS
Note: The cartesian form of Gauss’ divergence theorem is
 
∂F1 ∂F2 ∂F3
+ + dxdydz = (F1 dydz + F2 dzdx + F3 dxdy)
V ∂x ∂y ∂z S



Illustration 5.11: Verify Gauss’ divergence theorem for F = 4xz î − y 2 ĵ +
yz k̂ over the cube x = 0, x = 1, y = 0, y = 1, z = 0, z = 1.
Solution: By Gauss’s divergence theorem,
 

− −

∇ · F dV = F · n̂ dS
V S


(i) F = 4xz î − y 2 ĵ + yz k̂

− ∂ ∂  2 ∂
∇· F = (4xz) + −y + (yz)
∂x ∂y ∂z
= 4z − 2y + y = 4z − y

(ii) For the cube (in Figure 5.12): x varies from 0 to 1


y varies from 0 to 1
z varies from 0 to 1

  1 1 1


∇ · F dV = (4z − y)dxdydz
V 0 0 0
 1 1 
= 2z 2 − yz 1 dxdy
0
0 0
 1  1
= dx (2 − y)dy
0 0
 1
 y 2 
|x|10
= 2y − 2 
0
1 3
=2− = (a)
2 2
5.3 Gauss’ Divergence Theorem 157

Figure 5.12 Represents the cube

(iii) Surface S of the cube consist of 6 surfaces, S1 , S2 , S3 , S4 , S4 , S5 , and


S6 .
  

− →
− −

∴ F · n̂ dS = F · n̂ dS + F · n̂ dS+
S S1 S2
  

→ −
→ −

F · n̂ dS + F · n̂ dS + F · n̂ dS
S3 S4 S5



+ F · n̂ dS (b)
S6
dxdy
1. On S1 (OABC) : z = 0, n̂ = −k̂, dS = = dxdy
|n̂·k̂|
x and y both vary from 0 to 1.
     


∴ F · n̂ dS = 4xz î − y 2 ĵ + yz k̂ · −k̂ dxdy = 0
S1 S1

dxdy
2. On S2 (DEF G) : z = 1, n̂ = k̂, dS = = dxdy
|n̂·k̂|
x and y both vary from 0 to 1.
     


∴ F · n̂ dS = 4xz î − y 2 ĵ + yz k̂ · k̂ dxdy
S2 S2
 1 1   
1  2 1
= ydxdy =   = 1 |x|1 = 1
y
2 2 0 2
0 0 0 0
158 Green’s Theorem, Stokes’ Theorem, and Gauss’ Theorem

3. On S3 (OAF E) : y = 0, n̂ = −ĵ, dS = dzdx


|n̂·j| = dzdx
x and z both vary from 0 to 1.
     


∴ F · n̂ dS = 4xz î − y 2 ĵ + yz k̂ · −ĵ dzdx = 0
S3 S3

dzdx
4. On S4 (BCDG) : y = 1, n̂ = ĵ, dS = |n̂·j| = dzdx
x and z both vary from 0 to 1.
     


∴ F · n̂ dS = 4xz î − y 2 ĵ + yz k̂ · ĵ dzdx
s4 S4
 1 1
= −dzdx = −1
0 0

dydz
5. On S5 (OABC) : x = 0, n̂ = −î, dS = = dydz
|n̂·î|
y and z both vary from 0 to 1.
     


∴ F · n̂ dS = 4xz î − y 2 ĵ + yz k̂ · −î dydz = 0
S5 S5

dydz
6. On S6 (ABGF ) : x = 1, n̂ = î, dS = = dydz
|n̂·î|
y and z both vary from 0 to 1.
    


∴ F · n̂ dS = 4xz î − y 2 ĵ + yz k̂ · î dydz
S6 S6
 1 1
= 4zdzdx = 2
0 0

Substituting in the equation (b),




− 1 3
∴ F · n̂ dS = 0 + + 0 − 1 + 0 + 2 = (c)
S 2 2
From equations (a) and (c), we get
 

− −
→ 3
∇ · F dV = F · n̂ dS =
V S 2
5.3 Gauss’ Divergence Theorem 159

Hence, the Gauss theorem is verified.




Illustration 5.12: Verify Gauss’ divergence theorem for F = 2xz î + yz ĵ +
z 2 k̂ over the upper half of the sphere x2 + y 2 + z 2 = a2 .
Solution:
By Gauss’s divergence theorem,
 

− −

∇ · F dV = F · n̂ dS
V S



(i) F = 2xz î + yz ĵ + z 2 k̂

− ∂ ∂ ∂ 2
∇· F = (2xz) + (yz) + (z )
∂x ∂y ∂z
= 2z + z + 2z = 5z

(ii)  




∇ · F dV = 5z dxdydz
V V

Taking x = rsinθ cosφ , y = rsinθ sinφ , z = rcosθ , equation of


the sphere x2 + y 2 + z 2 = a2 reduces to r = a and dxdydz =
r2 sinθ dr dθ dφ.
For the upper half of the sphere (hemisphere in Figure 5.13),
r varies from 0 to a

Figure 5.13 Represents the cube


160 Green’s Theorem, Stokes’ Theorem, and Gauss’ Theorem

π
θ varies from 0 to 2
φ varies from 0 to 2π
  2π  π 

− 2
a
∇ · F dV = 5 rcosθ · r2 sinθ drdθdφ
V φ=0 θ=0 0
  π  4 a
2π 2 r 
=5 dφ cosθ sinθ dθ  
φ=0 θ=0 4 0
 π
1  cos2θ  2 a4 5
= 5|φ|2π
0 ·  −  · = πa4 (a)
2 2 0 4 4

(iii) The given surface is open and closed by the circular surface S2 in xy-
plane. Thus, the surface S consists of two surfaces S1 and S2 .
  

− →
− →

∴ F · n̂ dS = F · n̂ dS + F · n̂ dS (b)
S S1 S2

(1) Surface S1 (ABCEA) : This is the curved surface of the upper


half of the sphere.
Let φ = x2 + y 2 + z 2

∇φ 2xî + 2y ĵ + 2z k̂ xî + y ĵ + z k̂
∴ n̂ = = =
|∇φ| 4x2 + 4y 2 + 4z 2 a

Let R be the projection of S1 on the xy-plane, which is a circle


x2 + y 2 = a2 .

dxdy adxdy
dS =  =

n̂ · k̂  z



F · n̂dS
S1
  
 xî + y ĵ + z k̂ adxdy
= 2xz î + yz ĵ + z 2 k̂ ·
R a z
5.3 Gauss’ Divergence Theorem 161

 
= 2x2 + y 2 + z 2 dxdy
 R
 
= 2x2 + y 2 + a2 − x2 − y 2 dxdy
 R
 
= x2 + a2 dxdy
R

Taking x = rcosθ , y = rsinθ ,the equation of the circle x2 +y 2 =


a2 reduces to r = a and dxdy = r dr dθ. Along the radius vector
OP , r varies from 0 to a and for the complete circle, θ varies from
0 to 2π (See Figure 5.14).

  2π  a

→ 
∇ × F · n̂ dS = r2 cos2 θ + a2 r dr dθ
S1 0 0
 2π ! 4 a  2 a "
r   
2 r 
=   2
 4  cos θ + a  2  dθ
0 0 0
 2π ! 4 "
a 1 + cos2θ a4
= + dθ
0 4 2 2
 

4 5 1 sin2θ 2π 5
=a  θ+  = πa4
8 8 2 0 4

Figure 5.14 Represents the circle


162 Green’s Theorem, Stokes’ Theorem, and Gauss’ Theorem

(2) Surface S2 (ABCDA) : This is the circle x2 + y 2 = a2 in


xy−plane z = 0, n̂ = −k̂
dxdy
∴ dS =   = dxdy

n̂ · k̂ 
    


∴ F · n̂ dS = 2xz î + yz ĵ + z 2 k̂ · −k̂ dxdy = 0
S2 S2

Substituting in Equation (b),




→ 5
∴ F · n̂ dS = πa4 (c)
S 4
From Equations (a) and (c), we have
 

− −
→ 5
∇ · F dV = F · n̂ dS = πa4
V S 4
Hence, Gauss’ theorem is verified.
  − →
Illustration 5.13: Evaluate S yz î + zx ĵ + xy k̂ · d S , where S is the
surface of the sphere in the first octant.
Solution: By Gauss’ divergence theorem,
 
→ −
− → →

F · dS = ∇ · F dV (i)
S V


F = yz î + zx ĵ + xy k̂

− ∂ ∂ ∂
∴∇ · F = (yz) + (zx) + (xy) = 0
∂x ∂y ∂z
From equation (i),
 
→ −
− → −

F · dS = ∇ · F dV = 0
S V

Illustration 5.14: Evaluate S (lx + my + nz) dS, where l, m, n are the


direction cosines of the outer normal to the surface whose radius is 2 units.
Solution: By Gauss’
 divergence 
theorem,

− →

F · n̂ dS = ∇ · F dV (i)
S V
5.4 Exercise 163



(i) F · n̂ = lx + my + nz
   
= x î + y ĵ + z k̂ · l î + m ĵ + n k̂



∴ F = x î + y ĵ + z k̂


(ii) ∴∇ · F = ∂
∂x (x) + ∂
∂y (y) + ∂
∂z (z) = 3

(iii)
 


∇ · F dV = 3 dV
V V
Volume of the region bounded by
=3
the sphere of 2 unit radius
4
=3· π(2)3 = 32 π
3

From Equation (i),

5.4 Exercise
→ 
− 
1. Verify Green’s theorem for the function F = y 2 − 7y î+(2xy +2x)ĵ
and curve C : x2 + y 2 = 1.
(Answer: 9π)
9  2

2. Verify Green’s theorem for C 3x − 8y dx + (4y − 6xy)dy where C
is the boundary of the triangle with vertices (0, 0) , (1, 0) and (0, 1).
 
Answer : 53
9
3. Using Green’s theorem, evaluate the linear integral C (siny dx
+cosxdy) counterclockwise, where C is the boundary of the triangle
with vertices (0, 0) , (π, 0) , (π, 1).
 
Answer : − 1 + π2 − π1 cos1
9 #  .
4. Evaluate C x2 − 2xy dx + (x2 y + 3)dy around the boundary C of
the region y 2 = 8x, x = 2.
 
Answer : 128
5
164 Green’s Theorem, Stokes’ Theorem, and Gauss’ Theorem
9  
5. Verify Green’s theorem in the plane for C 2x − y 2 dx − xydy where
C is the boundary of the region enclosed by the circles x2 + y 2 = 1 and
x2 + y 2 = 9.
(Answer : 60π)
9    
6. Verify Green’s theorem for C x2 − 2xy dx + x2 y + 3 dy where C
2
is
 the boundary
1
 of the region bounded by y = x and the line y = x.
Answer : 4
→ 
− 
7. Verify Stokes’s theorem for F = x2 − y 2 î + 2xy ĵ, in the rectangular
region in the xy-plane bounded by the lines x = ±a, y = 0, y = b.
 
Answer : 4ab2


8. Verify Stokes’s theorem for F = (x + y) î + (y + z) ĵ − xk̂ and S is
the surface of the plane 2x + y + z = 2 which is in the first octant.
(Answer : − 2)
9
9. Evaluate by Stokes’s theorem C (ex dx + 2ydy − dz), where C is the
curve x2 + y 2 = 4, z = 2.
(Answer : 0)


10. Verify Stokes’s theorem for F = xz î+y ĵ +y 2 xk̂ over the surface of the
tetrahedron bounded by the planes y = 0, z = 0 and 4x + y + 2z = 4
above the yz-plane.
(Answer : 0)
→ 
− 
11. Verify Stokes’s theorem for F = x2 +y + 2 î + 2xy ĵ + 4zex k̂ over
the surface S of the paraboloid z = 9 − x2 + y 2 above the xy-plane.
(Answer : − 9π)

→ →

12. Evaluate using Stokes’s theorem S ∇ × F · n̂ dS where F = 3y î −
xz ĵ + yz 2 k̂ and S is the surface of the paraboloid x2 + y 2 = 2z bounded
by the plane z = 2 and C is its boundary traversed in the clockwise
direction.
(Answer : 20π)
13. Prove that √ dS
= √4π , where S is the ellipsoid a2 x2 +
S a2 x2 +b2 y 2 +c2 z 2 abc
b2 y 2 + c2 z 2 = 1.
5.4 Exercise 165

→ −
− → →

14. Evaluate S F · d S using divergence theorem where F = x3 î+y 3 ĵ +
z 3 k̂ and S is the surface of the sphere x2 + y 2 + z 2 = a2 .
 
Answer : 12
5 πa
5

→ −
− → →

15. Evaluate F · d S using Gauss’ divergence theorem where F 2xy î +
yz 2 ĵ +zx k̂ and S is the surface of the region bounded by x = 0, y = 0,
z = 0, y = 3, x + 2z = 6.
 
Answer : 351
2


16. Verify Gauss’ divergence theorem for F = (x2 − yz)î + (y 2 − zx)ĵ +
(z 2 − xy)k̂ over the region R bounded by the parallelepiped 0 ≤ x ≤
a, 0 ≤ y ≤ b, 0 ≤ z ≤ c.
(Answer : abc(a + b + c))


17. Verify Gauss’ divergence theorem for F = xî + y ĵ + z k̂ over the region
R bounded by the sphere x2 + y 2 + z 2 = 16.
(Answer : 256π)


18. Verify Gauss’ divergence theorem for F = 2xy î + 6yz ĵ + 3zxk̂ over
the region bounded by the coordinate planes and the plane x+y +z = 2.
 
Answer : 22
3
6
MATLAB Programming

6.1 Basic of MATLAB Programming


MATLAB is an acronym for MATrix LABoratory. MATLAB was created to
make it simple to use the LINPACK (linear system package) and EISPACK
(eigen system package) projects’ matrix tools.
MATLAB, see The MathWorks Inc., 2005, and D. Houcque, 2005, is
a high-performance technical computing language. It combines computing,
visualization, and a programming environment into one package. MATLAB
is also a contemporary programming language environment, with advanced
data structures, built-in editing and debugging tools, and object-oriented pro-
gramming capabilities. Because of these features, MATLAB is an outstanding
teaching and research tool.
In this section, we presume that the reader is familiar with basic termi-
nology like different windows in MATLAB, basic input commands, creating
MATLAB variables, error messages, functions, etc.

6.1.1 Basic of MATLAB Programming


Many mathematical functions are predefined in MATLAB. By typing_help
, and help users can access inbuilt commands for elemen-
tary and special mathematical functions. A large number of mathematical
functions can be evaluated using MATLAB. There are two types of arithmetic
operations in MATLAB: array operations and matrix operations. Users may
execute mathematical computations using these arithmetic operations, such
as adding two integers, raising the elements of an array to a specific power,
or multiplying two matrices.

167
168 MATLAB Programming

6.1.1.1 Introductory MATLAB programmes


In this section, we discuss some basic examples of MATLAB programming
to recall basic operations, functions, inputs, and mathematical computations.
And it will help us in further MATLAB programs.

1. Write a MATLAB program to find the addition, subtraction, and


division of two numbers.

Solution: Let a = 3 and b = 5 be two numbers for these operations.


Assume c1, c2, and c3 variables for storing these results. So, c1, c2,
and c3 are the outputs for these operations respectively.
Therefore

>> a=3;

>> b=5;

>> c1=a+b

c1 =

>> c2=a-b

c2 =

-2

>> c3=a/b

c3 =

0.6000

In this example, the semicolon at the end of the statements for a and b,
suppress output to run the program smoothly. Whereas in the statement
c1=a+b, c2=a-b, and c3=a/b are not followed by a semicolon so the
stored values “dumped” as the output.
6.1 Basic of MATLAB Programming 169

2. Write a MATLAB program to evaluate the given expression for any


two numbers.
b
2a + b2 − ab + − 10
a

Solution: In this MATLAB program, write a user-defined program.


Users define the programs are those programs in which the input of the
programs are taken from the user. The main advantage of the user define
programs are, the user can give any input rather than a fixed input. In
this program, we use the “input” command to take values from the user.
The syntax is given as follow:

Input:

Output:

Enter 1st number:


-10

Enter 2nd number:


9

c=

140.1000

In this example, is given for the newline. i.e., once the program is
run user can see the input statement, and the user can give input in the
next line in the “command window”.

3. Write a MATLAB program to print “Welcome to MATLAB Pro-


gramming”

Solution:
170 MATLAB Programming

Input:

>> fprintf (‘Welcome to MATLAB Programming \n’)

Output:
Welcome to MATLAB Programming
In this program, “fprintf (‘ ’)” is used to print given input in a single
quotation mark (‘ ’).
4. Write a user define MATLAB program to print square, cube, and
square roots to preserve 4 digits after the decimal point for any
number.

Solution:

Input:

Output:

1) If the number is 4 then


Enter the number : 4
The square of 4 equal 16.
The cube of 4 equal 64.
The square root of 4 is 2.0000.
2) If the number is 5 then
Enter the number : 5
The square of 5 equal 25.
The cube of 5 equal 125.
The square root of 5 is 2.2361.
6.1 Basic of MATLAB Programming 171

In this program, “%4u” represents 4 digit integer value of the input


number, and to preserve 4 digits after the decimal point we use “%6.4f”.
5. Write a MATLAB program to plot sinx for 0 ≤ x ≤ 2π.
Solution:
Input:

Output:

In this program, to plot sinx for 0≤x≤2π, use the inbuilt command
“plot(x,y)” then define x values using the command x=0:0.1:2*pi then
store the values of sinx in variable y.
6. Write a MATLAB program to draw any curve.
Solution:
Input:

Output:
172 MATLAB Programming

7. Write a MATLAB program to plot user define line pattern, color,


and thickness.

Solution:

Input:

Output:
6.1 Basic of MATLAB Programming 173

8. Write a MATLAB program to plot multiple curves.

Solution:

Input:

Output:

9. Write a MATLAB program to plot multiple panels.

Solution:

Input:
174 MATLAB Programming

Output:

10. Write a MATLAB program to plot a helix.

Solution:

Input:
6.1 Basic of MATLAB Programming 175

Output:

11. Write a MATLAB program to print any number consecutive 5


times.

Solution: In this program, we take input from the user as we have to


print any number and use “for loop” for printing any number consecutive
5 times.

Input:

Output:
Enter the number : 10
x=
10
x=
10
x=
10
x=
176 MATLAB Programming

10
x=
10

12. Write a MATLAB program to print the utility of dummy index for
any number consecutive 6 times. Also, take index from user.

Solution:

Input:

Output:
Enter the number : 2
ans =
2
ans =
4
ans =
8
ans =
16
ans =
32
ans =
64
Also, by user define index

Input
6.1 Basic of MATLAB Programming 177

Output:
Enter the number : 2
Enter the index : 3
ans =
2
ans =
4
ans =
8

13. Write a MATLAB program to find a sum upto 10.

Solution:

Input:

Output:
sum1 =
55
By user define number
178 MATLAB Programming

Input:

Output:
Enter a number upto which sum need to find: 5
a=
5
sum1 =
15
14. Write a MATLAB program to find the sum of the first four elements
in an array of six elements.
Solution:
Input:

Output:
sum1 =
5
By user define
6.1 Basic of MATLAB Programming 179

Output:

Enter an array : [1 2 3 4 5 6 7 8 9 10]


a=
Columns 1 through 8
1 2 3 4 5 6 7 8
Columns 9 through 10
9 10
Enter a number to find sum upto the given number of elements: 7
b=
7
sum1 =
28

15. Write a MATLAB a program for a double loop.

Solution:

Input:

Output:

n= 1m= 1
n= 1m= 2
n= 2m= 1
n= 2m= 2
n= 3m= 1
n= 3m= 2

16. A MATLAB program for complicated use of loop and index.

Solution:

Input:
180 MATLAB Programming

Output:
sum1 =
24
Summation performed by this program
sum1 = a(b(1))+a(b(2))+a(b(3))+a(b(4))
= a(2)+a(3)+a(5)+a(7)
= 5+7+9+3
= 24

17. Write a MATLAB program to find greater than or less than or


equals to of any two numbers.

Solution:

Input:

Output 1:

Enter 1st number : 10

a=
6.1 Basic of MATLAB Programming 181

10

Enter 2nd number : 12

b=
12

10 is less than or equal to 12

Output 2:

Enter 1st number : 10

a=

10

Enter 2nd number : -1

b=

-1

10 is greater than -1.000000e+00

18. Write a MATLAB program to represent the use of “if - elseif - else”
loop.

Solution:

Input:
182 MATLAB Programming

Output:
4 is less than 5 but greater than 1

19. Write a MATLAB program to determine whether a given year is a


leap year or not.

Solution:

Input:

Output:
Enter a year in YYYY format : 2020
2020 is a leap year
Enter a year in YYYY format : 2022
2022 is not a leap year
6.1 Basic of MATLAB Programming 183

6.1.1.2 Representation of a Vector in MATLAB


In this section, we represent a vector as a special case of a matrix. An array of
dimension 1×n is called a row vector and m×1 dimensional array is called a
column vector. The elements of a vector are enclosed by square brackets. The
elements of a row vector are separated using spaces or by commas, whereas
the elements of a column vector are separated using a semicolon(;).
For example, for a row vector v1 and v2

>> v1=[1 2 3 4 5]

v1 =

1 2 3 4 5

>> v2=[1, 2, 3, 4, 5]

v2 =

1 2 3 4 5

And for a column vector cv1 and cv2

>> cv1=[1;-4;3;9;-10]

cv1 =

1
-4
3
9
-10

>> cv2=[1;2;-2;0.1]
184 MATLAB Programming

cv2 =

1.0000
2.0000
-2.0000
0.1000

The transpose of a row or a column vector is obtained using an apostrophe


or a single quote (’).

For example, transpose of a row vector v1 and a column vector cv1

>> w=v1’

w=

1
2
3
4
5

>> w1=cv1’

w1 =
1 -4 3 9 -10

To access elements, we use colon notation (:). For example, for the first three
elements of the w1 vector,

>> w1(1:3)

ans =

1 -4 3
6.1 Basic of MATLAB Programming 185

Or, all elements after 3rd element

>> w1(3:end)

ans =

3 9 -10

Similarly, we can find elements in the column vector also.


20 Write a MATLAB program to find the addition of two vectors.
Solution:
Input:
a = [2 12 25];
b = [3 7 4];
c = a+b
Output:
c=
5 19 29
21 Write a MATLAB program to find the multiplication of a vector by
a scalar.
Solution:
Input:
a = [2 12 25];
b=4
c = a*b

Output:
c=
8 48 100
186 MATLAB Programming

6.1.1.3 Representation of a Matrix in MATLAB


A matrix is an array of numbers. We use the following steps to enter a matrix
in MATLAB.
1. We begin with a left square bracket. i.e., [
2. Separate each element in a row with space or comma. i.e., (,)
3. Use a semicolon (;) to separate each row in the matrix.
4. After adding all rows in a matrix, end matrix with the right square
bracket. i.e., ]
For example, to enter a matrix
! "
1 2
A=
3 4
We use
>> A=[1 2; 3 4]
and the output matrix in MATLAB will be

A=
1 2
3 4

For non-square matrix


⎡ ⎤
1 2
A=⎣ 3 4 ⎦
5 6
We have,
>> A=[1, 2; 3, 4; 5, 6]

A=
1 2
3 4
5 6
Once the matrix is stored in MATLAB, we can view a particular element of
the matrix. For example, to view elements in row 1st and column 2nd, we use
A(2,1)
6.1 Basic of MATLAB Programming 187

>> A(2,1)

ans =

To find the dimension of a vector or a matrix, we use the size command. For
example, to find the dimension of the matrix
⎡ ⎤
1 2
A=⎣ 3 4 ⎦
5 6

We have

>> size(A)

ans =

3 2

For a square matrix ! "


1 2
A=
3 4
We have

>> size(A)

ans =

2 2

i.e., size(A)=[m n]

To find the transpose of a matrix, we can use an apostrophe or a single quote


(’). For example, transpose of a matrix
! "
0 1 3
B=
−2 0 7

We have
188 MATLAB Programming

B=

0 1 3

-2 0 7

>> B’

ans =
0 -2
1 0
3 7

6.2 Some Miscellaneous Examples using MATLAB


Programming

Solution:
6.2 Some Miscellaneous Examples using MATLAB Programming 189

Output:

Solution:

Input:
x-coordinate of a vector: 3
y-coordinate of a vector: 4
z-coordinate of a vector: -5
Output:
The modulus of the vector (3,4,-5) = 7.071068

Solution:

a = [3 -1 -4];
b = [-2 4 -3];
c = [1 2 -1];
190 MATLAB Programming

z=3*a-2*b+4*c
Output:
z=
17 -3 -10

Solution:
a = [1 2 1];
b = [2 1 1];
c = [3 4 1];

z1 = a+2*b+c

z2 = z1.*z1

z3 = sum(z2)

z4 = sqrt(z3)

Output:
z1 =
8 8 4

z2 =

64 64 16

z3 =

144

z4 =

12
6.2 Some Miscellaneous Examples using MATLAB Programming 191

Solution:

Input-1:
x-coordinate of a vector: 1
y-coordinate of a vector: 0
z-coordinate of a vector: 0
Output-1:
The vector (1, 0, 0) is a unit vector.
Input-2:
x-coordinate of a vector: 1
y-coordinate of a vector: 1
z-coordinate of a vector: 0
Output-2:
The vector (1, 1, 0) is not a unit vector.
Input-3:
x-coordinate of a vector: 0
y-coordinate of a vector: 0
z-coordinate of a vector: 0
192 MATLAB Programming

Output-3:
The vector (0, 0, 0) is a zero vector.
Input-4:
x-coordinate of a vector: 1/sqrt(3)
y-coordinate of a vector: 0
z-coordinate of a vector: sqrt(2/3)
Output-4:
The vector (5.773503e-01, 0, 8.164966e-01) is a unit vector.

Solution:

Input:
The first vector in the form of [x y z]: [-1 6 8]
The second vector in the form of [x y z]: [1 3 -4]
Output:

z1 =

0 9 4

z2 =

-2 3 12
6.2 Some Miscellaneous Examples using MATLAB Programming 193

Solution:

Input:
Enter x-coordinate of an arbitrary vector : 3
Enter y-coordinate of an arbitrary vector : 4
Enter z-coordinate of an arbitrary vector : 5
Enter an arbitrary scalar for a product with a given arbitrary
vector : -1
Output:

a=

3 4 5

da =

-3 -4 -5
194 MATLAB Programming

Solution:

Output:

z1 =

20

z2 =

0.0000e+00 + 2.3220e+02i

x.y = 20

The angle between x and y is 0.000000.

Solution:
6.2 Some Miscellaneous Examples using MATLAB Programming 195

Input 1:
Enter a first vector in the format [x y z] = [1 2 3]
Enter a second vector in the format [x y z] = [-1 -2 -3]
Output 1:

z1 =

-14

z2 =

1.8000e+02 - 1.9085e+02i

x.y =-14

The angle between x and y is 180.000000


Input 2:
Enter a first vector in the format [x y z] = [1 0 0]
Enter a second vector in the format [x y z] = [0 1 0]
Output 2:

z1 =

z2 =
90

x.y =0
196 MATLAB Programming

The angle between x and y is 90.000000


Input 3:
Enter a first vector in the format [x y z] = [1 -5 6]
Enter a second vector in the format [x y z] = [6 3 -1]
Output 3:
z1 =

-15

z2 =

1.8000e+02 - 1.8624e+02i

x.y =-15
The angle between x and y is 180.000000

Solution:
6.2 Some Miscellaneous Examples using MATLAB Programming 197

Input 1:
Enter a first vector in the format [x y z] = [1 -2 2]
Enter a second vector in the format [x y z] = [0 0 -1]
Output 1:
(i) is true. 2.000000 < = 3.000000
(ii) is true. 2.449490 < = 4.000000
(iii) is true. 3.741657 > = 2.000000
Input 2:
Enter a first vector in the format [x y z] = [1 -1 -1]
Enter a second vector in the format [x y z] = [2 -4 7]
Output 2:
(i) is true. 8.306624 < = 14.387495
(ii) is true. 8.366600 < = 10.038675
(iii) is true. 8.602325 > = -6.574573

Solution:
198 MATLAB Programming

Input:
Enter a first vector in the format [x y z] =
[3 -2 1]
Enter a second vector in the format [x y z] =
[1 -2 1]
Output:
The projection of a on b is 3.265986.
The projection of b on a is 2.138090.

Solution:

Input 1:
Enter a first vector in the format [x y z] =
[1 0 0]
Enter a second vector in the format [x y z] =
6.2 Some Miscellaneous Examples using MATLAB Programming 199

[0 1 0]
Output 1:
c=
0
The vectors a and b are perpendicular to each other.
Input 2:
Enter a first vector in the format [x y z] =
[1 0 0]
Enter a second vector in the format [x y z] =
[1 0 0]
Output 2:
c=
1
The vectors a and b are not perpendicular to each other.
Input 3:
Enter a first vector in the format [x y z] =
[1 2 3]
Enter a second vector in the format [x y z] =
[-2 1 3]
Output 3:
c=
9
The vectors a and b are not perpendicular to each other.

Solution:
200 MATLAB Programming

Input:
Enter a first vector in the format [x y z] =
[2 -3 1]
Enter a second vector in the format [x y z] =
[1 -1 2]
Output:
c=
-5 -3 1
The cross product is (-5, -3, 1).

Solution:

Output:
The total work done is 10.000000 units.

Solution:
6.2 Some Miscellaneous Examples using MATLAB Programming 201

Output:
The moment of the force F about the point A is (-3, 19, -4).
The magnitude of the moment is 19.646883.

Solution:
t = 0:0.1:3*pi;
plot3(sin(2*t),cos(t),t)
f = @(t) sqrt(4*cos(2*t).∧ 2 + sin(t).∧ 2 + 1);
len = integral(f,0,3*pi)
Output:
202 MATLAB Programming

len =
17.2220

2 −y 2
xe−x

Solution:

Output:

For the function:


3x2 y − y 3 z 2
6.2 Some Miscellaneous Examples using MATLAB Programming 203

Solution:
t = 0:0.1:2*pi;
f = @(t)(-t.*sin(t)+cos(t).∧2+sin(t));
len = integral(f,0,2*pi)
Output:
len =
9.4248

Solution:
204 MATLAB Programming

Output:

Solution:

Output:
q1 =
2.734244598320928e+03
q2 =
2.734244599944285e+03

Solution:
fun = @(x,y) 1./(1 + x + y);
References 205

ymax = @(x) x;
q = integral2(fun,0,1,0,ymax)
Output:
q=
0.261624071883185

References
[1] The MathWorks Inc. MATLAB 7.0 (R14SP2). The MathWorks Inc.,
2005. The MathWorks Inc. MATLAB 7.0 (R14SP2). The MathWorks
Inc., 2005.
[2] D. Houcque. Introduction to MATLAB for Engineering Students. North-
western University, 2005.
Index

A Binormal, 60, 65
Absolute, 3 Bold, 1
Acceleration, 1, 106 Boundary, 135, 144, 153
Access, 167, 184 Bounded, xii, 124, 129, 134
Accurate, xi
Acronym, 167 C
Addition, v, 5, 6, 9, 168 Cables, 29
Adjacent, 38 Calculus, vi, 111
Adjoining, 58 Capabilities, 167
Advance, 66 Cartesian, 2, 4, 58
Aircraft, 55 Centrifugal, 1
Algebraic, 29, 37 Charge, 1
Analogous, 73 Circle, 64, 117, 126, 143
Analysis, 1 Circular, 127, 133, 160
Analytic, ix Circulation, vii, 112, 117
Angle, vi, ix, 12, 29 Clockwise, 66
Angled, 29 Closed, 112, 114, 123, 129
Angles, 12 Coefficients, 120
Angular, 89 Collinear, v, 8
Applied, 46, 48, 49, 50 Command, 167, 168, 171, 187
Arbitrary, 1, 92, 93, 95 Commands, x, 167
Arc Length, vi, 57, 61 Common, 8
Arc rate, xi, 65 Commutative, 10, 30
Area, vii, 1, 39, 122, 128 Component, 71, 124, 128, 137
Arithmetic, x, 167 Components, v, 2, 5, 13, 25
Arrow, 1 Computations, 167
Atmosphere, 56 Concept, ix, 1, 7, 55
Axis, 2, 4, 5, 111, 137 Cone, 76
Consecutive, 38, 60, 175, 176
B Conservative, 91, 113, 114, 120
Basic, v, 45, 55, 167, 168 Constant, 48, 54, 56, 72, 74

207
208 Index

Construction, 55 Directions, 7, 8, 10, 31


Contemporary, 167 Displaced, 55
Coordinate, 2, 4, 58, 71 Displacement, 1, 46, 48, 51
Coplanar, v, 8, 107 Displacing, 112
Cosine, 12 Distance, 1, 74
Cosines, vi, ix, 12, 16 Distributive, 10, 31, 40, 42
Counterbalance, 25 Divergence, vii, 55, 73, 86
Critical, 55 Dot, xi, 29, 30, 40
Cross, 37, 38, 39, 60 Drag, 1
Cube, 156, 157, 159, 170 Dumped, 168
Curl, 55, 73, 86, 89
Curvature, 60, 64, 67, 105 E
Curve, 67, 73, 82, 84 Elasticity, 55
Curves, 57, 58, 84, 106 Electric, 1, 45, 56, 87
Cylinder, 129, 134 Electrical, 1
Electromagnetic, ix, 103
D Electromagnetism, 55, 112
Debugging, 134 Electrostatics, 55
Decimal, 170, 172 Elements, 8, 31, 167, 178
Decreases, 75, 81 Ellipse, 84, 127, 128, 129
Density, 1, 56 Endpoints, 11, 58
Dependent, 1 Energy, 1
Derivation, ix Environment, 167
Derivative, xii, 64, 71, 80 Equal, 2, 10, 38, 59
Derivatives, 64, 74, 75, 135 Error, 158
Difference, 10 Essentials, ix
Different, 1, 8, 10, 56, 167 Evaluation, vii, 123, 135
Differentiable, 71, 86, 89, 146 Execute, 167
Differential, vi, ix, 55, 73, 111 Expression, 10, 40, 42, 115
Differentiating, 147
Differentiation, vi, 111 F
Digit, 171 Field, 1, 45, 55, 56, 73
Digits, 170, 171 Flowing, 56, 86, 123
Dimensional, ix, 2, 3, 5, 57 Fluid flow, 3, 57, 88
Dimensions, 12, 55, 76 Flux, 87, 123, 127, 128
Directed, 1 Force, 1, 25, 26, 46
Direction, ix, xi, 1, 2, Forces, 25, 46, 48, 54
Directional, xii, 73, 74, 75 Frequency, 1
Index 209

Function, 55, 57, 61, 73 Intersection, 56, 127, 144


Functions, 55, 56, 167, 158 Irrotational, 90, 92, 95, 112
Isothermal, 56
G
Gauss, 154, 156, 159, 162 L
Generators, 55 Laser, 55
Geometric, ix, xi, 1, 8 Law, 8, 9, 10, 11
Geometrical, 1, 30 Length, 1, 3, 57, 59
Geometrically, 30, 55 Level curves, 84
Geometry, ix, 45, 111 Lift, 1
Gradient, 55, 7, 74, 81 Line, vii, 8, 46, 58
Gravitational, 1, 56 Linear, 8, 89
Gravity, 25 Lines, 8, 85, 124, 136
Green, 135, 137, 138, 140 Locus, 42

H M
Hangs, 29 Machinery, ix
Heat flow, 55 Magnetic, 1, 56
Helix, 63 Magnitude, 1, 8, 18, 86
Hemisphere, 159 Magnitudes, 1, 50
Hexagon, 14 Mass, 1, 86
High-performance, 167 Mathematical, ix, 167, 168
Horizontal, 25 MATLAB, ix, 167, 170, 178
Matrices, 167
I
Maximum, 74, 108
Incompressibility, 87 Measure, vi, 30
Incompressible, 87 Measured, 1, 74
Increasing, 66 Measurement, ix, 48
Independence, ix, 113 Messages, 167
Independent, vii, 113, 114, 115 Moduli, 10, 16, 31
Independently, 1 Modulus, 3, 4, 9, 47
Initial, 1, 2, 7, 9 Moment, 1, 46, 47, 51
Input, 167, 159, 171, 175 Momentum, 1
Integral, 111, 122, 129, 135 Mutually, 32, 52, 60
Integrals, 111, 112, 113, 135
Integration, 111, 113, 114, 117 N
Intensity, 1, 56 Negative, 2, 3, 6, 10
Interpretation, vii, 38, 86, 69 Neighboring, 60
210 Index

Newline, 169 Points, 1, 2, 12, 60


Normal, 60, 64, 74, 84 Position, 2, 3, 12, 14
Null, 7, 39, 47 Positive, 6, 112, 126, 129
Potential, 1, 91, 93, 133
O
Power, 1, 167
Octant, 124, 125, 126, 129 Preserve, 170, 171
Operations, x, 167 Principal, 60, 64
Operator, vii, 73, 100 Print, 169, 175
Opposite, 2, 6, 7, 10 Product, x, xi, 7, 200
Order, 60 Products, 29, 31, 45, 60
Ordinary, 55, 73 Program, 167, 168, 169, 179
Origin, 2, 11, 58, 117 Projecting, 148, 155
Orthogonal, 66, 75, 81, 123 Projection, 30, 46, 73, 198
Osculating, 60, 64 Projections, vi, 30
Output, 168, 179, 186, 202
Outputs, x, 158 Q
Outward, 86, 87, 146, 154 Quantities, ix, 1, 55, 73
Quantity, 1, 46, 55
P
Paddlewheel, 90 R
Parabola, 114, 115, 144, 145 Radius, 56, 117, 128, 162
Parallel, 8, 39, 60, 86 Rectangle, 140
Parallelepiped, 86, 87 Rectangular, 5, 152
Parallelogram, 8, 9, 13, 38 Region, 55, 90, 112, 154
Parallelopiped, 87 Regular, 14
Parameter, 57, 58, 61, 62 Relativity, 111
Parameters, 62 Representation, ix, 1, 8, 112
Parametric, 57, 67, 112, 117 Resultant, 9, 48, 49, 50
Particle, 46, 48, 49, 51 Revolution, 76
Path, 112, 113, 115, 117 Right, 60, 66, 186
Perpendicular, 2, 31, 35, 47 Robots, 55
Perpendiculars, 2 Rotation, 64, 65, 89, 90
Physical, 1, 55, 86, 89 Rotational, 89
Physics, 45
Plane, 4, 47, 60, 122 S
Planes, 123, 129, 134, 155 Scalar, ix, 1, 29, 46
Plot, 171, 173, 174, 201 Scalars, 1, 10, 55
Point, 1, 7, 47, 56 Section, 6, 45, 64, 167
Index 211

Segment, 1, 58 Time, 1, 56, 86, 123


Segments, 8, 57 Times, 5, 7, 175, 176
Semicolon, 168, 183, 186 Tons, 25
Shuttle, 25 Torsion, 64, 65, 69, 105
Sinks, 87 Totality, 56
Skyscrapers, 25 Traced, 63
Smoothly, 168 Transformation, 135
Solenoidal, 87, 98, 123 Transformed, 135
Sources, 87 Triangle, 9, 10, 16, 124
Space, 1, 12, 25, 55
Special, 167, 183 U
Speed, 1 Understand, 7, 12, 55, 79
Sphere, 56, 125, 126, 159 Unit, 4, 5, 13, 18
Stationary, 56 Unity, 4, 5
Steady-state, 56
Steel, 25
V
Stokes, ix, vii, 135, 146
Straight, 58, 131, 132 Value, 1, 3, 19, 37
Subset, ix Variable, 55, 58, 115, 167
Subsurfaces, 122 Variables, 115, 168
Subtraction, 6, 168 Vector, 1, 9, 18, 154
Sum, 6, 8, 10, 178 Vectors, 1, 5, 14, 60
Surface, 56, 74, 76, 108 Velocity, 1, 86, 89, 123
Surfaces, 56, 74, 79, 122 Vertex, 13
Syntax, 168 Vertical, 25, 125, 129, 145
System, 2, 4, 55, 60 Vertices, 13
Visualization, 167
Visualize, 55
T Volume, 1, 87, 129, 163
Tangent, 58, 61, 64, 84 Vortex, 90
Tangents, 60, 72, 73
Taylorâs series, 86
Temperature, 1, 56 W
Tensions, 25 Work, 1, 46, 48, 112
Terminal, 1, 7, 9, 12 “Work done”, 45, 48, 49, 122
Theorem, 113, 135, 145, 146
Thermo-dynamical, 55 Z
Three, 2, 4, 12, 50 Zero, 4, 45, 82, 114
Thrust, 1
About the Authors

Nita H. Shah received her PhD in Statistics from Gujarat University in


1994. From February 1990 until now Professor Shah has been Head of
the Department of Mathematics in Gujarat University, India. She is a post-
doctoral visiting research fellow of University of New Brunswick, Canada.
Professor Shah’s research interests include inventory modeling in supply
chains, robotic modeling, mathematical modeling of infectious diseases,
image processing, dynamical systems and their applications, etc. She has
published 13 monograph, 5 textbooks, and 475+ peer-reviewed research
papers. Four edited books have been prepared for IGI-global and Springer
with coeditor Dr. Mandeep Mittal. Her papers are published in high-impact
journals such as those published by Elsevier, Interscience, and Taylor and
Francis. According to Google scholar, the total number of citations is over
3334 and the maximum number of citations for a single paper is over 177.
She has guided 28 PhD Students and 15 MPhil students. She has given talks
in USA, Singapore, Canada, South Africa, Malaysia, and Indonesia. She was
Vice-President of the Operational Research Society of India. She is Vice-
President of the Association of Inventory Academia and Practitioner and a
council member of the Indian Mathematical Society.

Jitendra Panchal is an Assistant Professor in the Department of Applied


Sciences and Humanities, Parul University, India. He has 7+ years of teach-
ing experience and 5+ years of research experience. His research interests
are in the fields of mathematical control theory applied to various types
of impulsive and/or fractional differential inclusions/systems with non-local
conditions, and integer/fractional-order mathematical modeling of dynamical
systems based on real-life phenomena. His 10 articles have been published in
international journals indexed in SCIE, Scopus and Web of Science.

213
River Publishers Series in Mathematical, Statistical and
Elementary Vector Calculus Computational Modelling for Engineering

and its Applications with


MATLAB Programming Elementary Vector Calculus

Elementary Vector Calculus and its Applications with


Nita H. Shah and its Applications with
Jitendra Panchal
MATLAB Programming

MATLAB Programming
Sir Isaac Newton, one of the greatest scientists and mathematicians
of all time, introduced the notion of a vector to define the existence of
gravitational forces, the motion of the planets around the sun, and the
motion of the moon around the earth. Vector calculus is a fundamental
scientific tool that allows us to investigate the origins and evolution of
space and time, as well as the origins of gravity, electromagnetism, and
nuclear forces. Vector calculus is an essential language of mathemati-
cal physics, and plays a vital role in differential geometry and studies
related to partial differential equations widely used in physics, engi-
neering, fluid flow, electromagnetic fields, and other disciplines. ­Vector
calculus represents physical quantities in two or three-­dimensional
space, as well as the variations in these quantities.
The machinery of differential geometry, of which vector calculus
is a subset, is used to understand most of the analytic results in a more
general form. Many topics in the physical sciences can be mathemati-
cally studied using vector calculus techniques.
This book is designed under the assumption that the readers have
no prior knowledge of vector calculus. It begins with an introduction to
vectors and scalars, and also covers scalar and vector products, vector
differentiation and integrals, Gauss’s theorem, Stokes’s theorem, and
Green’s theorem. The MATLAB programming is given in the last chapter.
This book includes many illustrations, solved examples, practice
examples, and multiple-choice questions.

Jitendra Pancha
Nita H. Shah
Nita H. Shah
River Publishers River Jitendra Panchal

You might also like